19952004各地物理高考真题 310页

  • 18.32 MB
  • 2021-05-13 发布

19952004各地物理高考真题

  • 310页
  • 当前文档由用户上传发布,收益归属用户
  1. 1、本文档由用户上传,淘文库整理发布,可阅读全部内容。
  2. 2、本文档内容版权归属内容提供方,所产生的收益全部归内容提供方所有。如果您对本文有版权争议,请立即联系网站客服。
  3. 3、本文档由用户上传,本站不保证质量和数量令人满意,可能有诸多瑕疵,付费之前,请仔细阅读内容确认后进行付费下载。
  4. 网站客服QQ:403074932
2004 年全国普通高等学校招生统一考试(江苏卷) 物 理 第Ⅰ卷(选择题 共 40 分) 一、本题共 10 小慰;每小题 4 分,共 40 分.在每小题给出的四个选项中,有的小题只有一 个选项正确,有的小题有多个选项正确.全部选对的得 4 分,选不全的得 2 分,有选错 或不答的得 0 分. 1.下列说法正确的是 A.光波是—种概率波 B.光波是一种电磁波 C.单色光从光密介质进入光疏介质时.光子的能量改变 D.单色光从光密介质进入光疏介质时,光的波长不变 2.下列说法正确的是 A.物体放出热量,温度一定降低 B.物体内能增加,温度一定升高 C.热量能自发地从低温物体传给高温物体 D.热量能自发地从高温物体传给低温物体 3.下列说法正确的是 A.α射线与γ射线都是电磁波 B.β射线为原子的核外电子电离后形成的电子流 C.用加温、加压或改变其化学状态的方法都不能改变原子核衰变的半衰期 D.原子核经过衰变生成新核,则新核的质量总等于原核的质量 4.若人造卫星绕地球作匀速圆周运动,则下列说法正确的是 A.卫星的轨道半径越大,它的运行速度越大 B.卫星的轨道半径越大,它的运行速度越小 C.卫星的质量一定时,轨道半径越大,它需要的向心力越大 D.卫星的质量一定时,轨道半径越大,它需要的向心力越小 5.甲、乙两个相同的密闭容器中分别装有等质量的同种气体,已知甲、乙容器中气体的压 强分别为 p 甲、p 乙,且 p 甲

0一侧的每个沙袋质量为 m=14 千克,x<0一侧的每个沙袋质量m′=10千克.一质量为M=48千克的小车 以某初速度从原点出发向正x方向滑行.不计轨道阻力.当车每经过一人身旁 时,此人就把沙袋以水平速度u朝与车速相反的方向沿车面扔到车上,u的大 小等于扔此袋之前的瞬间车速大小的2n倍.(n是此人的序号数) (1)空车出发后,车上堆积了几个沙袋时车就反向滑行? (2)车上最终有大小沙袋共多少个? 1995年全国普通高等学校招生统一考试 物理试卷答案及评分标准. 一、答案及评分标准:全题33分,每小题3分.答错的或不答的,都给0分. 1.(A) 2.(C) 3.(B) 4.(A) 5.(D) 6.(B) 7.(A) 8.(D) 9.(D) 10.(C) 11.(B) 二、答案及评分标准:全题35分,每小题5分,每小题全选对的给5分,选对但不 全的给2分,有选错的给0分,不答的给0分. 12.(A、B、C) 13.(B、D) 14.(C、D) 15.(A、B、C) 16.(B、C) 17.(A、C) 18.(C) 三、答案及评分标准:全题42分,其中24、25题各6分,其余的每小题5分.答案 正确的,按下列答案后面括号内的分数给分:答错的,不答的,都给0分. 19.6000……(5分) 20.6……(2分) 1.6×1014……(3分) 22.1.5v……(5分) 23.答案如图16 评分标准:全题5分,所定部位两端只要有一个错,就给零分.没有利用 像只按照入射角等于反射角而估画出的,只给1分. 24.图17所示为正确连线的一种.左偏. 评分标准:全题6分,电路图连接正确给4分,有任何错误都不给这4分. 答出"左偏"给2分. 26.1.2×103欧(3分,写成1.2×103欧或1200欧也给分;不写单位不给分) ×10(2分) 四、参考解答及评分标准 27.解:用h和h′分别表示S和S′移动的距离,用l表示S和S′未移动时 的距离,则有 l=u+v ② 根据透镜成像公式 由①②③式并代入数据可解得 l=64厘米 评分标准:全题6分,写出④式给2分,写出②式给2分,写出③式给1分.得 出正确结果再给1分. 28.解:(1)当两金属杆都以速度v匀速滑动时,每条金属杆中产生的感应 电动势分别为 ε1=ε=Bdv ① 由闭合电路的欧姆定律,回路中的电流强度 因拉力与安培力平衡,作用于每根金属杆的拉力的大小为 F1=F2=IBd ③ 由①②③式并代入数据得 (2)设两金属杆之间增加的距离为△L,则两金属杆共产生的热量 代入数据得 Q=1.28×10-2焦 评分标准:全题10分.第一问6分:求出①式给1分,求出②③式各得2分,结 果正确再给1分.第二问4分:求出④式给3分,结果正确再给1分.若用 Q=F1△L代替④式也同样给分. 29.第一种解法: 设气体最初温度为T0,则活塞刚离开卡环时温度为T0+△T,压强p1.由等 容升温过程得 设气柱高度为H1时温度为T1,由等压升温过程得 设气柱高度为H2时温度为T2,由等温膨胀过程(T2=T1)得 由①和③两式求得 由②和④两式得 代入数字得 T2=540K 评分标准:全题12分.求得①、②、③式各给3分.正确求得⑦式给2分,结 果正确再给1分(若利用①、②、③式得出正确结果而未写⑦式,也给这3 分). 第二种解法: 设气体最初温度为T0,则活塞刚离开卡环时温度为T0+△T0.设气柱高度 为H1时温度为T1,高度为H2时温度为T2. 由等压升温过程得 利用T1=T2,由①、②两式解得 代入数值得 T2=540K 评分标准:全题12分.求得①式给4分;求得②式给5分;正确求得③式给2 分,结果正确再给1分(若利用①、②式得出正确结果而未写③式的,也给 这3分). 30.解: (1)在小车朝正x方向滑行的过程中,第(n-1)个沙袋扔到车上后的车速为 vn-1,第n个沙袋扔到车上后的车速为vn,由动量守恒定律有 小车反向运动的条件是vn-1>0,vn<0,即 M-nm>0 ② M-(n+1)m<0 ③ 代入数字,得 n应为整数,故n=3,即车上堆积3个沙袋后车就反向滑行. (2)车自反向滑行直到接近x<0一侧第1人所在位置时,车速保持不变,而 车的质量为M+3m.若在朝负x方向滑行过程中,第(n-1)个沙袋扔到车上后车 速为vn-1′,第n个沙袋扔到车上后车速为vn′,现取在图中向左的方向(负x方 向)为速度vn′、vn-1′的正方向,则由动量守恒定律有 车不再向左滑行的条件是 vn-1′>0,vn′≤0 即 M+3m-nm′>0 ⑤ M+3m-(n+1)m′≤0 ⑥ n=8时,车停止滑行,即在x<0一侧第8个沙袋扔到车上后车就停住.故车上最 终共有大小沙袋3+8=11个. 评分标准:全题12分.第(1)问4分:求得①式给2分,正确分析车反向滑行条 件并求得反向时车上沙袋数再给2分.(若未求得①式,但求得第1个沙袋扔到 车上后的车速,正确的也给2分.通过逐次计算沙袋扔到车上后的车速,并求 得车开始反向滑行时车上沙袋数,也再给2分.) 第(2)问8分:求得④式给3分,⑤式给1分,⑥式给2分.求得⑦式给1分.得到 最后结果再给1分.(若未列出⑤、⑥两式,但能正确分析并得到左侧n=8的结 论,也可给上述⑤、⑥、⑦式对应的4分.) 1996年全国普通高等学校招生统一考试 (全国卷) 一、本题共8小题;每小题4分,共32分.在每小题给出的四个选项中只有一项 是正确的. 1.下列核反应方程式中,表示核聚变过程的是( ). 2.红、橙、黄、绿四种单色光中,光子能量最小的是( ). (A)红光 (B)橙光 (C)黄光 (D)绿光 3.一平面线圈用细杆悬于P点,开始时细杆处于水平位置,释放后让它在 如图所示的匀强磁场中运动.已知线圈平面始终与纸面垂直,当线圈第 一次通过位置Ⅰ和位置Ⅱ时,顺着磁场的方向看去,线圈中感应电流的 方向分别为( ) 位置Ⅰ 位置Ⅱ (A)逆时针方向 逆时针方向 (B)逆时针方向 顺时针方向 (C)顺时针方向 顺时针方向 (D)顺时针方向 逆时针方向 4.只要知道下列哪一组物理量,就可以估算出气体中分子间的平均距 离?( ). (A)阿伏伽德罗常数、该气体的摩尔质量和质量 (B)阿伏伽德罗常数、该气体的摩尔质量和密度 (C)阿伏伽德罗常数、该气体的质量和体积 (D)该气体的密度、体积和摩尔质量 5.根据玻尔理论,氢原子的电子由外层轨道跃迁到内层轨道后( ). (A)原子的能量增加,电子的动能减少 (B)原子的能量增加,电子的动能增加 (C)原子的能量减少,电子的动能减少 (D)原子的能量减少,电子的动能增加 6.在右图所示的实验装置中,平行板电容器的极板A与一灵敏的静电计 相接,极板B接地.若极板B稍向上移动一点,由观察到的静电计指针变 化作出平行板电容器电容变小的结论的依据是( ). (A)两极板间的电压不变,极板上的电量变小 (B)两极板间的电压不变,极板上的电量变大 (C)极板上的电量几乎不变,两极板间的电压变小 (D)极板上的电量几乎不变,两极板间的电压变大 7.一焦距为f的凸透镜,主轴和水平的x轴重合.x轴上有一光点位于透镜 的左侧,光点到透镜的距离大于f而小于2f.若将此透镜沿x轴向右平移 2f的距离,则在此过程中,光点经透镜所成的象点将( ) (A)一直向右移动 (B)一直向左移动 (C)先向左移动,接着向右移动 (D)先向右移动,接着向左移动 8.质量为1.0千克的小球从高20米处自由下落到软垫上,反弹后上升的最 大高度为5.0米.小球与软垫接触的时间为1.0秒,在接触时间内小球受 到合力的冲量大小为( ).(空气阻力不计,g取10米/秒2) (A)10牛·秒 (B)20牛·秒 (C)30牛·秒 (D)40牛·秒 二、本题共6小题;每小题6分,共36分.在每小题给出的四个选项中,至少有一 项是正确的.全部选对的得6分,选对但不全的得2分,有选错或不答的得0分. 9.一物体作匀变速直线运动,某时刻速度的大小为4米/秒,1秒钟后速度 的大小变为10米/秒.在这1秒钟内该物体的( ). (A)位移的大小可能小于4米 (B)位移的大小可能大于10米 (C)加速度的大小可能小于4米/秒2 (D)加速度的大小可能大于10米/秒2 10.LC回路中电容两端的电压u随时刻t变化的关系如下图所示,则( ). (A)在时刻t1,电路中的电流最大 (B)在时刻t2,电路的磁场能最大 (C)从时刻t2至t3,电路的电场能不断增大 (D)从时刻t3至t4,电容的带电量不断增大 11.如图a,b,c是一条电力线上的三个点,电力线的方向由a到c,a、b间的 距离等于b、c间的距离。用Ua、Ub、Uc和Ea、Eb、Ec分别表示a、b、 c三点的电势和电场强度,可以断定( ). (A)UaUbUc (B)EaEbEc (C)Ua-Ub=Ub-Uc (D)Ea=Eb=Ec 12.一根张紧的水平弹性长绳上的a、b两点,相距14.0米,b点在a点的右方. 当一列简谐横波沿此长绳向右传播时,若a点的位移达到正极大时,b 点的位移恰为零,且向下运动.经过1.00秒后,a点的位移为零,且向下运 动,而b点的位移恰达到负极大,则这简谐横波的波速可能等于( ). (A)4.67米/秒 (B)6米/秒 (C)10米/秒 (D)14米/秒 13.半径相等的两个小球甲和乙,在光滑水平面上沿同一直线相向运动. 若甲球的质量大于乙球的质量,碰撞前两球的动能相等,则碰撞后两 球的运动状态可能是( ). (A)甲球的速度为零而乙球的速度不为零 (B)乙球的速度为零而甲球的速度不为零 (C)两球的速度均不为零 (D)两球的速度方向均与原方向相反,两球的动能仍相等 14. 如果表中给出的是作简谐振动的物体的位移x或速度v与时刻的对应关 系,T是振动周期,则下列选项中正确的是( ). (A)若甲表示位移x,则丙表示相应的速度v (B)若丁表示位移x,则甲表示相应的速度v (C)若丙表示位移x,则甲表示相应的速度v (D)若乙表示位移x,则丙表示相应的速度v 三、本题共3小题;其中第15题5分,其余的每题6分,共17分.把答案填在题中的 横线上或按题目要求作图. 15.在"验证机械能守恒定律"的实验中,已知打点计时器所用电源的频 率为50赫.查得当地的重力加速度g=9.80米/秒2.测得所用的重物的 质量为1.00千克.实验中得到一条点迹清晰的纸带,把第一个点记作0, 另选连续的4个点A、B、C、D作为测量的点.经测量知道A、B、C、 D各点到0点的距离分别为62.99厘米、70.18厘米、77.76厘米、85.73 厘米.根据以上数据,可知重物由0点运动到C点,重力势能的减少量等 于 焦,动能的增加量等于 焦(取3位有效数字). 16.在用电流场模拟静电场描绘电场等势线的实验中,在下列所给出的 器材中,应该选用的是 (用器材前的字母表示). (A)6伏的交流电源 (B)6伏的直流电源 (C)100伏的直流电源 (D)量程0~0.5伏,零刻度在刻度盘中央的电压表 (E)量程0~300微安,零刻度在刻度盘中央的电流表在实验过程中,要 把复写 纸、导电纸、白纸铺放在木板上,它们的顺序(自上而下)是① ② ③ . 在实验中,按下电键,接通电路.若一个探针与基准点O接触,另 一探针已分别在基准点O的两侧找到了实验所需要的两点a、b(如 右上图),则当此探针与a点接触时,电表的指针应 (填"左偏"、" 指零"或"右偏");当此探针与b点接触时,电表的指针应 (填"左偏"、"指零"或"右偏"). 17.在用伏安法测电阻的实验中,所用电压表的内阻约为20千欧,电流表 的内阻约为10欧,选择能够尽量减小误差的电路图接线进行实验,读 得的各组数据用实心圆点标于坐标图上(如右图所示). (1)根据各点表示的数据描出I-U图线,由此求得该电阻的阻值 Rx= 欧(保留两位有效数字). (2)画出此实验的电路原理图. 四、本题共4小题;每小题5分,共20分.把答案填在题中的横线上,或按题目要 求作图. 18.如右图所示,一细导体杆弯成四个拐角均为直角的平面折线,其ab、cd 段长度均为l1,bc段长度为l2.弯杆位于竖直平面内,Oa、dO′段由轴承 支撑沿水平放置.整个弯杆置于匀强磁场中,磁场方向竖直向上,磁感 应强度为B.今在导体杆中沿abcd通以大小为I的电流,此时导体杆受 到的安培力对OO′轴的力矩大小等于 . 19.右图中abcd为一边长为l、具有质量的刚性导线框,位于水平面内,bc 边中串接有电阻R,导线的电阻不计.虚线表示一匀强磁场区域的边 界,它与线框的ab边平行.磁场区域的宽度为2l,磁感应强度为B,方向 竖直向下.线框在一垂直于ab边的水平恒定拉力作用下,沿光滑水平 面运动,直到通过磁场区域.已知ab边刚进入磁场时,线框便变为匀速 运动,此时通过电阻R的电流的大小为i0,试在右图的i-x坐标上定性画 出:从导线框刚进入磁场到完全离开磁场的过程中,流过电阻R的电 流i的大小随ab边的位置坐标x变化的曲线. 20.如图所示,倔强系数为k1的轻质弹簧两端分别与质量为m1、m2的物块 1、2拴接,倔强系数为k2的轻质弹簧上端与物块2拴接,下端压在桌面 上(不拴接),整个系统处于平衡状态.现施力将物块1缓慢竖直上提,直 到下面那个弹簧的下端刚脱离桌面.在此过程中,物块2的重力势能增 加了 ,物块1的重力势能增加了 . 21.在光滑水平面上有一静止的物体.现以水平恒力甲推这一物体,作用 一段时间后,换成相反方向的水平恒力乙推这一物体.当恒力乙作用 时间与恒力甲作用时间相同时,物体恰好回到原处,此时物体的动能 为32焦,则在整个过程中,恒力甲做的功等于 焦,恒力乙做的功等 于 焦. 五、本题共5小题,45分.解答应写出必要的文字说明、方程式和重要演算步 骤.只写出最后答案的不能得分.有数值计算的题,答案中必须明确写出数值 和单位. 22.(5分)一物块从倾角为θ、长为s的斜面的项端由静止开始下滑,物块 与斜面的滑动摩擦系数为μ,求物块滑到斜面底端所需的时间. 23.(8分)在折射率为n、厚度为d的玻璃平板上方的空气中有一点光源S, 从S发出的光线SA以角度θ入射到玻璃板上表面,经过玻璃板后从下表面射 出,如右图所示.若沿此光线传播的光从光源到玻璃板上表面的传播时间与 在玻璃板中的传播时间相等,点光源S到玻璃上表面的垂直距离l应是多少? 24.(8分)一质量为M的长木板静止在光滑水平桌面上.一质量为m的小滑 块以水平速度v0从长木板的一端开始在木板上滑动,直到离开木板.滑块 25.(12分)如图所示,有一个直立的气缸,气缸底到气缸口的距离为L0厘 米,用一厚度和质量均可忽略不计的刚性活塞A,把一定质量的空气封在气缸 内,活塞与气缸间的摩擦可忽略.平衡时活塞上表面与气缸口的距离很小(计 算时可忽略不计),周围大气的压强为H0厘米水银柱.现把盛有水银的一个瓶 子放在活塞上(瓶子的质量可忽略),平衡时活塞到气缸底的距离为L厘米.若 不是把这瓶水银放在活塞上,而是把瓶内水银缓缓不断地倒在活塞上方,这 时活塞向下移,压缩气体,直到活塞不再下移.求此时活塞在气缸内可能的位 置以及与之相对应的条件(即题中给出量之间应满足的关系).设气体的温度 不变. 26.(12分)设在地面上方的真空室内存在匀强电场和匀强磁场.已知电场 强度和磁感应强度的方向是相同的,电场强度的大小E=4.0伏/米,磁感应强度 的大小B=0.15特.今有一个带负电的质点以v=20米/秒的速度在此区域内沿 垂直场强方向做匀速直线运动,求此带电质点的电量与质量之比q/m以及磁 场的所有可能方向(角度可用反三角函数表示). 参考解答 一、1.B 2.A 3.B 4.B 5.D 6.D 7.C 8.C 二、9.A、D 10.B、C 11.A 12.A、C 13.A、C 14.A、B 三、15.7.62,7.56 16.B、E ①导电纸 ②复写纸 ③白纸 指零,指零 17.舍去不合理点的直线,如下图,2.4×103 四、18.IBl1l2 19.x (0→l) i=i0 (l→2l) i=0 (2l→3l)如图,且x=3l处i≥I0 21.8,24 五、22.设物块质量为m,加速度为a,物块受力情况如下图所示, mgsinθ-f=ma, N-mgcosθ=0, f=μN, 解得 a=gsinθ-μgcosθ, 23.设光线在玻璃中的折射角为r,则光线从S到玻璃板上表面的传播 由折射定律 sinθ=nsinr, 24.设第一次滑块离开时木板速度为v,由系统的动量守恒,有 设滑块与木板间摩擦力为f,木板长L,滑行距离s,如右图,由动能定理 25.设整瓶水银放在活塞上后,使气缸内气体增加的压强为h厘米水银柱, 由玻意耳-马略特定律H0L0=(H0+h)L, (1) h的大小反映了水银质量的大小. 当水银注入后,活塞不再下移时,设活塞上水银的深度为△H厘米,活 塞下移的距离为△x厘米,则由玻意耳-马略特定律 H0L0=(H0+△H)(L0-△x), (3) 可能发生两种情况: 1.水银比较少,瓶内水银全部注入后,尚未灌满或刚好灌满活塞上方的 气缸,这时 △H=h, (5) △H≤△x, (6) 由(2)、(4)、(5)三式,得 △x=L0-L, (7) 活塞到气缸底的距离 L′=L0-△x=L, (8) 由(4)、(6)、(7)三式,得 L≥H0, (9) 即若L≥H0,则L′=L. 2.瓶内水银比较多,当活塞上方的气缸灌满水银时,瓶内还剩有一定量 的水银,这时 △H=△x, (10) △H2a (D)a'=2a 4.(1)、(2)两电路中,当a、b两端与e、f两端分别加上220伏的交流电压时,测 得c、d间与g、h间的电压均为110伏.若分别在c、d两端与g、h两端加上110 伏的交流电压,则a、b间与e、f间的电压分别为 (A)220伏,220伏 (B)220伏,110伏 (C)110伏,110伏 (D)220伏,0 5.在双缝干涉实验中,以白光为光源,在屏幕上观察到了彩色干涉条纹,若在 双缝中的一缝前放一红色滤光片(只能透过红光),另一缝前放一绿色滤光 片(只能透过绿光),这时 (A)只有红色和绿色的双缝干涉条纹,其它颜色的双缝干涉条纹消失 (B)红色和绿色的双缝干涉条纹消失,其它颜色的双缝干涉条纹依然存 在 (C)任何颜色的双缝干涉条纹都不存在,但屏上仍有光亮 (D)屏上无任何光亮 二.本题共9小题;每小题5分,共45分.在每小题给出的四个选项中,有的小题 只有一个选项正确,有的小题有多个选项正确.全部选对的得5分,选不全 的得2分,有选错或不答的得0分. 6.在下列核反应方程中,x代表质子的方程是 7.光线在玻璃和空气的分界面上发生全反射的条件是 (A)光从玻璃射到分界面上,入射角足够小 (B)光从玻璃射到分界面上,入射角足够大 (C)光从空气射到分界面上,入射角足够小 (D)光从空气射到分界面上,入射角足够大 8.在下列叙述中,正确的是 (A)物体的温度越高,分子热运动越剧烈,分子平均动能越大 (B)布朗运动就是液体分子的热运动 (C)对一定质量的气体加热,其内能一定增加 (D)分子间的距离r存在某一值r0,当rr0时,斥力 小于引力 9.图中重物的质量为m,轻细线AO和BO的A、B端是固定的.平衡时AO是水平 的,BO与水平面的夹角为θ.AO的拉力F1和BO的拉力F2的大小是 10.为了增大LC振荡电路的固有频率,下列办法中可采取的是 (A)增大电容器两极板的正对面积并在线圈中放入铁芯 (B)减小电容器两极板的距离并增加线圈的匝数 (C)减小电容器两极板的距离并在线圈中放入铁芯 (D)减小电容器两极板的正对面积并减少线圈的匝数 11.简谐横波某时刻的波形图线如图所示.由此图可知 (A)若质点a向下运动,则波是从左向右传播的 (B)若质点b向上运动,则波是从左向右传播的 (C)若波从右向左传播,则质点c向下运动 (D)若波从右向左传播,则质点d向上运动 12.如图所示的电路中,电源的电动势恒定,要想使灯泡变暗,可以 (A)增大R1 (B)减小R1 (C)增大R2 (D)减小R2 13.如图所示的电路中,A1和A2是完全相同的灯泡,线圈L的电阻可以忽略.下 列说法中正确的是 (A)合上开关K接通电路时,A2先亮,A1后亮,最后一样亮 (B)合上开关K接通电路时,A1和A2始终一样亮 (C)断开开关K切断电路时,A2立刻熄灭,A1过一会儿才熄灭 (D)断开开关K切断电路时,A1和A2都要过一会儿才熄灭 14.在图示电路的三根导线中,有一根是断的,电源、电阻器R1、R2及另外两 根导线都是好的.为了查出断导线,某学生想先将万用表的红表笔连接在 电源的正极a,再将黑表笔分别连接在电阻器R1的b端和R2的c端,并观察万 用表指针的示数.在下列选挡中,符合操作规程的是 (A)直流10V挡 (B)直流0.5A挡 (C)直流2.5V挡 (D)欧姆挡 第Ⅱ卷 (非选择题 共90分) 注意事项: 1.第Ⅱ卷共8页,用钢笔或圆珠笔直接答在试题卷中(除题目有特殊规定 外). 2.答卷前将密封线内的项目填写清楚. 三.本题共3小题;其中第15题5分,其余的每题6分,共17分.把答案填在题中的 横线上或按题目要求作图. 15.一游标卡尺的主尺最小分度为1毫米,游标上有10个小等分间隔,现用此 卡尺来测量工件的直径,如图所示.该工件的直径为_____________毫米. 16.下列给出的器材中,哪些是"验证玻一马定律实验"所必需的,把这些器材 前面的字母填在横线上. A.带有刻度的注射器 B.刻度尺 C.弹簧秤 D.钩码若干个 答:_____________. 实验读数过程中,不能用手握住注射器,这是为了________________. 用橡皮帽封住注射器小孔,这是为了___________________________. 17.某电压表的内阻在20千欧~50千欧之间,现要测量其内阻,实验室提供下 列可选用的器材: 待测电压表V(量程3V) 电流表A1(量程200μA ) 电流表A2(量程5mA) 电流表A3(量程0.6A) 滑动变阻器R(最大阻值1KΩ) 电源ε(电动势4V) 电键K. (1)所提供的电流表中,应选用_______________________(填写字母代 号). (2)为了尽量减小误差,要求测多组数据.试在方框中画出符合要求的实 验电路图(其中电源和电键及其连线已画出). 四.本题共4小题;每小题5分,共20分.把答案填在题中横线上. 18.如图,在x轴的上方(y≥0)存在着垂直于纸面向外的匀强磁场,磁感应强度 为B.在原点O有一个离子源向x轴上方的各个方向发射出质量为m、电量 为q的正离子,速率都为v.对那些在xy平面内运动的离子,在磁场中可能到 达的最大x=________________,最大y=________________. 19.质量为m、电量为q的质点,在静电力作用下以恒定速率v沿圆弧从A点运 动到B点,其速度方向改变的角度为θ(弧度),AB弧长为s.则A,B两点间的 电 势 差 UA-UB=________________,AB 弧 中 点 的 场 强 大 小 E=________________. 20.已知地球半径约为6.4×106米,又知月球绕地球的运动可近似看作匀速圆 周运动,则可估算出月球到地心的距离约为________________米.(结果只 保留一位有效数字) 21.一内壁光滑的环形细圆管,位于竖直平面内,环的半径为R(比细管的半径 大得多).在圆管中有两个直径与细管内径相同的小球(可视为质点).A球 的质量为m1,B球的质量为m2.它们沿环形圆管顺时针运动,经过最低点时 的速度都为v0.设A球运动到最低点时,B球恰好运动到最高点,若要此时 两球作用于圆管的合力为零,那么m1,m2,R与v0 应满足的关系式是 _________________________. 五.本题共5小题,53分.解答应写出必要的文字说明、方程式和重要演算步骤. 只写出最后答案的不能得分.有数值计算的题,答案中必须明确写出数值 和单位. 22.(9分)有一个焦距为36厘米的凸透镜,在主轴上垂直放置一支蜡烛,得到一 个放大率为4的虚像.如果想得到放大率为4的实像,蜡烛应向哪个方向移动? 移动多少? 23.(9分)图中竖直圆筒是固定不动的,粗筒横截面积是细筒的4倍,细筒足够 长.粗筒中A、B两轻质活塞间封有空气,气柱长l=20厘米.活塞A上方的水银深 H=10厘米,两活塞与筒壁间的摩擦不计.用外力向上托住活塞B,使之处于平 衡状态,水银面与粗筒上端相平.现使活塞B缓慢上移,直至水银的一半被推 入细筒中,求活塞B上移的距离.设在整个过程中气柱的温度不变,大气压强 p0相当于75厘米高的水银柱产生的压强. 24.(11分)在方向水平的匀强电场中,一不可伸长的不导电细线的一端连着一 个质量为m的带电小球,另一端固定于O点.把小球拉起直至细线与场强平行, 然后无初速释放.已知小球摆到最低点的另一侧,线与竖直方向的最大夹角 为θ(如图).求小球经过最低点时细线对小球的拉力. 25.(12分)质量为m的钢板与直立轻弹簧的上端连接,弹簧下端固定在地上.平 衡时,弹簧的压缩量为x0,如图所示.一物块从钢板正上方距离为3x0的A处自 由落下,打在钢板上并立刻与钢板一起向下运动,但不粘连.它们到达最低点 后又向上运动.已知物块质量也为m时,它们恰能回到O点.若物块质量为2m, 仍从A处自由落下,则物块与钢板回到O点时,还具有向上的速度.求物块向上 运动到达的最高点与O点的距离. 26.(12分)如图1所示,真空室中电极K发出的电子(初速不计)经过U0=1000伏 的加速电场后,由小孔S沿两水平金属板A、B间的中心线射入.A、B板长 l=0.20米,相距d=0.020米,加在A、B两板间的电压u随时间t变化的u-t图线如图 2所示.设A、B间的电场可看作是均匀的,且两板外无电场.在每个电子通过电 场区域的极短时间内,电场可视作恒定的.两板右侧放一记录圆筒,筒在左侧 边缘与极板右端距离b=0.15米,筒绕其竖直轴匀速转动,周期T=0.20秒,筒的 周长s=0.20米,筒能接收到通过A、B板的全部电子. (1)以t=0时(见图2,此时u=0)电子打到圆筒记录纸上的点作为xy坐标系 的原点,并取y轴竖直向上.试计算电子打到记录纸上的最高点的y坐 标和x坐标.(不计重力作用) (2)在给出的坐标纸(图3)上定量地画出电子打到记录纸上的点形成的图 线. 图1 图2 图3 1997年普通高等学校招生全国统一考试 物理试题答案及评分标准 说明: (1)定出评分标准是为了使全国各地尽可能在统一标准下评定成绩.试题 的参考解答是用来说明评分标准的.考生如按其它方法或步骤解答,正确的, 同样给分;有错的,根据错误的性质,参照评分标准中相应的规定评分. (2)第一、二、三、四题只要求写出答案,不要求说明理由或列出算式, 只根据答案评分. (3)第五大题,只有最后后答案而无演算过程的,不给分;只写出一般公式 但未能与试题所给的具体条件联系的,不给分. 一.答案及评分标准:全题15分,每小题3分.答错的或不答的,都给0分. 1.A 2.D 3.C 4.B 5.C 二.答案及评分标准:全题45分,每小题5分.每小题全选对的给5分,选不全的 给2分,有选错的给0分,不答的给0分. 6.B、C 7.B 8.A、D 9.B、D 10.D 11.B、D 12.A、D 13.A、D 14.A 三.答案及评分标准:全题17分,其中15题5分,其余的每题6分.答案正确的,按 下列答案后面括号内的分数给分;答错的,不答的,都给0分. 15.29.80 (5分,答29.8的同样给5分) 16.A,B,C,D (2分.选不全的给0分) 保持气体的温度恒定 (2分) 保持气体的质量不变 (2分) 17.A1 (2分), 如右图 (4分,线路有错就不给这4分) 四.答案及评分标准:全题20分,每小题5分,答案正确的,按下列答案后面括号 内的分数给分;答错的,不答的,都给0分. 20.4×108 (5分.只要数量级对,就给5分) 五.参考解答及评分标准: 22.解:先求蜡烛的原位置 由放大率公式 得 v1=4u1 ① 由透镜成像公式 ② 解得 再求蜡烛移动后的位置,由放大率公式得 v2=4u2③ 由透镜成像公式 ④ 解得 所以蜡烛应向远离透镜的方向移动,移动的距离为 评分标准:本题9分. ①式2分,②式1分,③式2分,④式1分,⑤式2分. 物体移动方向正确的给1分. 23.解: 在以下的计算中,都以1厘米汞柱产生的压强作为压强的单位. 设气体初态的压强为p1,则有 p1=p0+H ① 设S为粗圆筒的横截面积,气体初态的体积V1=Sl. 设气体末态的压强为P2,有 ② 设末态气柱的长度为l',气体体积为V2=Sl' 由玻意耳定律得 P1V1=P2V2③ 活塞B上移的距离d为 ④ 代入数据解得 d=8厘米 ⑤ 评分标准:本题9分. ①式1分,②式2分,③式1分,④式3分,⑤式2分. 24.解:设细线长为l,球的电量为q,场强为E.若电量q为正,则场强方向在题图 中向右,反之向左.从释放点到左侧最高点,重力势能的减少等于电势能的增 加, mglcosθ=qEl(1+sinθ)① 若小球运动到最低点时的速度为v,此时线的拉力为T,由能量关系得 ② 由牛顿第二定律得 ③ 由以上各式解得 ④ 评分标准:本题11分. ①、②式各3分,③式2分,④式3分. 25.解:物块与钢板碰撞时的速度 ① 设v1表示质量为m的物块与钢板碰撞后一起开始向下运动的速度,因碰 撞时间极短,动量守恒, mv0=2mv1 ② 刚碰完时弹簧的弹性势能为EP.当它们一起回到O点时,弹簧无形变,弹 性势能为零,根据题给条件,这时物块与钢板的速度为零,由机械能守恒, ③ 设v2表示质量为2m的物块与钢板碰撞后开始一起向下运动的速度,则有 2mv0=3mv2 ④ 仍继续向上运动,设此时速度为v,则有 ⑤ 在以上两种情况中,弹簧的初始压缩量都是x0,故有 ⑥ 当质量为2m的物块与钢板一起回到O点时,弹簧的弹力为零,物块与钢 板只受到重力作用,加速度为g.一过O点,钢板受到弹簧向下的拉力作用,加速 度大于g.由于物块与钢板不粘连,物块不可能受到钢板的拉力,其加速度仍 为g.故在O点物块与钢板分离,分离后,物块以速度v竖直上升,则由以上各式 解得,物块向上运动所到最高点与O点的距离为 ⑦ 评分标准:本题12分. ①、②、③、④式各1分,⑤式2分,⑥式3分,得出⑦式再给3分. 26.解:(1)计算电子打到记录纸上的最高点的坐标 设v0为电子沿A、B板的中心线射入电场时的初速度,则 ① 电子在中心线方向的运动为匀速运动,设电子穿过A、B板的时间为t0, 则 l=v0t0 ② 图(1) 电子在垂直A、B板方向的运动为匀加速直线运动.对于恰能穿过A、B 板的电子,在它通过时加在两板间的电压uc应满足 ③ 联立①、②、③式解得 此电子从A、B板射出时沿y方向的分速度为 ④ 此后,此电子作匀速直线运动,它打在记录纸上的点最高,设纵坐标为y,由 图(1)可得 ⑤ 由以上各式解得 ⑥ 从题给的ut图线可知,加于两板电压u的周期T0=0.10秒,u的最大值 um=100伏,因为uCrB,则 A.在空气中 A 的波长大于 B 的波长 B.在水中 A 的传播速度大于 B 的传播速度 C.A 的频率大于 B 的频率 D.在水中 A 的波长小于 B 的波长 9.图中 A 为理想电流表,V1 和 V2 为理想电压表,R1 为定值电阻,R2 为可变电阻,电池 E 内 阻不计,则 A.R2 不变时,V2 读数与 A 读数之比等于 R1 B.R2 不变时, V1 读数与 A 读数之比等于 R1 C.R2 改变一定量时,V2 读数的变化量与 A 读数的变化量之 比的绝对值等于 R1 D.R2 改变一定量时,V1 读数的变化量与 A 读数的变化量之比的绝对值等于 R1 10.图为一空间探测器的示意图,P1、P2、P3、P4 是四个喷气发动机,P1、P3 的连线与空间 一固定坐标系的 x 轴平行,P2、P4 的连线与 y 轴平行。每台发动机开动时,都能向探测器 提供推力,但不会使探测器转动,开始时, 探测器以恒定的速率 v0 向正 x 方向平动。要 使探测器改为向正 x 偏负 y60°的方向以原 来的速率 v0 平动,则可 A.先开动 P1 适当时间,再开动 P4 适当时间 B.先开动 P3 适当时间,再开动 P2 适当时间 C.开动 P4 适当时间 D.先开动 P3 适当时间,再开动 P4 适当时间 第Ⅱ卷(非选择题 共 110 分) 二、本题共 3 小题;每小题 5 分,共 15 分。把答案填在题中的横线上。 11.裂变反应是目前核能利用中常用的反应。以原子核 为燃料的反应堆中,当 U 俘 获一个慢中子后发生的裂变反应可以有多种方式,其中一种 可表示为 十 一 → Xe + Sr + 3 235.0439 1.0087 138.9178 93.9154 反应方程下方的数字是中子及有关原子的静止质量(以 原子质量单位 u 为单位).已知 lu 的质量对应的能量为 9.3 ×102 MeV,此裂变反应释放 出的能量是 MeV。 12.空间存在以 ab、cd 为边界的匀强磁场区域,磁感强度大小为凡方向垂直纸面向外,区 域宽为 l1、现有一矩形线框处在图中纸面内,它的短边与 ab 重合,长度为 l2 长边的长度为 2l1,如图所示。某时刻线框以初速 v 沿与 ab 垂直的方向进入磁场区域,同时某人对线框施 以作用力,使它的速度大小和方向保持 不变,设该线框的电阻为 R。从线框开始进入磁场 到完全离开磁场的过程中,人对线框作用力所做的功等于___________________________. 13.假设在 NaCl 蒸气中存在由钠离子 Na+和氯离子 Cl-靠静电相互作用构成的单个氯化 钠 NaCl 分子。 若取 Na+与 Cl-相距无限远时其电势能力零,一个 NaCl 分子的电势能为 一 6.1eV。已 知使一个中性钠原子 Na 最外层的电子脱离钠原子而形成钠离子 Na+所需的 能量(电离能) 为 5.1 eV,使一个中性氯原子 Cl 结合一个电子形成氯离子 Cl-所放出的能 量(亲和能) 为 3.8 eV。由此可算出,在将一个 NaCl 分子分解成彼此远离的中性钠原子 Na 和中性氯原 子 Cl 的过程中,外界供给的总能量等于____________eV。 三、本题共 3 小题,共刀分,把答案填在题中的横线上或按题目要求作图。 14.(6 分)某同学用图 1 所示装置通过半径相同的 A、B 两球的碰撞来验证动量守恒定律。图中 PQ 是斜槽,以 为水平槽,实验时先使 A 球从斜槽上某一固定位置 G 由静止开始滚下,落到位于水平地面的记录纸上,留 下痕迹。重复上述操作 10 次,得到 10 个落点痕迹。 再把 B 球放在水平槽上靠近槽末端的地方,让 A 球仍从位置 G 由静止开始滚下,和下球碰 撞后,A、B 球分别在记录纸上留下各自的落点痕迹,重复这种操作 10 次。图:中 0 点是水 平槽未 端 R 在记录纸上的垂直投影点,B 球落点痕迹如图 2 所示,其中米尺水平放置,且 平行于 G、R、O 所在的平面,米尺的零点与 O 点对齐。 (1)碰撞后 B 球的水平射程应取为________cm。 (2)在以下选项中,哪些是本次实验必须进行的测量?答_________(填选项号)。 A.水平槽上未放日球时,测量人球落点位置到 O 点的距离 B.A 球与 B 球碰撞后,测量 A 球落点位置到 O 点的距离 C.测量 A 球或 B 球的直径 D.测量 A 球和 B 球的质量(或两球质量之比) E.测量 G 点相对于水平槽面的高度 15.(6 分)如图,一光源位于金属圆筒内部轴线上 A 点,与筒 B 端的距离为 d,d 无法直 接测量。另有凸透镜、光屏、米尺及带支架的光具座。现用这些器材测量 d。为此,先 将圆筒、凸透镜、光屏依次放在光具座支架上,令圆筒轴线与透镜主光轴重合,屏与江 源的距离足够远,使得移动透镜时,可在屏上两次出现光源的象。将圆筒及光屏位置固 定。由光路的可逆性可知,第一次成象的象距等于第二次成象的象距。然后进行以下的 测量: __________________________ __________________________ __________________________ __________________________. 用测得的物理量可得 d=________________________________________.(应说明各符号所代表的物理量) 16.(8 分)从下表中选出适当的实验器材,设计一电路来测量电流表 A1 的内阻 r1,要求方 法简捷,有尽可能高的测量精度,并能测得多组数据。 (1)在虚线方框中画出电路图,标明所用器材的代号。 (2)若选测量数据中的一组来计算 r1,则所用的表达式为 r1= ________________,式 中各符号的意义是: ________________________________________________________________. 四、本题共 6 小题,75 分。解答应写出必要的文字说明、方程式和重要演算步骤,只写出 最后答案的不得分。有数值计算的题,答案中必须明确写出数值和单位。 17.(11 分)一横截面积为 S 的气缸水平放置,固定不动。气缸壁是导热的。两个活塞 A 和 B 将气缸分融为 1、2 两气室,达到平衡时 1、2 两气室体 积之比为 3:2,如图所示,在室温不变的条件下,缓慢推动 活塞 A,使之向右移动一段距离 d。求活塞 B 向右移动的距 离。不计活塞与气缸壁之间的摩擦。 18.(2 分)一小型发电机的矩形线圈的匀强磁场中以恒定的角速度ω绕垂直于磁场方向的 固定轴转动。线圈匝数 n=100.穿过每匝线圈的磁通φ随时间按正弦规律变化,如图 所示,发电机内阻 r=5.0Ω,外 电路电阻 R=95Ω。已知感应电动 势的最大值 Em=nωφm,其中φm 为穿过每匝线圈磁通量的最大 值。求串联在外电路中的交流电流表(内阻不计)的读数。 19.(13 分)面积很大的水池,水深为 H,水面上浮着一正方体木块。木块边长为 a,密度 为水的 1/2,质量为 m。开始时,木块静止,有一半没入水中,如图所示。现用力 F 将 木块缓慢地压到池底。不计摩擦。求 (1)从木块刚好完全没入水中到停在池底的过程中,池水势能的改变量。 (2)从开始到木块刚好完全没入水的过程中,力 F 所做的功。 20.(12 分)2000 年 1 月 26 日我国发射了一颗同步卫星,其定点位置与东经 98°的经线在 同一平面内。若把甘肃省嘉峪关处的经度和纬度近似取为东经 98°和北纬α=40°, 已知地球半径 R、地球自转周期 T、地球表面重力加速度 g(视为常量)和光速 c。试求 该同步卫星发出的微波信号传到嘉峪关处的接收站所需的时间(要求用题给的已知量的 符号表示)。 21.(13 分)图示为一种可用于测量电子电量 e 与质量 m 比例 e/m 的阴极射线管,管内处于 真空状态。图中 L 是灯丝,当接上电源时可发出电子。A 是中央有小圆孔的金属板,当 L 和 A 间加上电压时(其电压值比灯丝电压大很多),电子将被加速并沿图中虚直线所示 的路径到达荧光屏 S 上的 O 点,发出荧光。P1、P2 为两块平行于虚直线的金属板,已知 两板间距为 d。在虚线所示的圆形区域内可施加一匀强磁场,已知其磁感强度为 B,方 向垂直纸面向外。a、b1、b2、c1、c2 都是固定在管壳上的金属引线,E1、E2、E3 是三个电 压可调并可读出其电压值的直流电源。 (1)试在图中画出三个电源与阴极射线管的有关引线的连线。 (2)导出计算 e/m 的表达式。要求用应测物理量及题给已知量表示。 22.(14 分)在原子核物理中,研究核子与核子关联的最有效途径是“双电荷交换反应”, 这类反应的前半部分过程和下述力学模型类似。两个小球 A 和 B 用轻质弹簧相连,在光 滑的水平直轨道上处于静止状态。在它们左边有一垂直于轨道的固定挡板 P,右边有一 小球 C 沿轨道以速度 v0 射向 B 球,如图所示。C 与 B 发生碰撞并立即结成一个整体 D。 在它们继续向左运动的过程中,当弹簧长度变到最短时,长度突然被锁定,不再改变。 然后,A 球与挡板 P 发生碰撞,碰后 A、D 都静止不动,A 与 P 接触而不粘连,过一段时 间,突然解锁定(锁定及解除锁定均无机械能损失)。已知 A、B、C 三球的质量均为 m。 (1)求弹簧长度刚被锁定后 A 球的速度。 (2)求在 A 球离开挡板 P 之后的运动过程中,弹簧的最大弹性势能。 2000 年全国普通高等学校招生全国统一考试 物理(广东卷)参考答案 一、 1.D 2.B 3.D 4.BCD 5.AD 6.D 7.AC 8.AB 9.BCD 10.A 二、 11.1.8ⅹ102(5 分) 12. (5 分) 13.4.8(5 分) 三、 14. 15. 16. 四、 17. 18. 19. 20. 21. 22. 2000 年全国普通高等学校招生全国统一考试物理(上海卷) 考生注意: 1.全卷共 8 页,24 题,在 120 分钟内完成。 2.第 21、22、23、24 题要求写出必要的文字说明、方程式和重要的演算步骤,只写出 最后答案,而未写出主要演算过程的,不能得分,有数字计算的问题,答案中必须明确写出 数值和单位。 一、(50 分)选择题,本大题共 10 小题,每小题 5 分,每小题给出的四个答案中,至少有 一个是正确的,把正确的答案全选出来,并将正确答案前面的字母填写在题后的方括号 内,每一小题全选对的得 5 分;选对但不全的,得部分分;有选错或不答的,得 0 分, 填写在方括号外的字母,不作为选出的答案。 1.下列关于光的说法中正确的是 A.在真空中红光波长比紫光波长短。 B.红光光子能量比紫光光子能量小。 C.红光和紫光相遇时能产生干涉现象 D.红光照射某金属时有电子向外发射,紫光照射该金属时一定也有电子向外发射。 2.关于 、  、 三种射线,下列说法中正确的是 A. 射线是原子核自发放射出的氦核,它的穿透能力最强。 B.  射线是原子核外电子电离形成的电子流,它具有中等的穿透能力。 C. 射线一般们随着 或  射线产生,它的穿透能力量强。 D. 射线是电磁波,它的穿透能力最弱。 3.一小球用轻绳悬挂在某固定点,现将轻绳水平拉直,然后由静止开始释放小球,考虑小 球由静止开始运动到最低位置的过程。 A.小球在水平方向的速度逐渐增大。 B.小球在竖直方向的速度逐渐增大。 C.到达最低位置时小球线速度最大。 D.到达最低位置时绳中的位力等于小球重力。 4.如图所示,两根平行放置的长直导线 a 和 b 载有大小相同方向相反的电流, a 受到的磁 场力大小为 1F ,当加入一与导线所在平面垂直的匀强磁场后, a 受到的磁场 力大小变为 2F ,则此时 b 受到的磁场力大小变为 A. 2F , B. 21 FF  , C. 21 FF  D. 212 FF  5.行驶中的汽车制动后滑行一段距离,最后停下;流星在夜空中坠落并发出明亮的光焰; 降落伞在空中匀速下降;条形磁铁在下落过程中穿过闭合线圈,线圈中产生电流,上述 不同现象中所包含的相同的物理过程是 A.物体克服阻力做功。 B.物体的动能转化为其它形式的能量。 C.物体的势能转化为其它形式的能量。 D.物体的机械能转化为其它形式的能量。 6.匀速上升的升降机顶部悬殊有一轻质弹簧,弹簧下端挂有一小球,若升降机突然停止, 在地面上的观察者看来,小武汉在继续上升的过程中 A.速度逐渐减小 B.速度先增大后减小 C.加速度逐渐增大 D.加速度逐渐减小 7.如图,沿波的传播方向上有间距均为 1 米的六个质点 a 、b 、 c 、 d 、 e 、 f ,均静 止在各自的平衡位置,一列横波以 1 米/秒的速度水平向右传播, 0t 时到达质点 a ,a 开 始由平衡位置向上运动, 1t 秒时,质点 a 第一次到达最 高点,则在 4 秒<t <5 秒这段时间内 A.质点 c 的加速度逐渐增大 B.质点 a 的速度逐渐增大 C.质点 d 向下运动 D.质点 f 保持静止 8.如图所示,长度相同的三根轻杆构成一个正三角形支架,在 A 处固定质量为 2 m 的小球, B 处固定质量为 m 的小球,支架悬挂在 O 点,可绕过 O 点并与支架所 在平面相垂直的固定轴转动,开始时 OB 与地面相垂直,放手后开始 运动,在不计任何阻力的情况下,下列说法正确的是 A.A 球到达最低点时速度为零。 B.A 球机械能减少量等于 B 球机械能增加量。 C.B 球向左摆动所能达到的最高位置应高于 A 球开始运动时的高度。 D.当支架从左向右回摆动时,A 球一定能回到起始高度。 9.两木块自左向右运动,现用高速摄影机在同一底片上多次曝光,记录下木块每次曝光时 的位置,如图所示,连续两次曝光的时间间隔是相等的,由图可知 A.在时刻 2t 以及时刻 5t 两木块速度相同。 B.在时刻 1t 两木块速度相同。 C.在时刻 3t 和时刻 4t 之间某瞬间两木块速度相同。 D.在时刻 4t 和时刻 5t 之间某瞬时两木块速度相同。 10.如图( a ),圆形线圈 P 静止在水平桌面上,其正上方悬挂一相同的线圈 Q,P 和 Q 共轴, Q 中通有变化电流,电流随时间变化的规律如图 B.所示,P 所受的重力为 G,桌面对 P 的支持力为 N,则 A. 1t 时刻 N>G。 B. 2t 时刻 N>G。 C. 2t 时刻 N<G。 D. 4t 时刻 N=G。 二、(20 分)填空题,本大题共 5 小题,每小题 4 分,答案写在题中横线上的空白处,不要 求写出演算过程。 11.如图,在场强为 E 的匀强电场中有相距为l 的 A、B 两点,边线 AB 与电场线的夹角为 ,将一电量为 q 的正电荷从 A 点移到 B 点,若沿 直线 AB 移动该电荷,电场力做的功 1W ;若沿路径 ACB 移动该电荷,电场力做的功 2W ;若沿曲线 ADB 移动该电荷,电场力做的功 3W , 由 此 可 知 , 电 荷 在 电 场 中 移 动 时 , 电 场 力 做 功 的 特 点 是: 。 12.一架飞机水平匀速地在某同学头顶飞过,当他听到飞机的发动机声从头顶正上方传来时, 发现飞机在他前上方约与地面与 60°角的方向上,所此可估算出此飞机的速度约为声 速的 倍。 13.右图为人手臂面骨骼与肌 肉的生理结构示意图,手上托 着重量为 G 的物体,(1)在方 框 中 画 出 前 臂 受 力 示 意 图 (手、手腕、尺骨和挠骨看成 一个整体,所受重力不计,图中 O 点看作固定转动轴,O 点受力 可以不画)。(2)根据图中标尺估算出二头肌此时的收缩约为 。 14.某脉冲激光器的耗电功率为 2×103 瓦,每秒钟输出 10 个光脉冲,每个脉冲持续的时间 为 10-8 秒,携带的能量为 0.2 焦耳,则每个脉冲的功率为 瓦,该激光 器将电能转化为激光能量的效率为 。 15.右图为一名宇航员“漂浮”在地球外层空间的照片,根据照 片展现的情景提出两个与物理知识有关的问题(所提的问题可以 涉及力学、电磁学、热学、光学、原子物理学等各个部分,只需 提出问题,不必作出回答和解释): 例:这名“漂浮”在空中的宇航员相对地球是运动还是静止 的? (1) 。 (2) 。 三、(30 分)实验题,本大题共 5 小题,第 16、18(2),19,20 题是填 空题,第 17,18(1)是选择题。 16.(4 分)右图为用频闪摄影方法拍摄的研究物体作平抛运动规律的照 片,图中 A、B、C 为三个同时由同一点出发的小球, AA  为 A 球在光滑水 平面上以速度 v 运动的轨迹; BB  为 B 球以速度 v 被水平抛出后的运动轨 迹; CC  为 C 球自由下落的运动轨迹,通过分析上述三条轨迹可得出结 论: 。 17.(4 分)单色光源发出的光经一狭缝,照射到光屏上,可观察到的图象是 18.(7 分)用右图所示装置做“研究有固定转动轴物体平衡条件”的实 验,力矩盘上各同 心圆的间距相等。 (1)在用细线悬挂钩码前,以下哪些措施是必要的 A.判断力矩盘是否处在竖直平面。 B.判断横杆 MN 是否严格保持水平。 C.判断力矩盘与转轴间的摩擦是否足够小。 D.判断力矩盘的重心是否位于盘中心。 (2)在 A、B、C 三点分别用细线悬挂钩码后,力矩盘平衡,如图所示,已知每个钩码 所受的重力为 1 牛,则此时弹簧秤示数为 牛。 (3)现同时撤去 B、C 两处的钩码,而改用一根细线悬挂 5 牛的钩码,为使力矩盘仍然 保持原平衡状态,且弹簧秤示数不变,试在图中用直线画出该细线在力矩盘上的悬挂位置。 19.(8 分)某同学按如图所示电路进行实验,实验时该同学将变 阻器的触片 P 移到不同位置时测得各电表的示数如下表所示 序号 A1 示数(安) A2 示数(安) V1 示数(伏) V2 示数(伏) 1 0.60 0.30 2.40 1.20 2 0.44 0.32 2.56 0.48 将电压表内阻看作无限大,电流表内阻看作零。 (1)电路中 , r 分别为电源的电动势和内阻, 1R , 2R , 3R 为定值电阻,在这五个 物理量中,可根据上表中的数据求得的物理量是(不要求具体计算) 。 (2)由于电路发生故障,发现两电压表示数相同了(但不为零),若这种情况的发生是 由用电器引起的,则可能的故障原因是 。 20.(7 分)现有一根粗细均匀长约 40 厘米,两端开口的玻璃管,内有一小段水柱,一个弹 簧秤,一把毫米刻度尺,一小块橡皮泥,一个足够高的玻璃容器,内盛有冰和水的混合 物,选用合适的器材,设计一个实验,估测当时的室内温度,要求:(1)在右边方框中 画出实验示意图;(2)写出要测定的物理量 , 写出可直接应用的物理量 ,(3)写出计算室温的表达 式 。 四、(50 分)计算题 21.(12 分)风洞实验室中可以产生水平方向的、大小可调节的风力,现将一套有小球的细 直杆放入风洞实验室,小球孔径略大于细杆直径。 (1)当杆在水平方向上固定时,调节风力的大小,使小球在杆上作匀速运动,这时小 班干部所受的风力为小球所受重力的 0.5 倍,求小球与 杆间的滑动摩擦因数。 (2)保持小球所受风力不变,使杆与水平方向间 夹角为 37°并固定,则小球从静止出发在细杆上滑下 距离 S 所需时间为多少?(sin37°=0.6,cos37°=0.8) 22.(12 分)如图所示,粗细均匀,两端开口的 U 形管竖直放置,管的内径很小,水平部分 BC 长 14 厘米,一空气柱将管内水银分隔成左右两段,大气压强 相当于高为 76 厘米水银柱的压强。 (1)当空气柱温度为 2730 T 开,长为 80 I 厘米时,BC 管内 左边水银柱长 2 厘米,AB 管内水银柱长也是 2 厘米,则右边水银柱总 长是多少? (2)当空气柱温度升高到多少时,左边的水银恰好全部进入竖直管 AB 内? (3)当空气柱温度为 490 开时,两竖直管内水银柱上表面高度各为多少? 23.(13 分)如图所示,固定水平桌面上的金属框架 cdef,处在竖直向下的匀强磁场中,金 属棒 ab 搁在框架上,可无摩擦滑动,此时 adeb 构成一个边长为 I 的正方形,棒的电阻 为 r ,其余部分电阻不计,开始时磁感强度为 0B 。 (1)若从 0t 时刻起,磁感强度均匀增加,每 秒增量为 k ,同时保持棒静止,求棒中的感应电流, 在图上标出感应电流的方向。 (2)在上述(1)情况中,始终保持棒静止,当 1tt  秒末时需加的垂直于棒的水平拉 力为多大? (3)若从 0t 时刻起,磁感强度逐渐减小,当棒以恒定速度 v 向右作匀速运动时,可 使棒中不产生感应电流,则磁感强度应怎样随时间变化(写出 B 与t 的关系式)? 24.(13 分)阅读如下资料并回答问题: 自然界中的物体由于具有一定的温度,会不断向外辐射电磁波,这种辐射因与温度有关, 称为势辐射,势辐射具有如下特点:○1 辐射的能量中包含各种波长的电磁波;○2 物体温度越 高,单位时间从物体表面单位面积上辐射的能量越大;○3 在辐射的总能量中,各种波长所占 的百分比不同。 处于一定温度的物体在向外辐射电磁能量的同时,也要吸收由其他物体辐射的电磁能 量,如果它处在平衡状态,则能量保持不变,若不考虑物体表面性质对辐射与吸收的影响, 我们定义一种理想的物体,它能 100%地吸收入射到其表面的电磁辐射,这样的物体称为黑 体,单位时间内从黑体表面单位央积辐射的电磁波的总能量与黑体绝对温度的四次方成正 比,即 4 0 TP ,其中常量 31067.5  瓦/(米 2·开 4)。 在下面的问题中,把研究对象都简单地看作黑体。 有关数据及数学公式:太阳半径 696000sR 千米,太阳表面温度 5770T 开,火星 半径 3395r 千米,球面积, 24 RS  ,其中 R 为球半径。 (1)太阳热辐射能量的绝大多数集中在波长为 2×10-9 米~1×10-4 米范围内,求相应的 频率范围。 (2)每小量从太阳表面辐射的总能量为多少? (3)火星受到来自太阳的辐射可认为垂直射可认为垂直身到面积为 2r ( r 为火星半 径)的圆盘上,已知太阳到火星的距离约为太阳半径的 400 倍,忽略其它天体及宇宙空间的 辐射,试估算火星的平均温度。 2000 年全国普通高等学校招生统一考试(上海卷) 物理参考答案 说明: (1)定出评分标准是为了尽可能在统一的标准下评定成绩。试题的参考答案是用来说 明评分标准的,考生按其他方法或步骤解答,正确的,同样给分。有错的,根据错误的性质, 参照评分标准中相应的规定评分。 (2)第一、二、三题只要求写出答案,不要求写出演算过程。 (3)第 21、22、23、24 题只有最后答案而无演算过程的,不难分。解答中单纯列出与 解题无关的文字公式,或虽列出公式,但文字符号与题中所给定的不同,不给分。 (4)需作数字计算的问题,对答案的有效数字不作严格要求。一般按试题要求或按试 题情况取二位或三位有效数字即可。 一、选择题 1.BD 2.C 3.AC 4.A 5.AD 6.AC 7.ACD 8.BCD 9.C 10.AD 评分标准:全题 50 分,每小题 5 分,全选对得 5 分,选对但不全得 2 分,有选错或全 部不选得 0 分。 二、填空题 11. cosqEI , cosqEl , cosqEl 电场力做功的大小与路径无关,只与始末位置有关。 12.0.58(或 3/3 ) 13. (1)见右图 (2)8G 14.2×103,0.001 只要属于与照片情景有关的物理问题均可得分,例如:此宇航员是否受地球引力作用? 此宇航员受力是否平衡?宇航员背后的天空为什么是黑暗的?等等,若解答不以问题的形式 出现、不属于物理问题或与照片情景无关,均不能得分。 评分标准:全题 20 分,每小题 4 分,其中第 11 题每格 1 分,第 13 题受力图 2 分,填 空 2 分,第 14 题、15 题每格 2 分。 三、实验题 16.作平抛运动的物体在水平方向作匀速直线运动,在竖直方向作自由落体运动(或平抛运 动是水平方向的匀速直线运动和竖直方向的自由落体运动的合成)。(4 分) 17.A (4 分) 18. (1)A,C,D (4 分) (2)4 (2 分) (3)见右图(悬线在转轴右侧竖直方向与第二圆环相切,钩码 不画不扣分)。(1 分) 19. (1) 2R , 3R , (1 分,1 分,2 分) (2) pR 短路或 2R 断开 (4 分) 20. (1)见右下图, (3 分) (2)玻璃管放在室温中时空气柱的长度 1l ,玻璃管浸在 冰水内时空气柱的长度 2l ,冰水的温度 0T (2 分,每个空格 1 分) (3) 210 /llTT  (2 分) 评分标准:全题 30 分,具体见于各题答案后,其中第 18(1)有多个正确选项,全选 对得 4 分,选对但不全得 2 分,有选错或全部不选得 0 分。19(1)若填 r , 1R 的,在本小 题的得分中,多填一个扣 1 分,扣完为止,如填 pR ,不扣分。20(1)中,若画入弹簧秤, 其余正确,扣 1 分,玻璃管不密封的不能得分。20(2)中 0T 用具体数值表示的,同样给分。 四、计算题 21. (1)设小球所受的风力为 F,小球质量为 m mgF  ○1 5.0/5.0/  mgmgmgF ○2 (2)设杆对小球的支持力为 N,摩擦力为 f 沿杆方向 mafmgninF  cos ○3 垂直于杆方向 0cossin   ngFN ○4 Nf  ○5 可解得 g gm Fgm fngFa 4 3sin)(sincos 2 2   ○6 2 2 1 atS  ○7 g S g St 3 8 4/3 2  ○8 评分标准:全题 12 分。 (1)3 分。正确得出○2 式,得 3 分。仅写出○1 式,得 1 分。 (2)9 分,正确得出○6 式,得 6 分,仅写出○3 、○4 式,各得 2 分,仅写出○5 式,得 1 分,正确得出○8 式,得 3 分,仅写出○7 式,得 2 分,g 用数值代入的不扣分。 22. (1)U 形管两端均开口,所以两竖直管内水银面高度应相同,即右边竖直管内水 银柱高度为 20 h (厘米) ○1 右边水平管内水银柱长度为 14- 0t -2=4(厘米) 右边水银柱总长是 4+2=6(厘米) ○2 (2)左边的水银全部进入竖直管内时,两竖直管内水银面高度均为 41 h (厘米) ○3 此时,右边水平管内水银柱长度为 2 厘米,所以空气柱长为 1l =14-2=12(厘米) ○4 1 110 0 000 )()( T lhP T lhp  ○5 420878 1280273)( )( 000 110 01    lhP lhPTT (开) ○6 (3)设温度为 4902 T 开时,空气柱长为 2l 等压过程 2 2 1 1 T I T I  ○7 14420 12490 1 1 22  T lTl (厘米) ○8 其中有 2 厘米进入左边竖直管内 ∴右管内水银面高度为 41 h (厘米) ○9 左管内水银上表面高度为 6242 h (厘米) ○10 评分标准:全题 12 分。 (1)3 分,正确得出结果○2 ,得 3 分,仅得出○1 式,得 1 分。 (2)5 分,正确得出○4 式,得 2 分,仅得出○3 式,得 1 分,正确得出○6 式,得 3 分, 仅得出○5 式,得 2 分。 (3)4 分,正确得出○8 式,得 2 分,仅写出○7 式,得 1 分,得出○9 式、○10式,各 1 分。 23. (1)感应电动势 2kit  ○1 感应电流 r ki rI 2   ○2 方向:逆时针(见右图) (2) 1tt  秒时, 10 ktBB  ○3 BIIF  ○4 r kIktBF 3 10 )(  ○5 (3)总磁通量不变 2 0)( lBvtlBl  ○6 vll lBB  0 ○7 评分标准:全题 13 分。 (1)5 分,正确得出○2 式,得 4 分,仅得出○1 式得 2 分,方向正确,得 1 分。 (2)4 分,正确得出○5 式,得 4 分,仅得出○3 式,得 1 分,仅写出○4 式,得 2 分。 (3)4 分,正确得出○7 式,得 4 分,仅得出○6 式,得 2 分。 24. (1) /cV  ○1 1332 1 105.1102/1000.3  V (赫) ○2 1352 1 103101/1000.3  V (赫) ○3 辐射的频率范围为 3×1013 赫-1.5×1013 赫 (2)每小量从太阳表面辐射的总能量为 tTRW 42 s4 ○4 代入数所得 W=1.38×1010 焦 ○5 (3)设火星表面温度为 T,太阳到火星距离为 d ,火星单位时间内吸收来自太阳的辐 射能量为 2 2 42 4 4 d rTRP s    ○6 sRd 400 224 )400/(rTP   ○7 火星单位时间内向外辐射电磁波能量为 424 TrP t   ○8 火星处在平衡状态 tt PP   ○9 即 42224 4)400/( TrrT   ○10 由○10式解得火星平均温度 204800/  TT (开) ○11 评分标准:全题 13 分 (1)3 分,正确得了○1 ,○2 ,○3 式,各得 1 分。 (2)5 分,正确得出○5 式,得 5 分,仅写出○4 式,得 3 分。 (3)5 分,正确得出○10式,得 4 分,仅写出○6 式或○7 式,得 1 分;仅写出○8 式,得 1 分,正确得出○11式,得 1 分。 2000 年全国普通高等学校招生全国统一考试 物理(天津、江西卷) 本试卷分第Ⅰ卷(选择题)和第Ⅱ卷(非选择题)两部分,第Ⅰ卷 1 至 3 页,第Ⅱ卷 4 至 11 页共 150 分,考试时间 120 分钟。 第Ⅰ卷(选择题 共 40 分) 注意事项: 1.答第Ⅰ卷前,考生务必将自己的姓名、准考证号、考试科目、试卷类型(A 或 B)用 铅笔涂写在答题卡上。 2.每小题选出答案后,用铅笔把答题卡上对应题目的答案标号涂黑,如需改动,用橡 皮擦干净后,再选涂其它答案。不能答在试题卷上。 3.考试结束,将本试卷和答题卡一并交回。 4.必要时可以使用下列物理量。 真空中光速 m/s100.3 3c 万有引力常量 2211 /kgmN107.6  G 普朗克常量 sJ106.6 34  h 电子的电量 C106.1 19c 地球半径 m104.6 6R 电子的质量 kg101.9 31cm 一、本题共 10 小题,每小题 4 分,共 40 分,在每小题给出的四个选项中,有的小题只有一 个选项正确,有的小题有多个选项正确,全部选对的得 4 分,选不全的得 2 分,有选错 或不答的得 0 分。 1.最近几年,原子核科学在超重元素岛的探测方面取得重大进展,1996 年科学家们在研究 某两个重离子结合成超重元素的反应时,发现生成的超重元素的核 XA Z 经过 6 次 衰变 后的产物是 Fm253 100 ,由此,可以判定生成的超重元素的原子序数和质量数分别是 A.124、259 B.124、265 C.112、265 D.112、277 2.对于一定量的理想气体,下列四个论述中正确的是 A.当分子热运动变剧烈时,压强必变大。 B.当分子热运动变剧烈时,压强可以不变。 C.当分子间的平均距离变大时,压强必变小。 D.当分子间的平均距离变大时,压强必变大。 3.在高速公路的拐弯处,路面造得外高内低,即当车向右拐弯时,司机左侧的路面比右侧 的要高一些,路面与水平面间的夹角为 。设拐弯路段是半径为 R 的圆弧,要使车速 为 v 时车轮与路面之间的横向(即垂直于前进方向)摩擦力等于零, 应等于 A. Rg v 2 sin arc B. Rg v 2 tgarc C. Rg v 22sin arc2 1 D. Rg v 2 ctg arc 4.对于水平放置的平行板电容器,下列说法正确的是 A.将两极板的间距加大,电容将增大 B.将两极板平行错开,使正对面积减小,电容将减小 C.在下板的内表面上放置一面积和极板相等、厚度小于极板间的陶瓷板,电容将增大 D.在下板的内表面上放置一面积和极板相等、厚度小于极板间距的铝板,电容将增大 5.图中活塞将气缸分成甲、乙两气室,气缸、活塞(连同拉杆)是绝热的,且不漏气,以 甲E 、 乙E 分别表示甲、乙两气室中气体的内能,则在将拉杆缓慢向外拉的过程中 A. 甲E 不变, 乙E 减小 B. 甲E 增大, 乙E 不变 C. 甲E 增大, 乙E 减小 D. 甲E 不变, 乙E 不变。 6.图为 X 射线管的结构示意图, E 为灯丝是源,要使射线管发 出 X 射线,须在 K 、 A 两电极间加上几万伏的直流高压。 A.高压电源正极应接在 P 点, X 射线从 K 极发出 B.高压电源正极应接在 P 点, X 射线从 A 极发出 C.高压电源正极应接在 Q 点, X 射线从 K 极发出 D.高压电源正极应接在 Q 点, X 射线从 A 极发出 7.一列横波在 0t 时刻的波形如图中实线所示,在 st 1 时 刻的波形如图中虚线所示,由此可以判定此波的 A.波长一定量 4 ㎝ B.周期一定量 4s C.振幅一定是 2 ㎝ D.传播速度一定是 1 ㎝/s 8.A 与 B 是两束平行的单色光,它们从空气射入水中的折射角分别为 Ar 、 Br ,若 Ar > Br ; 则 A.在空气中 A 的波长大于 B 的波长 B.在水中 A 的传播速度大于 B 的传播速度 C.A 的频率大于 B 的频率 D.在水中 A 的波长小于 B 的波长 9.图中 A、B、C 是本相交流电源的三根相线,O 是中线,电源的相电压为 220V, 1L 、 2L 、 3L 是三个“220V 60W”的灯泡,开关 1K 断开, 2K 、 3K 闭 合,由于某种原因,电源中线在图中O处断了,那么 2L 和 3L 两灯泡将 A.立刻熄灭 B.变得比原来亮一些 C.变得比原来暗一些 D.保持亮度不变 10.图为一空间探测器的示意图, 1P 、 2P 、 3P 、 4P 是四个喷气发动机, 1P 、 3P 是连线与空间一固定坐标 系的 x 轴平行, 2P 、 4P 的连线与 轴平行,每台发动 机开动时,都能向探测器提供推力,但不会使探测器 转动,开始时,探测器以恒定的速率 0v 向正 x 方向平行,要使探测器改为向正 x 偏负 60° 的方向以原来的速率 0v 平动,则可 A.先开动 1P 适当时间,再开动 4P 适当时间 B.先开动 3P 适当时间,再开动 2P 适当时间 C.开动 4P 适当时间 D.先开动 3P 适当时间,再开动 4P 适当时间 第Ⅱ卷(非选择题 共 110 分) 注意事项: 1.第Ⅱ卷共 8 页,用钢笔或圆珠笔直接答在试题卷中(除题目有特殊规定外)。 2.答卷前将密封线内的项目填写清楚。 二、本题共 3 小题;每小题 5 分,共 15 分,把答案填在题中的横线上。 11.已知金属铯的逸出功为 1.9eV,在光电效应实验中,要使铯表面发出的光电子的最大动 能为 1.0eV,入射光的波长应为 m。 12.空间存在以 ab 、 cd 为边界的匀强磁场区域,磁感强度 大小为 B, 方向垂直纸面向外,区域宽为 1l ,瑞有一矩形线 框处在图中纸面内,它的短边与 ab 重合,长度为 2l ,长边的 长度为 2 1l ,如图所示,某时刻线框以初速 v 沿与 ab 垂直的方向进入磁场区域,同时某人 对线框施以作用力,使它的速度大小和方向保持不变,设该线框的电阻为 R ,从线框开始 进 入 磁 场 到 完 全 离 开 磁 场 的 过 程 中 , 人 对 线 框 作 用 力 所 做 的 功 等 于 。 13.假设在 NaCI 蒸气中存在由钠离子 Na+和氯离子 CI-靠静电相互作用构成的单个氯化钠 NaCI 分子,若取 Na+与 CI-相距无限远时其电势能为零,一个 NaCI 分子的电势能为- 6.1eV,已知使一个中性钠原子 Na 最外层的电子脱离钠原子面形成钠离子 Na+所需的能 量(电离能)为 5.1eV,使一个中性氯原子 CI 结合一个电子形成氯离子 CI-所放出的能 量(新和能)为 3.8eV。由此可算出,在将一个 NaCI 分子分解成彼此远离的中性钠原 子 Na 和中性氯原子 CI 的过程中,外界供给的总能量等于 eV/ 三、三题共 3 小题,共 20 分,把答案填在题中的横线上。 14.(6 分)某同学用图 1 所示装置通过半径相同的 A、B 两球的碰撞来验证动量守恒定律, 图 1 中 PQ 是斜槽,QR 为水平槽,实验时先使 A 球从斜槽上某一固定位置 G 由静止开始 滚下,落到位于水平地面的记录纸上,留下痕迹,重复上述操作 10 次,得到 10 个落点 痕迹,再把 B 球放在水平槽上靠近槽末端的地方,让 A 球仍从位置 G 由静止开始滚下, 和 B 球碰撞后,A、B 球分别在记录纸上留下各自的落点痕迹,重复这种操作 10 次,图 1 中 O 点是水平槽末端 R 在记录纸上的垂直投影点,B 球落点痕迹如图 2 所示,其中米 尺水平放置,且平行于 G、R、O 所在的平面,米尺的零点与 O 点对齐。 (1)碰撞后 B 球的水平射程应为 ㎝。 (2)在以下选项中,哪些是本次实验必须进行的测量?答: (填选项号) A.水平槽上放 B 球时,测量 A 球落点位置到 O 点的距离 B.A 球与 B 球碰撞后,测量 A 球落点位置到 O 点的距离 C.测量 A 球或 B 球的直径。 D.测量 A 球和 B 球的质量(或两球质量之比) E.测量 C 点相对于水平槽面的高度。 15.(6 分)如图,一光源位于金属圆筒内部轴线上 A 点,与筒 B 端的距离为 d , d 无法直 接测量。另有凸透镜、光屏、米尺及带支架的光具座,现用这些器材测量 d ,为此,先 将圆筒、凸透镜、光屏依次放在光具座支架上,令圆筒轴线与透镜主光轴重合,屏与光 源的距离足够远。使得移动透镜时,可在屏上两次出现光源的象,将圆筒及光屏位置固 定,由光路的可逆性可知,第一次成象的距等于第二次成象的物距,然后进行以下的测 量: 用测得的物理量可得 d 。 (应说明各符号所代表的物理量) 16.(8 分)图 1 为示波器面板,图 2 为一信号源。 (1)若要观测此信号源发出的正弦交流信号的波形,应将信号源的 a 端与示波器面板 上的 接线柱相连,b 端与 接线柱相连。 (2)若示波器所显示的输入波形如图 3 所示,要将波形上移,应调节面板上的 旋钮;要使此波形横向展宽,应调节 旋钮;要使屏上能够显示 3 个完整 的波形,应调节 旋钮。 四、本题共 6 小题,75 分,解答应写出必要的文字说明、方程式和重要演算步骤,只写出 最后答案的不能得分。有数值计算的题,答案中必须明确写出数值和单位。 17.(11 分)有一实用氧气钢瓶,瓶内氧气的压强 Pa100.5 3 ,温度 27t ℃,求氧 气的密度,氧的摩尔质量 kg/mol102.3 2 ,结果取两位数字。 18.(12 分)一小型发电机内的矩形线圈在匀强磁场中以恒定的角速度 绕垂直于磁场方向 的固定轴转动,线圈匝数 100n ,穿过每匝线圈的磁通量  随时间按正弦规律变化, 如图所示,发电机内阻  0.5r ,外电路电阻  95R ,已知感应电动势的最大值  nEm ,其中 m 为穿过每匝线圈磁通量的最大值,求串联在外电路中的交流电流 表(内阻不计)的读数。 19.(13 分)图中是用电动砂轮打磨工件的装置,砂轮的转轴过图中 O 点垂直于纸面,AB 是一长度 ml 60.0 ,质量 kgm 50.01  的均匀刚性细杆,可绕过 A 端的固定轴在竖直 面(图中纸面)内无摩擦地转动,工件 C 固定在 AB 杆上,其质量 kgm 5.12  ,工件的 重心、工件与砂轮的接触点 P 以及 O 点都在过 AB 中点的竖直线上,P 到 AB 杆的垂直距 离 md 1.0 ,AB 杆始终处于水平位置,砂轮与工件之间的动摩擦因数 06.0 (1)当砂轮静止时,要使工件对砂轮的压力 1000 F N,则施于 B 端竖直向下的力 BF 应是多大? (2)当砂轮逆时针转动时,要使工件对砂轮的压力仍为 1000 F N,则施于 B 端竖直向下的力  BF 应是多大? 20.(12 分)2000 年 1 月 26 日我国发射了一颗同步卫星,其定点位置与东经 98°的经线在 同一平面内,若把甘肃省嘉峪关处的经度和纬度近似取为东经 98°和北纬 40 , 已知地球半径 R、地球自转周期 T、地球表面重力加速度 g(视为常量)和光速 c。试求 该同步卫星发出的微波信号传到嘉峪关处的接收站所需的时间(要求用题给的已知量的 符号表示)。 21.(13 分)如图,两个共轴的圆筒形金属电极,外电极接地,其上均匀分布着平行于轴线 的四条狭缝 a 、b 、c 和 d ,外筒的外半径为 0r ,在圆筒之外的足够大区域中有平行于 轴线方向的均匀磁场,磁感强度的大小为 B,在两极间加上电压,使两圆筒之间的区域 内有沿半径向外的电场,一质量为 m 、带电量为 q 的粒子,从紧靠内筒且正对狭缝 a 的 S 点出发,初速为零。如果该粒子经过一段时间的运动之后恰好又回到出发点 S,则 两电极之间的电压 U 应是多少?(不计重力,整个装置在直空中) 22.(14 分)在原子核物理中,研究核子与核子关联的最有效途径是“双电荷交换反应”。 这类反应的前半部分过程和下述力学模型类似,两个小球 A 和 B 用轻质弹簧相连,在光 滑的水平直轨道上处于静止状态,在它们左边有一垂直于轨道的固定挡板 P,右边有一 小球 C 沿轨道以速度 0 射向 B 球,如图所示,C 与 B 发生碰撞并立即结成一个整体 D, 在它们继续向左运动的过程中,当弹簧长度变到最短时,长度突然被锁定,不再改变, 然后,A 球与挡板 P 发生碰撞,碰后 A、D 都静止不动,A 与 P 接触而不粘连,过一段时 间,突然解除锁定(锁定及解除锁定均无机械能损失),已知 A、B、C 三球的质量均为 m , (1)球弹簧长度刚被锁定后 A 球的速度。 (2)球在 A 球离开挡板 P 之后的运动过程中,弹簧的最大弹性势能。 2000 年全国普通高等学校招生统一考试(天津、江西卷) 物理参考答案 说明: (1)定出评分标准是为了使全国各地尽可能在统一标准下评定成绩,试题的参考解答 是用说明评分标准的,考生如按其它方法或步骤解答,正确的,同样给分;有错的,根据错 误的性质,参照评分标准中相应的规定评分。 (2)第一、二、三题只要求写出答案,不要求说明理由或列出算式,只根据答案评分。 (3)第四大题,只有最后答案而无演算过程的,不给分;只写出一般公式但未能与试 题所给的具体条件联系的,不给分。 一、答案及评分标准:全题 40 分,每小题 4 分,每小题全选对的给 4 分,选不全的给 2 分, 有选错的给 0 分,不答的给 0 分。 1.D 2.B 3.B 4.BCD 5.C 6.D 7.AC 8.AB 9.C 10.A 二、答案及评分标准:全题 15 分,每小题 5 分,答案正确的,按下列答案后面括号内的分 数给分;答错的,不答的,都给 0 分。 11.4.3×10-9(5 分) 12. 1 2 2 )(2 lR Bl (5 分) 13.4.8(5 分) 三、答案及评分标准:全题 20 分,其中 14 题 6 分,15 题 6 分,16 题 8 分,答案正确的, 按下列答案后面括号内的分数给分;答错的,不答的,都给 0 分。 14.(1)64.7(3 分,答数在 64.2~65.2 范围内的都给分。) (2)A、B、D(3 分,不是 A、B、D 的均给零分。) 15.移动透镜,当习上得到光源清晰的象时,测量象距 ;继续移动透镜,当屏上得到光源 的另一个清晰的象时,测量端面 B 与透镜的距离 l 。 ld   (6 分,完全答对给 6 分,否则不给分,如果测量方法与参考答案不同,但结 果正确,同样给分。) 16.(1) 输入(1 分),地(1 分) (2)6(2 分,填写↓↑的也给 2 分),X 增益(2 分),扫描范围和扫描微调(2 分,答 对其中 1 个即可给这 2 分) 四、参考解答及评分标准: 17.参考解答: 设钢瓶内氧气的摩尔数为 n ,体积为 V,则有 RT pVn  ○1 氧气密度 V n  ○2 由○1 、○2 式联立得 RT n  ○3 以题给数据据代入得 2kg/m64 ○4 评分标准:本题 11 分,○1 、○2 两式各 1 分。○3 式 6 分(没有○1 、○2 两式而直接写出○3 式 的给 8 分),结果○4 式 3 分。 18.参考解答: 已知感应电动势的最大值 mm naE  ○1 设线圈在磁场中转动的周其为 T,则有 T  2 ○2 根据欧姆定律,电路中电流的最大值为 rR EI m m  ○4 设交流电流表的读数 I,它是电流的有效值,根据有效值与最大值的关系,有 mII 2 1 ○4 由题给的 1 图线可读得 Wbm -2100.1  ○5 BT 21014.3  ○6 解以上各式,并代入数据,得 AI 4.1 ○7 评分标准:本题 12 分。 ○2 式 1 分,○3 式 2 分,○4 式 3 分,○5 式 2 分,○6 式 2 分,○7 式 2 分。 19.参考解答: 解:(1)当砂轮静止时,把 AB 杆和工件看成一个物体,它受到的外力对 A 轴的力矩有: 重力的力矩( 2)( 21 lgmm  ) 砂轮对工件的支持力的力矩 20 lF BF 的力矩 lFB 由力矩的平衡,得 lFlgmmlF B 2)(2 210 ○1 解得 ])([2 1 210 gmmFFB  ○2 代入数据得 NFB 40 ○3 (2)当砂轮转动时,除重力、支持力和  BF 的力矩外,还有砂轮作用于工件的摩擦力 的力矩 dF0 。 由力矩的平平衡;得 lFFlgmmdFF BB  2)(2 1 2100  ○4 解得 I dFgmmFFB 0210 ])([2 1  ○5 代入数据得 NFB 30 ○6 评分标准:本题 13 分 第(1)问 5 分,其中○1 式 3 分,○2 式 1 分,○3 式 1 分。 第(2)问 8 分,其中○4 式 6 分,○5 式 1 分,○6 式 1 分。 20.参考解答: 设 m 为卫星质量,M 为地球质量,r 为卫星到地球中心的距离, 为卫星绕地心转动 的角速度,由万有引力定律和牛顿定律有, 2 2 mr r mMG  ○1 式中 G 为万有引力恒量,因同步卫星绕地心转动的角速度 与地球自转的角速度相等有 T  2 ○2 因 mg R MmG 2 得 2gRGM  ○3 设嘉峪关到同步卫星的距离为 L ,如图所示,由余弦定理 cos222 rRRrL  ○4 所求时间为 c Lt  ○5 由以上各式得 c agTRRRgTR t cos 4 2 4 3 1 2 22 2 3 2 2 22              ○6 评分标准:本题 12 分。 ○1 式 1 分,○2 式 2 分,○3 式 1 分,○4 式 5 分,○5 式 1 分,○6 式 2 分。 21.参考解答: 带电粒子从 S 出发,在两筒之间的电场力作用下加速,沿径向 穿出 a 而进入磁场区,在洛仑兹力作用下做匀速圆周运动,粒子再 回到 S 点的条件是能沿径向穿过狭缝b ,只要穿过了b ,粒子就会 在电场力作用下选减速,再反向回速,经b 重新进入磁场区,然后, 粒子将以同样方式经过 c 、 d ,再经过 a 回到 S 点。 设粒子射入磁场区的速度为 ,根据能量守恒,有 qUm 2 2 1  ○1 设粒子在洛仑兹力作用下做匀速圆周运动的半径为 R,由洛仑兹力公式和牛顿定律得   qBRm  2 ○2 由并面分析可知,要回到 S 点,粒子从 a 到b 必经过 4 3 圆周,所以半径 R 必定等于筒的 外半径 0r ,即 0rR  ○3 由以上各式解得 m BqrU 2 22 0 ○4 评分标准:本题 13 分。 ○1 式 2 分,○2 式 2 分,经分析得出○3 式 6 分,解得○4 式 3 分。 22.参考解答: (1)设 C 球与 B 球粘结成 D 时,D 的速度为 1 ,由动量守恒,有 10 )(  mm  ○1 当弹簧压至最短时,D 与 A 的速度相等,设此速度为 2 ,由动量守恒,有 21 32  mm  ○2 由○1 、○2 两式得 A 的速度 02 3 1  ○3 (2)设弹簧长度被锁定后,贮存在弹簧中的势能为 pE ,由能量守恒,有 pEmm  2 2 2 1 32 122 1  ○4 撞击 P 后,A 与 D 的动能都为零,解除锁定后,当弹簧刚恢复到自然长度时,势能全部 转变成 D 的动能,设 D 的速度为 3 ,则有 2 3)2(2 1  mE p ○5 以后弹簧伸长,A 球离开挡板 P,并获得速度,当 A、D 的速度相等时,弹簧伸至最长, 设此时的速度为 4 ,由动最守恒,有 43 32  mm  ○6 当弹簧伸到最长时,其势能量大,设此势能为  E ,由能量守恒,有  pEm 2 4 2 3 32 12m2 1  ○7 解以上各式得 2 036 1 mE p  ○8 评分标准:本题 14 分。 第(1)问 5 分,其中○1 式 2 分,○2 式 2 分,○3 式 1 分。 第(2)问 9 分,其中○4 式 2 分,○5 式 3 分,○6 式 1 分,○7 式 2 分,○8 式 1 分。 2001 年普通高等学校春季招生考试 物 理 (北京、内蒙古、安徽卷) 本试卷分第Ⅰ卷(选择题)和第Ⅱ卷(非选择题)两部分,第Ⅰ卷 1 至 3 页,第Ⅱ卷 4 至 11 页,共 150 分.考试时间 120 分钟. 第Ⅰ卷(选择题 共 40 分) 注意事项: 1.答第Ⅰ卷前,考生务必将自己的姓名、准考证号、考试科目用铅笔涂写在答题卡上. 2.每小题选出答案后,用铅笔把答题卡上对应题目的答案标号涂黑.如需改动,用橡皮擦干 净后,再选涂其它答案.不能答在试题卷上. 3.考试结束,将本试卷和答题卡一并交回. 一、本题共 10 小题;每小题 4 分,共 40 分.在每小题给出的四个选项中,有的小题只有一 个选项正确,有的小题有多个选项正确.全部选对的得 4 分,选不全的得 2 分,有选错 或不答的得 0 分. 1.如图所示,两根相同的轻弹簧 1S 、 2S ,劲度系数皆为 mNk /104 2 .悬挂的重物的质量分别为 kgmkgm 42 21  和 .若不计弹簧质量, 取 2/10 smg  ,则平衡时弹簧 1S 、 2S 的伸长量分别为 A. cm5 、10 cm B.10cm 、 cm5 C.15cm 、10cm D.10 cm 、15 cm 2.下列现象中,与原子核内部变化有关的是 A. 粒子散射 B.光电效应 C.天然放射现象 D.原子发光现象 3.下列说法中正确的是 A.物体的分子热运动动能的总和就是物体的内能 B.对于同一种气体,温度越高,分子平均动能越大 C.要使气体的分子平均动能增大,外界必须向气体传热 D.一定质量的气体,温度升高时,分子间的平均距离一定增大 4.初速为 0v 的电子,沿平行于通电长直导线的方向射出, 直导线中电流方向与电子的初始运动方向如图所示,则 A.电子将向右偏转,速率不变 B.电子将向左偏转,速率改变 C.电子将向左偏转,速率不变 D.电子将向右偏转,速率改变 5.一定质量的理想气体经过一系列过程,如图所示. 下列说法中正确的是 A. ba  过程中,气体体积增大,压强减小 B. cb  过程中,气体压强不变,体积增大 C. ac  过程中,气体压强增大,体积变小 D. ac  过程中,气体内能增大,体积不变 6.将物体以一定的初速度竖直上抛.若不计空气阻力,从抛出到落回原地的整个过程中, 下列四个图线中正确的是 7.一平行板电容器,两板之间的距离 d 和两板面积 S 都可以调节,电容器两板与电池相连 接.以Q 表示电容器的电量, E 表示两极板间的电场强度,则 A.当 d 增大、 S 不变时, Q 减小、 E 减小 B.当 S 增大、 d 不变时, Q 增大、 E 增大 C.当 d 减小、 S 增大时, Q 增大、 E 增大 D.当 S 减小、 d 减小时, Q 不变、 E 不变 8.在如图所示的电路中,电容器 C 的上极板带正电. 为了使该极板仍带正电且电量增大,下列办法中可 采用的是 A.增大 1R ,其他电阻不变 B.增大 2R ,其他电阻不变 C.增大 3R ,其他电阻不变 D.增大 4R ,其他电阻不变 9.有一列沿水平绳传播的简谐横波,频率为 10 Hz ,振动方向沿竖直方向.当绳上的质点 P 到达其平衡位置且向下运动时,在其右方相距 m6.0 处的质点Q 刚好到达最高点.由 此可知波速和传播方向可能是 A. sm /8 ,向右传播 B. sm /8 ,向左传播 C. sm /24 ,向右传播 D. sm /24 ,向左传播 10.一物体放置在倾角为 的斜面上,斜面固定于加速上升的电梯中, 加速度为 a ,如图所示.在物体始终相对于斜面静止的条件下, 下列说法中正确的是 A.当 一定时, a 越大,斜面对物体的正压力越小 B.当 一定时, a 越大,斜面对物体的摩擦力越大 C.当 a 一定时, 越大,斜面对物体的正压力越小 D.当 a 一定时, 越大,斜面对物体的摩擦力越小 第Ⅱ卷(非选择题 共 110 分) 注意事项: 1.第Ⅱ卷共 8 页,用钢笔或圆珠笔直接答在试题卷中(除题目有特殊规定外). 2.答卷前将密封线内的项目填写清楚. 二、本题共 3 小题;每小题 5 分,共 15 分。把答案填在题中的横线上. 11.平衡下列核反应方程式: (1)  xnNBy x ,13 7 ______________, y _________________. (2)  pLi7 3 ______________ 12.一质量为 kg15100.4  、电量为 9100.2  C 的带正电质点,以 sm /100.4 4 的速度 垂直于电场方向从 a 点进入匀强电场区域,并从b 点离开电场区域.离开电场时的速度 为 sm /100.5 4 .由此可知,电场中 a 、b 两点间的电势差  ba UU ____________ V; 带电质点离开电场时,速度在电场方向的分量为______________ sm / .不考虑重力作 用. 13.质量为 kgm 10.0 的小钢球以 smv /100  的水平速度抛出, 下落 mh 0.5 时撞击一钢板,撞后速度恰好反向, 则钢板与水平面的夹角  _____________. 刚要撞击钢板时小球动量的大小为_________________. (取 2/10 smg  ) 三、本题共 3 小题;其中第 14、15 两题每题 6 分,第 16 题 8 分,共 20 分.把答案填在题 中的横线上或按题目要求作图. 14.已知打点计时器接的交流电源频率是 f ,用它记录一个匀变速运动小车的位移,打出 的一条纸带和已选好的计数点 0,1、2、3、4 如图所示.某同学测量出 1 与 2 两点间的 距 离 为 12S , 3 与 4 两 点 间 的 距 离 为 34S , 由 此 可 算 出 小 车 运 动 的 加 速 度 a _______________. 15.做测定凸透镜焦距的实验时,把蜡烛和光屏放在透镜的主光轴上,与主光轴垂直.若这 时在它们之间无论怎样移动透镜,光屏上都得不到清晰的蜡烛像,则应采取的措施是 _________________________________.为了求得凸透镜的焦距,测出蜡烛到光屏的距 离 L 和蜡烛在光屏上两次成像时透镜的两个位置之间的距离 d ,则该透镜的焦距 f _______________________. 16.在测定金属电阻率的实验中,用伏安法测量电阻.将实验电路图画在方框中.为了完成 整个实验,除你在电路中已画出的器材外,还需要用哪些仪器?用这些仪器测量哪些 量? 答: __________ ___________ _____________________________________________. 计算电阻率  的公式是  ________________________. 四、本题共 6 小题,75 分.解答应写出必要的文字说明、方程式和重要演算步骤.只写出 最后答案的不能得分.有数值计算的题,答案中必须明确写出数值和单位. 17.(11 分)简述光的全反射现象及临界角的定义,并导出折射率为 n 的玻璃对真空的临界 角公式。 18.(12 分)两个星球组成双星,它们在相互之间的万有引力作用下,绕连线上某点做周期 相同的匀速圆周运动。现测得两星中心距离为 R,其运动周期为 T,求两星的总质量。 19.(12 分)如图所示,AB、CD 为两根平行的相同的均匀电阻丝,EF 为另一根电阻丝, 其电阻为 R,它可以在 AB、CD 上滑动并保持与 AB 垂直,EF 与 AB、CD 接触良好.图 中电压表为理想电压表.电池的电动势和内阻都不变.B、D 与电池两极连接的导线的 电阻可忽略.当 EF 处于图中位置时,电压表的读数为 VU 0.41  .已知将 EF 由图中 位置向左移动一段距离 L 后,电压表的读数变为 VU 0.32  .若将 EF 由图中位置向 右移动一段距离 L ,电压表的读数 3U 是多少? 20.(12 分)两根足够长的固定的平行金属导轨位于同一水平面内,两导轨间的距离为l 。 导轨上面横放着两根导体棒 cdab和 ,构成矩形回路,如图所示.两根导体棒的质量皆 为 m ,电阻皆为 R,回路中其余部分的电阻可不计.在整个导轨平面内都有竖直向上 的匀强磁场,磁感应强度为 B .设两导体棒均可沿导轨无摩擦地滑行.开始时,棒 cd 静止,棒 ab 有指向棒 cd 的初速度 0v (见图).若两导体棒在运动中始终不接触,求: (1)在运动中产生的焦耳热最多是多少. (2)当 ab 棒的速度变为初速度的 4 3 时, cd 棒的加速度是多少? 21.(14 分)如图所示,一水平放置的气缸,由截面积不同的两圆筒联接而成.活塞 A、B 用一长为 l3 的刚性细杆连接,它们可以在筒内无摩擦地沿地沿水平方向左右滑动.A、 B 的截面积分别为 230cmS A  、 215cmSB  .A、B 之间封闭着一定质量的理想气体.两 活塞外侧(A 的左方和 B 的右方)都是大气,大气压强始终保持为 PaP 5 0 100.1  .活 塞 B 的中心连一不能伸长的细线,细线的另一端固定在墙上.当气缸内气体温度为 KT 5401  ,活塞 A、B 的平衡位置如图所示,此时细线中的张力为 NF 301  . (1)现使气缸内气体温度由初始的 540 K 缓慢下降,温度降为多少时活塞开始向右移 动? (2)继续使气缸内气体温度下降,温度降为多少时活塞 A 刚刚右移到两圆筒联接处? (3)活塞 A 移到两圆筒联接处之后,维持气体温度不变,另外对 B 施加一个水平向左 的推力,将两活塞慢慢推向左方,直到细线拉力重新变为 30N.求此时的外加推力 2F 是多 大. 22.(14 分)如图所示,A、B 是静止在水平地面上完全相同的两块长木板。A 的左端和 B 的右端相接触。两板的质量皆为 kgM 0.2 ,长度皆为 ml 0.1 .C 是一质量为 kgm 0.1 的小物块.现给它一初速度 smv /0.20  ,使它从 B 板的左端开始向右滑 动.已知地面是光滑的,而 C 与 A、B 之间的动摩擦因数皆为 10.0 .求最后 A、B、 C 各以多大的速度做匀速运动.取重力加速度 2/10 smg  2001 年普通高等学校春季招生考试 (北京、内蒙古、安徽卷) 物理试题答案及评分标准 说明: (1)定出评分标准是为了使全国各地尽可能在统一标准下评定成绩.试题的参考解答 是用来说明评分标准的.考生如按其它方法或步骤解答,正确的,同样给分;有错的,根据 错误的性质,参照评分标准中相应的规定评分. (2)第一、二、三题只要求写出答案,不要求说明理由或列出算式,只根据答案评分. (3)第四大题,只有最后答案而无演算过程的,不给分;只写出一般公式但未能与试 题所给的具体条件联系的,不给分. 一、答案及评分标准:全题 40 分,每小题 4 分.每小题全选对的给 4 分,选不全的给 2 分, 有选错的给 0 分,不答的给 0 分. 1.C 2.C 3.B 4.A 5.AD 6.BC 7.AC 8.AD 9.BC 10.BC 二、答案及评分标准:全题 15 分,每小题 5 分.答案正确的,按下列答案后面括号内的分 数给分;答错的,不答的,都给 0 分. 11.(1)5(1 分),10(2 分) (2) 或He4 2 (2 分) 12. 2100.9  (2 分), 4100.3  (3 分) 13. 45 (2 分) sNsmkg  2/2 或 (3 分) 三、答案及评分标准:全题 20 分,其中 14、15 两题,每题 6 分,每 16 题 8 分.答案正确 的,按下列答案后面括号内的分数给分;答错的,不答的,都给 0 分. 14. )(32 1234 2 SSf  (6 分) 15.加大蜡烛和光屏之间的距离(3 分) L dL 4 22  (3 分) 16.实验电路图如图(4 分.变阻器接成分压电路和电流表“内接”的都同样给分)(电路 图中有任何错误都不给这 4 分). 用螺旋测微器测量金属丝的直径 d (1 分).用米尺测金属丝的长度l (1 分) Il Ud 4 2 (2 分),式中 U 是电压表测得的电压值,I 是电流表测得的电流值. 四、参考解答及评分标准: 17.参考解答: 光线从光密介质射向光疏介质时,折射角大于入射角.若入射角增大到某一角度 C,使 折射角达到 90 ,折射光就消失.入射角大于 C 时只有反射光.这种现象称为全反射.相 应的入射角 C 叫做临界角. 光线由折射率为 n 的玻璃到真空,折射定律为 inr sinsin  ① 其中 ri和 分别为入射角和折射角.当入射角i 等于临界角 C 时,折射角 r 等于 90 ,代入① 式得 nC 1sin  ② 评分标准:本题 11 分. 能正确叙述全反射现象及临界角定义的,给 6 分(如果没有说明全反射,只说临 界角是发生全反射的最小入射角的,只给 2 分).导出②式或 nC 1arcsin 的,给 5 分. 18.参考解答: 设两星质量分别为 21 MM 和 ,都绕连线上 O 点作周期为 T 的圆周运动,星球 1 和星球 2 到 O 的距离分别为 21 ll 和 .由万有引力定律和牛顿第二定律及几何条件可得 1 2 12 21 )2( lTM R MMG  ① 2 2 22 21 )2( lTM R MMG  ② Rl  21 ③ 联立解得 2 32 21 4 GT RMM  ④ 评分标准:本题 12 分.①、②、③、④各占 3 分. 19.参考解答: 令  表示 AB、CD 上单位长度电阻丝的电阻,当 EF 处于图中位置时,设 EB、FD 两段 电阻丝的长度皆为l .由于电压表是理想电压表,故电压表的读数就是 EF 两端的电压.由 分压关系得, 12 UlRr R    ① 当 EF 由图示位置向左移动一段距离 L ,EB、FD 两段电阻丝的总电阻为 )(2 Ll  . 由分压关系得 2)(2 ULlRr R    ② 当 EF 由 图 示 位 置 向 右 移 动 一 段 距 离 L , EB 、 FD 两 段 电 阻 丝 的 总 电 阻 为 )(2 Ll  .由分压关系得, 3)(2 ULlRr R    ③ 由以上三式,代入数值解得 VU 63  ④ 评分标准:本题 12 分. ①、②、③、④式各 3 分. 20.参考解答: ab 棒向 cd 棒运动时,两棒和导轨构成的回路面积变小,磁通量发生变化,于是产生感 应电流.ab 棒受到与运动方向相反的安培力作用作减速运动,cd 棒则在安培力作用下作加 速运动.在 ab 棒的速度大于 cd 棒的速度时,回路总有感应电流, ab 棒继续减速, cd 棒 继续加速.两棒速度达到相同后,回路面积保持不变,磁通量不变化,不产生感应电流,两 棒以相同的速度 v 作匀速运动. (1)从初始至两棒达到速度相同的过程中,两棒总动量守恒,有 mvmv 20  ① 根据能量守恒,整个过程中产生的总热量 2 0 22 0 4 1)2(2 1 2 1 mvvmmvQ  ② (2)设 ab 棒的速度变为初速度的 4 3 时, cd 棒的速度为 'v ,则由动量守恒可知 '4 3 00 mvvmmv  ③ 此时回路中的感应电动势和感应电流分别为 Blvv )'4 3( 0  ④ RI 2  ⑤ 此时 cd 棒所受的安培力 IBlF  ⑥ cd 棒的加速度 m Fa  ⑦ 由以上各式,可得 mR vlBa 4 0 22  ⑧ 评分标准:本题 12 分. 第(1)问 6 分,其中①、②各 3 分.第(2)问 6 分,其中③式 1 分,④式 2 分,⑤式 1 分,⑧式 2 分. 21.参考解答: (1)设气缸内气体压强为 FP, 为细线中的张力,则活塞 A、B 及细杆这个整体的平衡 条件为 000  FSPPSPSSP BBAA 解得 BA SS FPP  0 ① 对于初始状态, NFF 301  , 代入①式,就得到气缸中气体的初始压强 PaSS FPP BA 51 01 102.1  由①式看出,只要气体压强 0PP  ,细线就会拉直且有拉力,于是活塞不会移动.使 气缸内气体温度降低是等容降温过程,当温度下降使压强降到 0P 时,细线拉力变为零,再 降温时活塞开始向右移,设此时温度为 2T ,压强 02 PP  .有 1 0 1 2 P P T T  ② 得 KT 4502  . ③ (2)再降温,细线松了,要平衡必有气体压强 0PP  .是等压降温过程,活塞右移、 体积相应减小,当 A 到达两圆筒联接处时,温度为 3T , 32 32 T lS T lSlS BBA  ④ 得 KT 2703  ⑤ (3)维持 KT 2703  不变,向左推活塞,是等温过程,最后压强为 4P .有 BA B lSlS lS P P  2 3 0 4 ⑥ 推力 2F 向左,由力的平衡条件得 0210440  FFSPSPSPSP BBAA ⑦ 解得 NF 902  ⑧ 评分标准:本题 14 分. 第(1)问 6 分,其中①、②、③式各 2 分.第(2)问 4 分,其中④、⑤式各 2 分.第(3)问 4 分,其中⑥式 2 分,⑦式 1 分,⑧式 1 分. 22.参考解答: 先假设小物块 C 在木板 B 上移动 x 距离后,停在 B 上.这时 A、B、C 三者的速度相等, 设为 V.由动量守恒得 VMmmv )2(0  ① 在此过程中,木板 B 的位移为 s ,小木块 C 的位移为 xs  .由功能关系得 2 0 2 2 1 2 1)( mvmVxsmg   222 1 MVmgs  相加得 2 0 2 2 1)2(2 1 mvVMmmgx  ② 解①、②两式得 gmM Mvx )2( 2 0  ③ 代入数值得 mx 6.1 ④ Bx比 板的长度l 在.这说明小物块 C 不会停在 B 板上,而要滑到 A 板上.设 C 刚滑到 A 板上的速度为 1v ,此时 A、B 板的速度为 1V ,则由动量守恒得 110 2MVmvmv  ⑤ 由功能关系得 mglMVmvmv  2 1 2 1 2 0 22 1 2 1 2 1 ⑥ 以题给数据代入解得 20 248 1 V 5 242 5 24821 v 由于 1v 必是正数,故合理的解是 smV /155.020 248 1  , ⑦ smv /38.15 242 1  ⑧ 当滑到 A 之后,B 即以 smV /155.01  做匀速运动.而 C 是以 smv /38.11  的初速在 A 上向右运动.设在 A 上移动了 y 距离后停止在 A 上,此时 C 和 A 的速度为 2V ,由动量守 恒得 211 )( VMmmvMV  ⑨ 解得 smV /563.02  ⑩ 由功能关系得 mgyVMmMVmv  2 2 2 1 2 1 )(2 1 2 1 2 1 解得 my 50.0 y 比 A 板的长度小,故小物块 C 确实是停在 A 板上.最后 A、B、C 的速度分别为 smVVA /563.02  , smVVB /155.01  , smVV AC /563.0 . 评分标准:本题 14 分. 正确论证了 C 不能停在 B 板上而是停在 A 板上,占 8 分.求出 A、B、C 三者的最后速度, 占 6 分. 2001 年普通高等学校招生全国统一考试(广东、河南卷) 物 理 本试卷分第Ⅰ卷(选择题)和第Ⅱ卷(非选择题)两部分,第Ⅰ卷 1 至 3 页,第Ⅱ卷 4 至 11 页,共 150 分,考试时间 120 分钟. 第Ⅰ卷(选择题 共 40 分) 注意事项: 1.答第Ⅰ卷前,考生务必将自已的姓名、准考证号、考试科目、试卷类型(A 或 B)用铅 笔涂写在答题卡上. 2.每小题选出答案后,用铅笔把答题卡上对应题目的答案标号涂黑,如需改动,用橡皮擦 干净后,再选涂其它答案,不能答在试题卷上. 3.考试结束,将本试卷和答题卡一并交回. 一、本题共 10 小题;每小题 4 分,共 40 分,在每小题给出的四个选项中,有的小题只有一 个选项正确,有的小题有多个选项正确,全部选对的得 4 分,选不全的得 2 分,有选错 或不答的得 0 分. 1.一单摆做简谐振动,对摆球所经过的任何一点来说,相继两次通过该点时,摆球的 A.速度必相同 B.加速度必相同 C.动量必相同 D.动能必相同 2.在下列四个方程中,x1、x2、x3 和 x4 各代表某种粒子. ① nU 1 0 235 92  1 138 54 95 38 3XeU x ② 2 2 1H x nHe 1 0 3 2  ③ U235 92 3 234 90Th x ④ HeMg 4 2 24 12  4 27 13 Al x 以下判断中正确的是 A.x1 是中子 B.x2 是质子 C.x3 是粒子 D.x4 是氘核 3.一个理想变压器,原线圈和副线圈的匝数分别为 n1 和 n2,正常工作时输入和输出的电压、 电流、功率分别为 U1 和 U2、I1 和 I2、P1 和 P2,已知 n1>n2,则 A.U1>U2,P1<P2 B.P1=P2 ,I1<I2, C.I1<I2,U1>U2 D.P1>P2,I1>I2 4.在 X 射线管中,由阴极发射的电子被加速后打到阳极,会产生包括 X 光在内的各种能量 的光子,其中光子能量的最大值等于电子的动能.已知阳极与阴极之间的电势差 U、普 郎克常数 h、电子电量 e 和光速 c,则可知该 X 射线管发出的 X 光的 A.最短波长为 eUh c B.最长波长为 eU ch C.最小频率为 h eU D.最大频率为 h eU 5.如图所示,p 字形发光物经透镜 L 在毛玻璃光屏 M 上成一实像,观察者处于 E 处,他看 到屏 M 上的像的形状为 A.q B.p C.d D.b 6.一列简谐横波在图中 x 轴上传播,a、b 是其中相距为 0.3m 的两点,在某时刻,a 点质元 正位于平衡位置向上运动,b 点质元恰好运动到下方最大位移处.已知横波的传播速度为 60ms-1,波长大于 0.3m. A.若该波沿 x 轴负方向传播,则频率为 150Hz B.若该波沿 x 轴负方向传播,则频率为 100Hz C.若该波沿 x 轴正方向传播,则频率为 75Hz D.若该波沿 x 轴正方向传播,则频率为 50Hz 7.如图所示,虚线框 abcd 内为一矩形匀强磁场区域,ab=2bc,磁场方向垂直于纸面;实 线框a´b´c´d´是一正方形导线框,a´b´边与ab边平行.若将导线框匀速地拉离磁场区域, 以 W1 表示沿平行于 ab 的方向拉出过程中外力所做的功,W2 表示以同样速率沿平行于 bc 的方向拉出过程中外力所做的功,则 A.W1=W2 B.W2=2W1 C.W1=2W2 D.W2=4W1 8.按照玻尔理论,下列关于氢原子的论述正确的是 A.第 m 个定态和第 n 个定态的轨道半径 rm 和 rn 之比为 rm∶rn=m2∶n2B.C.D. B.第 m 个定态和第 n 个定态的能量 Em 和 En 之比为 Em∶En=n2∶m2 C.电子沿某一轨道绕核运动,若其圆周运动的频率是 v,则其发光频率也是 v D.若氢原子处于能量为 E 的定态,则其发光频率为 v= h E 9.惯性制导系统已广泛应用于弹道导弹工程中,这个系统的重要元件之一是加速度计.加速 度计的构造原理的示意图如图所示:沿导弹长度方向安装的固定光滑杆上套一质量为 m 的滑块,滑块两侧分别与劲度系数均为 k 的弹簧相连;两弹簧的另一端与固定壁相连. 滑块原来静止,弹簧处于自然长度.滑块上有指针,可通过标尺测出滑块的位移,然后 通过控制系统进行制导,设某段时间内导弹沿水平方向运动,指针向左偏离 0 点的距离 为 s,则这段时间内导弹的加速度 A.方向向左,大小为 ks/m B.方向向右,大小为 ks/m C.方向向左,大小为 2ks/m D.方向向右,大小为 2ks/m 10.如图,平行板电容器经开关 K 与电池连接,a 处有一带电量非常小的点电荷.K 是闭合 的,Ua 表示 a 点的电势,f 表示点电荷受到的电场力.现将电容器的 B 板向下稍微移动, 使两板间的距离增大,则 A.Ua 变大,f 变大 B.Ua 变大,f 变小 C.Ua 不变,f 不变 D.Ua 不变,f 变小 第Ⅱ卷(非选择题 共 110 分) 注意事项: 1.第Ⅱ卷共 8 页,用钢笔或圆珠笔直接答在试题卷中(除题目有特殊规定外). 2.答卷前将密封线内的项目真写清楚. 二、本题共 3 小题;每小题 5 分,共 15 分,把答案填在题中的横线上. 11.某测量员是这样利用回声测距离的:他站在两平行峭壁间某一位置鸣枪,经过 1.00 秒 钟一次听到回声,又经过 0.50 秒钟再次听到回声. 已知声速为 340m/s,则两峭壁间的 距为 m. 12.如图所示,质量为 m、横截面为直角三角形的物块 ABC,∠ABC=,AB 边靠在竖直墙面上,F 是垂直于斜面 BC 的推力. 现物块静止不动,则摩擦 力的大小为 . 13.如图所示,q1、q2、q3 分别表示在一条直线上的三个点电荷,已知 q1 与 q2 之间的距离为 l1,q2 与 q3 之间的距离为 l2,且每个电荷 都处于平衡状态. (1)如 q2 为正电荷,则 q1 为 电荷,q3 为 电荷. (2)q1、q2、q3 三者电量大小之比是 ∶ ∶ . 三、本题共 3 小题,共 20 分,把答案填在题中的横线上或按题目要求作图. 14.(5 分)某同学以线状白炽灯为光源,利用游标卡尺两脚间形成的狭缝观察光的衍射现 象后,总结出以下几点: a.若狭缝与灯丝平行,衍射条纹与狭缝平行 b.若狭缝与灯丝垂直,衍射条纹与狭缝垂直 c.衍射条纹的疏密程度与狭缝的宽度有关 d.衍射条纹的间距与光的波长有关 以上几点中,你认为正确的是 . 15.(6 分)一打点计时器固定在斜面上某处,一小车拖着穿过打点计时器的纸带从斜面上 滑下,如图 1 所示. 图 2 是打出的纸带的一段. (1)已知打点计时器使用的交流电频率为 50Hz,利用图 2 给出的数据可求出小车下滑的加速度 a= . (2)为了求出小车在下滑过程中所受的阻力,还需测量 的物理量有 . 用测得的量及加速度 a 表示阻力的计算式为 f= . 16.(9 分)图 1 中 E 为电源,其电动势为,R1 为滑线变阻器,R2 为电阻箱,A 为电流表. 用 此电路,经以下上步骤可近似测得 A 的内阻 RA:①闭合 K1,断开 K2,调节 R1,使电流 表读数等于其量程 I0;②保持 R1 不变,闭合 K2,调节 R2,使电流表读数等于 2 0I ,然 后读出 R2 的值,取 RA≈R2 . (1)按图 1 所示电路在图 2 所给出的实物图中画出连接导线. (2)真实值与测得值之差除以真实值叫做测量结果的相对误差,即 A A R RR 2 . 试导出它与电 源电动势、电流表量程 I0 及电流表内阻 RA 的关系式. (3)若 I0=10mA,真实值 RA 约为 30,要想使测量结果的相对误差不大于 5%,电源电 动势最小应为多少伏? 四、本题共 6 小题,75 分,解答应写出必要的文字说明、方程式和重要演算步骤,只写出 最后答案的不能得分. 有数值计算的题,答案中必须明确写出数值和单位. 17.(12 分)质量为 M 的小船以速度 V0 行驶,船上有两个质量皆为 m 的小孩 a 和 b,分别 静止站在船头和船尾. 现小孩 a 沿水平方向以速率 v(相对于静止水面)向前跃入水中, 然后小孩 b 沿水平方向以同一速率 v(相对于静止水面)向后跃入水中. 求小孩 b 跃出 后小船的速度. 18.(12 分)如图所示,在 y<0 的区域内存在匀强磁场,磁场方向垂直于 xy 平面并指向纸 面外,磁感强度为 B. 一带正电的粒子以速度 v0 从 O 点射入磁场,入射方向在 xy 平面 内,与 x 轴正向的夹角为,若粒子射出磁场的位置与 O 点的距离为 l,求该粒子的电 量和质量之比 m q . 19.(12 分)无人飞船“神州二号”曾在离地面高度为 H=3.4×105m 的圆轨道上运行了 47 小时. 求在这段时间内它绕行地球多少圈?(地球半径 R=6.37×106m,重力加速度 g =9.8m/s2) 20.(13 分)如图 1 所示. 一对平行光滑轨道放置在水平面上,两轨道间距 l=0.20m,电阻 R=1.0;有一导体杆静止地放在轨道上,与两轨道垂直,杆及轨道的电阻皆可忽略不 计,整个装置处于磁感强度 B=0.50T 的匀强磁场中,磁场方向垂直轨道面向下. 现用 一外力 F 沿轨道方向拉杆,使之做匀加速运动,测得力 F 与时间 t 的关系如图 2 所示. 求 杆的质量 m 的加速度 a . 21.(13 分)在一密封的啤酒瓶中,下方为溶有 CO2 的啤酒,上方为纯 CO2 气体. 在 20℃时, 溶于啤酒中的 CO2 的质量为 mA=1.050×10-3kg,上方气体状态 CO2 的质量为 mB=0.137 ×10-3kg,压强为 p0=1 标准大气压. 当温度升高到 40℃时,啤酒中溶解的 CO2 的质量 有 所 减 少 , 变 为 Am = mA - △ m , 瓶 中 气 体 CO2 的 压 强 上 升 到 p1. 已 知 : 0 160.0 p p m m A A  ,啤酒的体积不因溶入 CO2 而变化,且不考虑容器体积的啤酒体积 随温度的变化. 又知对同种气体,在体积不变的情况下 T P 与 m 成正比. 试计算 p1 等于 多少标准大气压(结果保留两位有效数学). 22.(13 分)一个圆柱形的竖直的井里存有一定量的水,井的侧面和底部是密闭的,在井中 固定地插着一根两端开口的薄壁圆管,管和井共轴,管下端未触及井底. 在圆管内有一 不漏气的活塞,它可沿圆管上下滑动.开始时,管内外水面相齐,且活塞恰好接触水面, 如图所示. 现用卷扬机通过绳子对活塞施加一个向上的力 F,使活塞缓慢向上移动,已 知管筒半径 r=0.100m,井的半径 R=2r,水的密度=1.00×103kg/m/3,大气压0=1.00 ×105Pa. 求活塞质量,不计摩擦,重力加速度 g=10m/s2.) 2001 年普通高等学校招生全国统一考试(广东卷、河南卷) 物理试题答案 说明: (1)定出评分标准是为了使全国各地尽可能在统一标准下评定成绩。试题的参考解答 是用来说明评分标准的。考生如按其它方法或步骤解答,正确的,同样给分;有错的,根据 错误的性质,参照评分标准中相应的规定评分。 (2)第一、二、三题只要求写出答案,不要求说明理由或列出算式,只根据答案评分。 (3)第四大题,只有最后答案而无演算过程的,不给分;只写出一般公式但未能与试 题所给的具体条件联系的,不给分。 一、答案及评分标准:全题 40 分,每小题 4 分。每小题全选对的给 4 分,选不全的给 2 分, 有选错的给 0 分,不答的给 0 分。 1.BD 2.AC 3.BC 4.D 5.C 2.AD 7.B 8.AB 9.D 10.B 二、答案及评分标准:全题 15 分,每小题 5 分,答案正确的,按下列答案后面括号内的分 数给分;答错的,不答的,都给 0 分。 11.425 (5 分) 12.mg+Fsin(5 分) 13.(1)负 负 (2 分,两空都对才给这 2 分) (2) 2 1 212 2 21 )(:1:)( l ll l ll  (3 分,三者之比正确才给这 3 分) 三、答案及评分标准:全题 20 分,其中 14 题 5 分,15 题 6 分,16 题 9 分,答案正确的, 按下列答案后面括号内的分数给分;答错的,不答的,都给 0 分。 14.a,c,d (答对 1 个给 1 分,答对 2 个给 3 分,答对 3 个给 5 分,有错就不给分) 15.(1)4.00m/s2 (3 分,3.90—4.10m/s2 之间都正确) (2)小车质量;斜面上任意两点间距离及这两点的高度差。(1 分,有错就不给分) mal hmg  (2 分) 16.(1)连线如图所示 (2)由步骤①得 0 1 IRR A   ① 由步骤②得 2 0 2 2 2 2 1 I RR R AR RRR A A A     ② 解得 A A A RI R RR  02  ③ (3)6V 评分标准:本题共 9 分。第(1)问 3 分,有错就不给分;第(2)问 5 分,其中①式 1 分, ②式 2 分,③式 2 分;第(3)问 1 分。 四、参考解答及评分标准: 17.参考解答: 设小孩 b 跃出后小船向前行驶的速度为 V。根据动量守恒定律,有 (M+2m)V0=MV+m-m ① 解得 0)21( VM mV  评分标准:本题 12 分。①式 8 分,②式 4 分。 18.参考解答: 带正电粒子射入磁场后,由于受到洛仑兹力的作用, 粒子将沿图示的轨迹运动,从 A 点射出磁场,O、A 间 的距离为 l。射出时速度的大小仍为,射出方向与轴的 夹角仍为。由洛仑兹力公式和牛顿定律可得, RmBq 2 0 0   式中 R 为圆轨道的半径 解得 qB mR 0 ① 圆轨道的圆心位于 OA 的中垂线上,由几何关系可得 sin2 Rl  ② 联立①、②两式,解得 lBm q  sin2 0 ③ 评分标准:本题 12 分。①式 4 分,②式 6 分,③式 2 分。 19.参考解答: 用 r 表示飞船圆轨道半径 r=R+H=6.71×206 m 式中 M 表示地球质量,m 表示飞船质量, 表示飞船绕地球运行的角速度,G 表示万有引 力常数。利用 g R MG  2 及①式,得 2 3 2  r gR ② 由于 T  2 ,T 表示周期。解得 g r R rT 2 ③ 代入数值解得绕行圈数为 n=31 评分标准:本题 12 分 ①、②、③式各 3 分。得出正确结果再给 3 分(结果得 29 圈至 32 圈的都给这 3 分)。 20.参考解答: 导体杆在轨道上做匀加速直线运动,用表示其速度,t 表示时间,则有 =at ① 杆切割磁力线,将产生感应电动热,  =Bl ② 在杆、轨道和电阻的闭合回路中产生电流 RI  ③ 杆受到的安培力的 f=Ibl ④ 根据牛顿第二定律,有 F-f=ma ⑤ 联立以上各式,得 atR lBmaF 22  ⑥ 由图线上取两点代入⑥式,可解得 a=10m/s,m=0.1kg 评分标准:本题 13 分。得出⑥式给 6 分,得出最后结果再给 7 分。 21.参考解答: 在 40℃时,溶入啤酒的 CO2 的质量为 m’A=mA-△m ① 因质量守恒,气态 CO2 的质量为 m’s=ms+△m ② 由题设, 0 160.0' P P m m A A  ③ 由于对同种气体,体积不变时, T P 与 m 成正比,可得 293 313' 0 1   s s m m P P ④ 由以上各式解得 01 313 2936.0 1 P m m m m P B A B A                ⑤ 算得 P1=1.6 标准大气压 ⑥ 评分标准:本题 13 分。②式 3 分,④式 4 分,⑤式 4 分,⑥式 2 分。 22.参考解答: 从开始提升到活塞升至内外水面高度差为 mPg Ph 100 0  的过程中,活塞始终与管内液 体接触。(再提升活塞时,活塞和水面之间将出现真空,另行讨论。)设活塞上升距离为 h1, 管外液面下降距离为 h2, h0= h1+h2 ① 因液体体积不变,有 122 2 12 3 1)( h rR rhh      ② 得 mmhh 5.7104 3 4 3 01  ③ 题给 H=9m>h1,由此可知确实有活塞下面是真空的一段过程。 活塞移动距离从零到 h1 的过程中,对于水和活塞这个整体,其机械能的增量应等于除 重力外其他力所做的功。因为始终无动能,所以机械能的增量也就等于重力势能增量,即 2)( 0 1 2 hghrE  ④ 其他力有管内、外的大气压力和拉力 F。因为液体不可压缩,所以管内、外大气压力做 的总功 P0(R2-r2)h2-P0r2h1=0,故外力做功就只是拉力 F 做的功,由功能关系知 W1=△E ⑤ 即 Jg PrhgrW 4 2 022 0 2 1 1018.18 3 8 3)(   ⑥ 活塞移动距离从 h1 到 H 的过程中,液面不变,F 是恒力 F=r2P0,做功 W2=F(H-h1)= r2P0(H-h1)=4.71×103 J ⑦ 所求拉力 F 做的总功为 W1+W2=1.65×104 J ⑧ 评分标准:本题 13 分。③式 4 分,⑥式 3 分,⑦式 5 分,⑧式 1 分。 2001 年全国普通高等学校招生全国统一考试 物 理 本试卷分第Ⅰ卷(选择题)和第Ⅱ卷(非选择题),第Ⅰ卷 1 至 3 页,第Ⅱ卷 4 至 11 页共 150 分,考试时间 120 分钟。 第Ⅰ卷(选择题 共 40 分) 一、本题共 10 小题;每小题 4 分,共 40 分,在每小题给出的四个选项中,有的小题只有一 个选项正确,有的小题有多个选项正确,全部选对的得 4 分,先不全的得 2 分,有选错 或不答的得 0 分。 1. 在下列的四个方程中,X1、X2、X3 和 X4 各代表某种粒子 ① ② ③ ④ 以下判断正确的是 A.X1 是中子 B.X2 是质子 C.X3 是α粒子 D.X4 是氘核 2.一个理想变压器,原线圈和副线圈的匝数分别是 n1 和 n2,正常工作时输入和输出的电压、 电流、功率分别为 U1 和 U2、I1 和 I2、P1 和 P2,已知 n1>n2,则 A.U1>U2 P1U2 D.P1>P2 I1>I2 3.在 x 射线管中,由阴极发射的电子被加速后打到阳极,会产生包括 X 光在内的光子,其中 光子能量的最大值等于电子的动能。已知阳极与阴极之间的电势差 U。普朗克常数 h, 电子电量 e 和光速 c,则可知该 x 射线管发出的 X 光的 A.最短波长为 B.最长波长为 C.最小频率为 D.最大频率为 4.如图所示,p 字形发光物经透镜 L 在毛玻璃光屏 M 上成一实像,观察者处于 E 处,他看 到屏 M 上的像的形状为 A.q B.p C. d D.b 5.如图所示,虚线框 abcd 内为一矩形匀强磁场区域,ab=2bc, 磁 场方向垂直于纸面,是线框 是一正方形导线框, 边 与 ab 边平行。若将导线框匀速地拉离磁场区域,以 表示沿 平行于 ab 的方向拉出过程中外力所做的功。 表示以同样的 速率沿平行于 bc 的方向拉出过程中外力所做的功,则 A. B. C. D. 6.按照玻尔理论,下列关于氢原子的论述正确的是 A.第 m 个定态和第 n 个定态的轨道半径 之比为 B.第 m 个定态和第 n 个定态的能量 之比为 C.电子沿某一轨道绕核运动,若其圆周运动的频率是 ,则其发光频率也是 D.若氢原子处于能量为 E 的定态,则其发光频率为 7.如图,虚线 A.b 和 c 是某静电场中的等势面,它们的电势分别为 、 。 一带正电的粒子射入电场中,其运动轨迹如实线 KLMN 所示,由图可知: A. 粒子从 K 到 L 的过程中,电场力做负功。 B. 粒子从 L 到 M 的过程中,电场力做负功。 C. 粒子从 K 到 L 的过程中,静电势能增加。 D.粒子从 L 到 M 的过程中,动能减少。 8.惯性制导系统已广泛应用于弹道式导弹工程中,这个系统的重要元件之一是加速度计, 加速度计的构造原理的示意图如图所示:沿导弹长度方向安装的固定光滑杆上套一质量 为 m 的 滑块,滑块两侧分别与劲度系数均为 k 的弹簧相连;两弹簧的另一端与固定端 相连,滑块原来静止,弹簧处于自然长度,滑块上有指针,可通过标尺测出滑块的位移, 然后通过控制系统进行制导,设某段时间内导弹沿水平方向运动,指针向左偏离 O 点的 距离为 x,则这段时间内导弹的加速度 A. 方向向左,大小为 B. 方向向右,大小为 C. 方向向左,大小为 2 D.方向向右,大小为 2 9.细长轻绳下端栓一小球构成单摆,在悬挂点正下方 摆长处有一个能挡住摆线的钉子 A, 如图所示,现将单摆向左方拉开一个小角度,然后无初速度地释放,对于以后的运动, 下列说法正确的是 A.摆球往返运动一次的周期比无钉子时的单摆周期小 B.摆球在左、右两侧上升的最大高度一样 C.摆球在平衡位置左右两侧走过的最大弧长相等 D.摆球在平衡位置右侧的最大摆角是左侧的两倍。 10.如图所示,在平面 xy 内有一沿水平轴 x 正向传播的简谐横波,波速为 3.0m/s,频率为 2.5HZ,振幅为 。已知 t=0 时刻 P 质点的位移为 ,速度沿 y 轴正向。Q 点在 P 点右方处,对于 Q 点的质元来说 A. 在 t=0 时,位移为 y= B. 在 t=0 时,速度沿 y 轴负方向。 C. 在 t=0.1s 时,位移为 y= D.在 t=0.1s 时,速度沿 y 轴正方向。 第Ⅱ卷(非选择题 共 110 分) 二、本题共 3 小题;每小题 5 分,共 15 分,把答案填在题中的横线上。 11.某测量员是这样利用回声测距离的:他站在两平行峭壁间某一位置鸣枪,经过 1.00 秒 钟第一次听到回声,又经过 0.50 秒钟再次听到回声。已知哦声速为 340m/s,则两峭壁 间的距离为 。 12.如图所示,质量为 m,横截面为直角三角形的物块 ABC, 。 AB 边靠在竖直墙面上,F 是垂直于斜面 BC 的推力。现物块静止不动, 则摩擦力的大小为 。 13.如图所示, ,分别表示在一条直线上的三个点 电荷,已知 之间的距离为 ,已知 之间的距 离为 ,且每个电荷都处于平衡状态 (1)如 为负电荷,则 为 电荷, 为 电荷。 (2) 三者电量大小之比是 : : 三、本题共 3 小题,共 20 分,把答案填在题中的横线上或按题目要求作图。 14.(5 分)某同学以线状白炽灯为光源,利用游标卡尺两脚间形成的狭缝观察光的衍射现 象后,总结出以下几点: a.若狭缝与灯丝平行,衍射条纹与狭缝平行 b.若狭缝与灯丝垂直,衍射条纹与狭缝垂直 c.衍射条纹的疏密程度与狭缝宽度有关 d.衍射条纹的间距与光的波长有关 以上几点中,你认为正确的是 15.(6 分)一打点计时器固定在斜面上某处,一小车拖着穿过条点计时器的纸带从斜面上 滑下,如图 1 所示,图 2 是打出的纸带的一段。 (1)已知打点计时器使用的交流电频率为 50Hz,利用图 2 给出的数据可求出小四下滑的加 速度 a= (2)为了求出小车在下滑过程中所受的阻力,还需测量的物理量有 用测得的量及加速度 a 表示阻力的计算式为 f= 16.(9 分)图 1 中 E 为电源,其电动势为ε,R1 为滑线变阻器,R2 为电阻箱,A 为电流表, 用此电路,经以下步骤可近似得 A 的内阻 RA:①闭合 K1,断开 K2,调节 R1 使电流表读 数等于其量程 I0;②保持 R1 不变,闭合 K2,调节 R2,使电流表读数等于 I0/2,然后读 出 R2 的值,取 RA (1)按图 1 所示电路在图 2 中所给的实物图中画出连接导线。 (2)真实值和测量值之差除以真实值叫测量结果的相对误差,即 ,试导出它与电 动势ε,电流表量程 I0 及电流表内阻 RA 的关系式。 (3)若 I0=10mA,真实值 RA 约为 30Ω,要想使测量结果相对误差不大于 5%,电源电动势最 小应为多少伏? 四、本题共 6 小题,75 分,解答应写出必要的文字说明、方程式和重要演算步骤,只写出 最后答案的不能得分,有数值计算的题,答案中必须明确写出数值和单位。 17.(12 分)质量为 M 的小船以速度 V0 行驶,船上有两个质量皆为 m 的小孩 a 和 b,分别 静止站在船头和船尾,现小孩 a 沿水平方向以速率 (相对于静止水面)向前跃入水中, 然后小孩 b 沿水平方向以相同的速率 (相对于静止水面)向后跃入水中,求小孩 b 跃 出后小船的速度。 18.(12 分)如图所示,在 y<0 的区域内存在匀强磁场,磁场方向垂直于 xy 平面并指向纸 面外,磁感强度为 B,一带正电的粒子以速度 V0 从 O 点射入磁场,入射方向在 xy 平面 内,与 x 轴正方向的夹角为θ,若粒子射出磁场的位置与 O 点的距离为 l,求该粒子的 电量和质量之比 q/m。 19.(12 分)“和平号”空间站已于今年 3 月 23 日成功地坠落在太平洋海域,坠落过程可 简化为从一个近圆轨道(可近似看作圆轨道)开始,经过与大气摩擦,空间站的绝大 部分经过升温、熔化,最后汽化面销毁,剩下的残片坠入大海。此过程中,空间站原 来的机械能中,除一部分用于销毁和一部分被残片带走外,还有一部分能量 通过 其他方式散失(不考虑坠落过程中化学反应的能量)(1)试导出以下各物理量的符 号表示散失能量 的公式。(2)算出 的数值。(结果保留两位有效数字) 坠落开始时空间站的质量 ; 轨道离地的高度为 h=146Km 地球半径 R=6.4×106m; 坠落窨范围内重力加速度可看作 g=10m/s2; 入海残片的质量 m=1.2×104Kg; 入海残片的温升高ΔT=3000K; 入海残片的入海速度为声速 ; 空间站材料每 1 千克升温 1K 平均所需能量 C=1.0×103J; 每销毁 1 千克材料平均所需能量 J。 20.(13 分)如图 1 所示,一对平行光滑轨道放置在水平面上,两轨道间距 l=0.20m,电阻 R=1.0Ω,有一导体杆静止放在轨道上,与两轨道垂直,杆及轨道的电阻可忽略不计, 整个装置处于磁感强度 B=0.50T 的匀强磁场中,磁场方向垂直轨道面向下,现用一外力 F 沿轨道方向拉杆,使之做匀运动,测得力 F 与时间 t 的关系如图 2 所示,求杆的质量 m 和加速度 a. 21.(13 分)在一密封的啤酒瓶中,下方为溶有 CO2 的啤酒,上方为纯 CO2 气体,在 20℃时, 溶液于啤酒中的 CO2 的质量为 mA=1.050×10-3Kg,上方气体状态 CO2 质量为 mB=0.137×10-3Kg,压强为 P0=1 个大气压,当温度升高到 40℃时,啤酒中溶解的 CO2 的 质量减少,变为 ,瓶中气体 CO2 的压强上升到 P1,已知 。 啤酒的体积不因溶入 CO2 而变化,且不考虑容器体积和啤酒体积随温度变化,又知对同 种气体,在体积不变的情况下 P/T 与 m 成正比,试计算 P1 等于多少标准大气压(结果 保留两位有效数字)。 22.(13 分)一个圆柱形的竖直的井里存有一定量的水,井的侧面和底部是密闭的,在井 中固定地插着一根两端开口的薄壁圆管,管和井共轴,管下端未触及井底,在圆管内有 一不漏气的活塞,它可沿圆管上下滑动,开始时,管内外水面相齐,且活塞恰好触及水 面,如图所示,现用卷扬机通过绳子对活塞施加一个向上的力 F,使活塞缓慢向上移动, 已知管筒半径 r=0.100m,井的半径 R=2r,水的密度ρ=1.00×103Kg/m3,大气压强为 P0=1.00×103Pa,求活塞上升 H=9.00m 的过程中拉力 F 所做的功。(井和管在水面以上 及水面以下的部分足够长,不计活塞质量,不计摩擦,重力加速度 g=10m/s2)。 2001 年全国普通高等学校招生全国统一考试 物理参考答案 一、答案及评分标准:全题 40 分,每小题 4 分,每小题分选对的给 4 分,选不分的给 2 分, 有选错的给 0 分,不答的给 0 分。 1.AC 2.BC 3.D 4.C 5.B 6.AB 7.AC 8.D 9.AB 10.BC 二、答案及评分标准:全题 15 分,每小题 5 分,答案正确的,按下列答案后面括号内的分 数给分;答错的,不答的,都给 0 分。 11.425(5 分) 12.mg+Fsina (5 分) 13.(1)负 负 (2 分,两空都对才给 2 分) (2) 2 1 212 2 21 )(:1:)( l ll l ll  (3 分,三者之比正确才给 3 分) 三、答案及评分标准: 14.a,c,d (答对 1 个给 1 分,答对 2 个给 3 分,答对 3 个给 5 分) 15.(1)4.00/s2 (3 分,3.90-4.10/s2 之间都正确) (2)小车质量 m;斜面上任意两点间距离 l 及这两点的高度差 h。(1 分,有错就不给分) mal hmg  16.(1)连线如图示 (3)6v 四、答案及评分标准: 17. 18. 19. 20. 21. 22. 2001 年全国普通高等学校招生全国统一考试 物理(上海卷) 考生注意: 1.答卷前,考生分必将姓名、高考座位号、校验码等填写清楚。素 2.本试卷共 8 页,23 题,满分 150 分.考试时间 120 分钟。请考生用钢笔或圆珠笔将答案 直接写在试卷上。 3.第 19、20、21、22、23 题要求写出必要的文字说明、方程式和重要的演算步骤。只写出 最后答案,而未写出主要演算过程的,不能得分。有数字计算的问题,答案中必须明确写出 数值和单位。 第Ⅰ卷(选择题 共 40 分) -、(40 分)选择题,本大题共 8 小题,每小题 5 分,每小题给出的四个答案中,至少有 一个是正确的,把正确答案全选出来,并将正确答案前面的字母填写在题后的方括号内, 每小题全选对的得 5 分;选对但不全,得部分分;有选错或不答的,得 0 分。填写在方 括号外的字母,不作为选出的答案。 1.跳伞运动员在刚跳离飞机、其降落伞尚未打开的一段时间内,下列说法中正确的是 A.空气阻力做正功 B.重力势能增加 C.动能增加 D.空气阻力做负功. 2.卢瑟福原子核式结构理论的主要内容有 A.原子的中心有个核,叫做原子核 B.原子的正电荷均匀分布在整个原子中 C.原子的全部正电荷和几乎全部质量都集中在原子核里 D.带负电的电子在校外绕着核旋转 3.A、B 两点各放有电量为十 Q 和十 2Q 的点电荷,A、 B、C、D 四点在同一直线上,且 AC =CD=DB.将一正电荷从 C 点沿直线移到 D 点,则 A.电场力一直做正功 B.电场力先做正功再做负功 B.电场力一直做负功 D.电场力先做负功再做正功 4.组成星球的物质是靠引力吸引在一起的,这样的星球有一个最大的自转速率.如果超过 了该速率,星球的万有引力将不足以维持其赤道附近的物体做圆周运动。由此能得到半 径为 R、密度为ρ、质量为 M 且均匀分布的星球的最小自转周期 T。下列表达式中正确 的是 A.T=2π B.T=2π C.T= D.T= 5.如图所示,有两根和水平方向成。角的光滑平行的金属轨道,上端接有可变电阻 R,下 端足够长,空间有垂直于轨道平面的匀强磁场,磁感强度为及一根质量为 m 的金属杆从 轨道上由静止滑下。经过足够长的时间后,金属杆的速度会趋近于一个最大速度几,则 A.如果 B 增大,vm 将变大 B.如果α变大,vm 将变大 C.如果 R 变大,vm 将变大 D.如果 m 变小,vm 将变大 6.如图所示是一种延时开关,当 S1 闭合时,电磁铁 F 将衔铁 D 吸下,C 线路接通。当 S1 断 开时,由于电磁感应作用,D 将延迟一段时间才被释放。则 A.由于 A 线圈的电磁感应作用,才产生延时释放 D 的作用 B.由于 B 线圈的电磁感应作用,才产生延时释放 D 的作用 C.如果断开 B 线圈的电键 S2,无延时作用 D.如果断开 B 线圈的电键 S2,延时将变长 7.如图所示的电路中,闭合电键,灯 L1、L2 正常发光,由于电路出现故障,突然发现灯 L1 变亮,灯 L2 变暗,电流表的读数变小,根据分析,发生的 故障可能是 A.R1 断路 B.R2 断路 C.R3 短路 D.R4 短路 8.一升降机在箱底装有若干个弹簧,设在某次事故中,升降机吊索在空中断裂,忽略摩擦 力,则升降机在从弹簧下端触地后直到最低点的一段运动过程 中, A.升降机的速度不断减小 B.升降机的加速度不断变大 C.先是弹力做的负功小于重力做的正功,然后是弹力做的负功大于 重力做的正功 D.到最低点时,升降机加速度的值一定大于重力加速度的值。 第Ⅱ卷(非选择题 共 110 分) 二、(20 分)填空题,本大题共 5 小题,每小题 4 分,答案写在题中横线上的空白处,不 要求写出演算过程。 9.请将右面三位科学家的姓名按历史年代先后顺序排列:______、______、______。 任选其中二位科学家,简要写出他们在物理学上的主要贡献各一项:______ ,______。 牛顿 爱因斯坦 伽利略 10.A、B 两幅图是由单色光分别射到圆孔而形成的图象,其中图 A 是光的______(填干涉 或衍射)图象。由此可以判断出图 A 所对应的圆孔的孔径______(填大于或小于)图 B 所对应的圆孔的孔径。 11.一束质量为 m、电量为 q 的带电粒子以平行于两极板的速度 v0 进入匀强电场,如图所示, 如果两极板间电压为 U,两极板间的距离为 d,板长为 L,设 粒子束不会击中极板,则粒子从进入电场到飞出极板时电势 能的变化量为______(粒子的重力忽略不计) 12.如图所示,有四列简谐波同时沿 x 轴正方向传播,波速分别是 v、2 v、3 v 和 4 v,a、 b 是 x 轴上所给定的两点,且 ab=l。在 t 时刻 a、b 两点间四列波的波形分别如图所示, 则由该时刻起 a 点出现波峰的先后顺序依次是图______;频率由高到低的先后顺序依次 是图 。 A B C D 13.图 A 是在高速公路上用超声波测速仪测量车速的示意图,测速仪发出并接收超声波冲信 号,根据发出和接收到的信号间的时间差,测出被测物体的速度。图 B 中 p1、、p2 是测 速仪发出的超声波信号,n1、n2 是 p1、p2 由汽车反射回来的信号。设测速仪匀速扫描, p1、p2 之间的时间间隔Δt=1.0 s,超声波在空气中传播的速度是 v=340m/s,若汽车 是匀速行驶的,则根据图 B 可知,汽车在接收到 p1、p2 两个信号之间的时间内前进的距 离是______,汽车的速度是______m/s。 图 A 图 B 三、(30 分)实验题,本大题共 5 小题。第 14、15 小题是选择题,每小题 5 分,第 16 小 题 6 分,第 17、18 小题各 7 分 14.(5 分)光电效应实验的装置如图所示,则下面说法中正确的是 A.用紫外光照射锌板,验电器指针会发生偏转 B.用红色光照射锌板,验电器指针会发生偏转 C.锌板带的是负电荷 D.使验电器指针发生偏转的是正电荷 15.(5 分)某同学用同一个注射器做了两次验证波意耳定 律的实验,操作完全正确。根据实验数据却在 P-V 图上画出了两条不同双曲线。造成这 种情况的可能原因是 A.两次实验中空气质量不同 B.两次实验中温度不同 C.两次实验中保持空气质量、温度相同,但所取的气体压的数据 不同 D.两次实验中保持空气质量、温度相同,但所取的气体体的数据不同。 16.(6 分)要求测量由 2 节干电池串联而成的电池组的电动势ε和内阻 r(约几欧),提 供下列器材:电压表 V(量程 3V,内阻 1kΩ)、电压表 V2(量程 15V,内阻 2kΩ)、 电阻箱(0~9999Ω)、电键、导线若干。 某同学用量程为 15 V 的电压表连接成如图所示的电路,实 验步骤如下: (l)合上电键 S,将电阻箱 R 阻值调到 R1=10Ω,读得电 压表的读数为 U1 (2)将电阻箱 R 阻值调到 R2=20Ω,读得电压表的读数为 U2,由方程组 U1=ε-U1r/R1、U2=ε-U2/R2 解出ε、r 为了减少实验误差,上述实验在选择器材和实验步骤中,应做哪些改进? 17.(7 分)利用打点计时器研究一个约 1. 4 高的商店卷帘窗的运动。将纸带 粘在卷帘底部,纸带通过打点计时器随帘在竖直面内向上运动。打印后的 纸带如图所示,数据如表格所示。纸带中 AB、BC、CD……每两点之间的时 间间隔为 0.10s,根据各间距的长度,可计算出卷帘窗在各间距内的平均速 度 V 平均。可以将 V 平均近似地作为该间距中间时刻的即时速度 V。 (1)请根据所提供的纸带和数据,绘出卷帘窗运动的 V-t 图线。 (2)AD 段的加速度为 m/s2,AK 段的平均速度为 m/s。 18.(7 分)某学生为了测量一物体的质量,找到一个力电转换器,该转换器的输出电压正 比于受压面的压力(比例系数为 k),如图所示.测量时先调节输入端的电压。使转换 器空载时的输出电压为 0;而后在其受压面上放一物体,即可测得与物体的质量成正比 的输出电压 U。现有下列器材:力电转换器、质量为 m0 的砝码、电压表、滑动变阻器、 干电池各一个、电键及导线若干、待测物体(可置于力电转换器的受压面上)。 请完成对该物体质量的测量。 (l)设计一个电路,要求力电转换器的输入电压可调,并且使电压的调节范围尽可能大, 在方框中画出完整的测量电路图。 (2)简要说明测量步骤,求出比例系数 k,并测出待测物体的质量 m. (3)请设想实验中可能会出现的一个问题。 四、(60 分)计算题 19.(10 分)(10)1791 年,米被定义为:在经过巴黎的子午线上,取从赤道到北极长度 的一千万分之一。请由此估算地求的半径 R。(答案保留二位有效数字) (2)太阳与地球的距离为 1.5×1011 m,太阳光以平行光束入射到地面。地球表面 2/3 的面 积被水面所覆盖,太阳在一年中辐射到地球表面水面部分的总能量 W 约为三 1.87×1024J。 设水面对太阳辐射的平均反射率为 7%,而且将吸收到的 35%能量重新辐射出去。太阳辐射 可将水面的水蒸发(设在常温、常压下蒸发 1kg 水需要 2.2×106J 的能量),而后凝结成雨 滴降落到地面。 A.估算整个地球表面的年平均降雨量(以毫米表示,球面积为 4πR2)。 B.太阳辐射到地球的能量中只有约 50%到达地面,W 只是其中的一部分。太阳辐射到地球 的能量没能全部到达地面,这是为什么?请说明二个理由。 20.(10 分)如图 A 所示,一质量为 m 的物体系于长度分别为 l1、l2 的两根细线上,l1 的一 端悬挂在天花板上,与竖直方向夹角为θ,l2 水平拉直,物体处于平衡状态。现将 l2 线剪断,求剪断瞬时物体的加速度。 (l)下面是某同学对该题的一种解法: 解:设 l1 线上拉力为 T1,线上拉力为 T2,重力为 mg,物体在三力作用下保持平衡 T1cosθ=mg, T1sinθ=T2, T2=mgtgθ 剪断线的瞬间,T2 突然消失,物体即在 T2 反方向获得加速度。因为 mg tgθ=ma,所以加速 度 a=g tgθ,方向在 T2 反方向。 你认为这个结果正确吗?请对该解法作出评价并说明理由。 (2)若将图 A 中的细线 l1 改为长度相同、质量不计的轻弹簧,如图 B 所示,其他条件不变, 求解的步骤和结果与(l)完全相同,即 a=g tgθ,你认为这个结 果正确吗?请说明理由。 21.(12 分)如图所示,一定量气体放在体积为 V0 的容器中,室温为 T0=300K 有一光滑导 热活塞 C(不占体积)将容器分成 A、B 两室,B 室 的体积是 A 室的两倍,A 室容器上连接有一 U 形管(U 形管内气体的体积忽略不计),两边水银柱高度差为 76cm,右室容器中连接有一阀门 K,可与大气相通。 (外界大气压等于 76cm 汞柱)求: (1)将阀门 K 打开后,A 室的体积变成多少? (2)打开阀门 K 后将容器内的气体从 300 K 分别加热到 400 K 和 540 K,U 形管内两边水银 面的高度差各为多少? 22.(3 分)半径为 a 的圆形区域内有均匀磁场,磁感强度为 B=0.2T,磁场方向垂直纸面 向里,半径为 b 的金属圆环与磁场同心地放置,磁场与 环面垂直,其中 a=0.4m,b=0.6m,金属环上分别接有 灯 L1、L2,两灯的电阻均为 R0=2Ω,一金属棒 MN 与金属 环接触良好,棒与环的电阻均忽略不计 (1)若棒以 v0=5m/s 的速率在环上向右匀速滑动,求棒滑过圆环直径 OO’的瞬时(如图所 示)MN 中的电动势和流过灯 L1 的电流。 (2)撤去中间的金属棒 MN 将右面的半圆环 OL2O’以 OO’为轴向上翻转 90o,若此时磁场随 时间均匀变化,其变化率为ΔB/Δt=(4 /3.14)T/s,求 L1 的功率。 无字母 23.(18 分)如图所示,光滑斜面的底端 a 与一块质量均匀、水平放置的平极光滑相接, 平板长为 2L,L=1m,其中心 C 固定在高为 R 的竖直支架上,R=1m,支架的下端与垂 直于纸面的固定转轴 O 连接,因此平板可 绕转轴 O 沿顺时针方向翻转.问: (l)在外面上离平板高度为 h0 处放置一滑块 A, 使其由静止滑下,滑块与平板间的动摩擦因数μ =0.2,为使平板不翻转,h0 最大为多少? (2)如果斜面上的滑块离平板的高度为 h1=0.45 m,并在 h1 处先后由静止释放两块质量相 同的滑块 A、B,时间间隔为Δt=0.2s,则 B 滑块滑上平板后多少时间,平板恰好翻转。(重 力加速度 g 取 10 m/s2) 2001 年全国普通高等学校招生统一考试(上海卷) 物理参考答案 说明: (1)定出评分标准是为了尽可能在统一的标准下评定成绩。试题的参考答案是用来说明评 分标准的,考生按其他方法或步骤解答,正确的,同样得分;有错的,根据错误的性质,参 照评分标准中相应的规定评分。 (2)第一、二、三题只要求写出答案,不要求写出演算过程。 (3)第 19、20、21、22、23 题只有最后答案而无演算过程的,不给分。解答中单纯列出与 解答无关的文字公式,或虽列出公式,但文字符号与题中所给定的不同,不给分。 (4)需作数字计算的问题,对答案的有效数字不作严格要求。一般按试题要求或按试题情 况取二位或三位有效数字即可。 -.选择题 1.CD 2.ACD 3.B 4.AD 5.BC 6.BC 7.A 8.CD 评分标准:全题 40 分,每小题 5 分,全选对的得 5 分,选对但不全的得 2 分,有选错或全 部不选的得 0 分。 二.填空题 9.伽利略,牛顿,爱因斯坦。 伽利略:望远镜的早期发明,将实验方法引进物理学等; 牛顿:发现运动定律,万有引力定律等; 爱因斯坦:光电效应,相对论等。 10.衍射,小于 11. 12.BDCA,DBCA 13.17,17.9 评分标准:全题 20 分,每小题 4 分。第 9 小题排序正确得 2 分,分别写出两位科学家的贡 献各得 1 分。第 10、 11、12、13 小题每空格 2 分。 三.实验题 14.A、D 15.A、B 16.应选用量程为 3V 的电压表. 改变电阻箱阻值 R,读取若干个 U 的值,由 I=U/R 计算出电流的值,然后作出 U-I 图线, 得到ε、r。 17.(l)如图所示, (2)aAD=5m/s2, vax=1.39m/s 18.(1)设计的电路图如图所示。 (2)测量步骤与结果: ①调节滑动变阻器,使转换器的输出电压为零; ②将砝码放在转换器上,记下输出电压 U0; ③将待测物放在转换器上,记下输出电压 U1; 由 U0=km0g,得 k=U0/m0g 测得 U=kmg,所以 m=m0U/U0。 (3)①因电源电压不够而输出电压调不到零; ②待测物体质量超出转换器量程。 评分标准:全题 30 分。14、15 小题每小题 5 分,全选对得 5 分,选对但不全得 2 分,有选 错的得 0 分。 16 小题第 1 空格得 2 分,第 2 空相得 4 分。17 小题(1)正确画出 v-t 图,得 3 分,后面 四点(H 用 K)连成直线不扣分;(2)每空格各得 2 分。18 小题(1)正确设计出电路图得 2 分;(2)正确写出测量步骤得 2 分,只写出部分步骤得 1 分;写出两个测量方程并得出 结果得 2 分;(3)提出与本题有关的问题得 1 分. 四.计算题 19.解:(1)2πR×1/4=1.00×107 R=6.37×106 m ① (2)A.设太阳在一年中辐射到地球水面部分的总能量为 W,W=1.87×1024J 凝结成雨滴年降落到地面水的总质量为 m m=W×0.93×0.65/(2.2×106)=5.14×1017 kg ② 使地球表面覆盖一层水的厚度为 h h=m/ρs 地球 h=1.01×103mm ③ 整个地球表面年平均降雨量约为 1.0×103 mm B.大气层的吸收,大气层的散射或反射,云层遮挡等。 评分标准:全题 10 分。第(1)小题 3 分,第(2)小题 7 分。其中(1)得出①给 3 分, 写出 R=6.4×106 m,同样给分。 (2)A.得出②给 2 分,得出③给 2 分。 B.写出 1 个原因,得 1 分;2 个或 2 个以上正确的原因,得 3 分;如果写出其它合理的原 因,也同样给分。 20.解:(1)错。 因为 I2 被剪断的瞬间,l1 上的张力大小发生了变化。 (2)对。 因为 G 被剪断的瞬间,弹簧 U 的长度末及发生变化,乃大小和方向都不变。 评分标准:全题 10 分。第(1)小题 6 分,第(2)小题 4 分。其中 (1)结论正确,得 3 分;评价和说明理由正确,得 3 分。 (2)结论正确,得 2 分;评价和说明理由正确,得 2 分。 21.解:(1)开始时,PA0=2 大气压,VA0=V0/3 打开阀门,A 室气体等温变化,pA=l 大气压,体积 VA pA0VA0=pAVA ① ② (2)从 T0=300K 升到 T,体积为 V0,压强为 PA,等压过程 ③ T1=400K<450K,pA1=pA=p0,水银柱的高度差为 0 从 T=450K 升高到 T2=540K 等容过程, ④ =1.2 大气压 ⑤ T2=540K 时,水银高度差为 15.2cm 评分标准:全题 12 分.第(1)小题 4 分,第(2)小题 8 分.其中 (1)得出①、②各得 2 分。 (2)得出③式,得 3 分;结果正确,得三分。 得出④、⑤式,各得 1 分;结果正确,得 2 分。 22 解:(1)ε1=B2av=0.2×0.8×5=0.8V ① I1=ε1/R=0.8/2=0.4A ② (2)ε2=ΔФ/Δt=0.5×πa2×ΔB/Δt=0.32V ③ P1=(ε2/2)2/R=1.28×102W ④ 1,28 *10-2 评分标准:全题 13 分.第(1 小题 6 分,第(2)小题 7 分。其中 (1)正确得出①式得 3 分,得出②式得 3 分; (2)得出③式 4 分,得出④式得 3 分。 23.解:(1)设 A 滑到 a 处的速度为 v0= ① f=uN,N=mg,f=ma, a=ug ② 滑到板上离 a 点的最大距离为 v0 2=2ugs0, s0=2gh0/2ug=h0/u ③ A 在板上不翻转应满足条件:摩擦力矩小于正压力力矩,即 M 摩擦≤M 压力 umgR≤mg(L-s0) ④ h0≤u(L-Ur)=0.2(1-0.2)=0.16 m ⑤ (2)当 h=0.45m,vA= = =3m/s vA=vB=3m/s ⑥ 设 B 在平板上运动直到平板翻转的时刻为 t,取Δt=0.2s sA=vA(t+Δt)-ug(t+Δt)2/2 ⑦’ sB=vBt-ugt2/2 ⑦ 两物体在平板上恰好保持平板不翻转的条件是 2umgR=mg(L-sA)+mg(L-sB) ⑧ 由⑦+⑦’式等于⑧式,得 t=0.2s 评分标准:全题 15 分。第(1)小题 7 分,第(2)小题 8 分。其中 (1)得出①、②、③各得 1 分,判断 M 摩擦≤M 压力正确得 2 分,④、⑤式各得 1 分。 (2)得出⑤式得 1 分,①式得 1 分,写出③式得 3 分,最后结果正确得 3 分。 2002 年普通高等学校招生全国统一考试(全国卷) 理科综合能力测试 本试卷分第Ⅰ卷(选择题)和第Ⅱ卷(非选择题)两部分,第Ⅰ卷 1 至 5 页,第Ⅱ卷 6 至 15 页,满分 300 分。考试用时 150 分钟。 第Ⅰ卷 (选择题 共 20 题,每题 6 分,共 120 分) 注意事项: 1.答第Ⅰ卷前,考生务必将自己的姓名、准考证号、考试科目涂写在答题卡上。考试结 束, 将试题卷和答题卡一并交回。 2.每小题选出答案后,用铅笔把答题卡上对应题目的答案标号涂黑,如需改动,用橡皮 擦 干净后,再选涂其它答案标号,不能答在试题卷上。 在下列各题的四个选项中,只有一个选项是符合题目要求的。 以下数据可供解题时参考: 原子量:H 1 C 12 N 14 O 16 Na 23 P 31 Cl 35.5 Ca 40 Fe 56 Cu 64 1.下列各类人群中,一段时期内人体摄人和排出的氮量基本相等的是 A.健康儿童 B.重创伤恢复期病人 C.健康成年男子 D,禁食用病人 2.下列关于细胞周期的叙述,正确的是 A.成熟的生殖细胞产生后立即进人下一个细胞周期 B.机体内所有的体细胞处于细胞周期中 C.抑制 DNA 的合成,细胞将停留在分裂期 D.细胞分裂间期为细胞分裂期提供物质基础 3.番镇种子播种在苗床上,在适宜的条件下,第 6 天子叶展开,第 9 天幼叶出现。研究人员 从种于到幼苗形成期间每天测定其于重,并绘制成曲线。下面四个曲线四中,正确的是 0 3 6 9 天数 0 3 6 9 天数 0 3 6 9 天数 0 3 6 9 天数 4.一只羊的卵细胞核被另一只羊的体细胞核置换后,这个卵细胞经过多次分裂,再植入第三 只羊的子宫内发育,结果产下一只羊羔。这种克隆技术具有多种用途,但是不能 A.有选择地繁殖某一性别的家畜 B.繁殖家畜中的优秀个体 C.用于保存物种 D.改变动物的基因型 5.自然界中生物种内及种间是相互作用、相互影响的。下述观点不正确的是 A.林鸽群较大时被在鹰捕食的几率降低一 B.炉鱼有时捕食护鱼的幼鱼,这有利于炉鱼种的维持 C,自然界中的猴群经过争斗建立了优劣等级制度,并依次占据资源,铂 J 出冷的保持是 有利的 D.自然界中物种间捕食对一个种有利,但会使另一个种消失 6.以下说法正确的是 A.纳米材料是指一种称为“纳米”的新物质制成的材料 B.绿色食品是指不合任何化学物质的食品 C.生物固氮是指植物通过叶面宜接吸收空气中的氮气 D.光导纤维是以 M 氧化硅为主要原料制成的 7. 0.01lmol·L 的某一元弱酸溶液 pH=4,则它的电离度为 A.l% B.2% C.5% D.10% 8.某温度下,100g 饱和氯化钠溶液中含有氯化钠 26.5g.若向此溶液中添加 3.5g 氯化钠 和 6.5gA 水,则所得溶液的科质质量分数是 A. 30% B. %1005.6100 5.35.26   C. 26.5 % D. %1005.35.6100 5.35.26   9.有人曾建议用 AG 表示溶液的酸度(acidity grade)AG 的定义为 AG=lg([H+]/[OH-])。 下列表述正确的是 A.在 25℃时,若溶液呈中性,则 pH=7,AG=1 B.在 25℃时,若溶液呈性,则 Ph<7,AG<0 C.在 25℃时,若溶液呈中性,则 pH>7,AG>0 D.在 25℃时,若溶液呈中性,则 AG=2(7-pH) 10.常温下,将甲酸和氢氧化钠溶液混合,所得溶液 pH=7,则此溶液中 A. [HCOO—]>[Na+] B. [HCOO—]<[Na+] C. [HCOO—]=[Na+] D. 无法确定[HCOO—]和[Na+]的关系 11.两种元素原子的核外电子层数之比与最外层电子数之比相等,则在周期表的前 10 号元素 中,满足上述关系的元素共有 A. 1 对 B. 2 对 C. 3 对 D. 4 对 12.用足量的 co 还原 32.0g 某种氧化物,将生成的气体通入足量澄清石灰水中,得到 60g 沉 淀,则该氧化物是 A. FeO B. Fe2O3 C. CuO D. Cu2O 13.化合价为 n 的某种元素的硝酸盐的式量为 x ,其氢氧化物的式量为 y ,则 n 的值是 A. 45 yx  B. 45 xy  C. 79 yx  D. 79 xy  14. L 多巴是一种有机物,它可用于帕金森综合症的治疗,其结构简式如下: OH CHCOOHCH 2 2NH HO 这种药物的研制是基于获得 2000 年诺贝尔生理学或医学奖和获得 2001 年诺贝尔化 学奖的研究成果。下列关于 L 多巴酸碱性的叙述正确的是 A. 既没有酸性,又没有碱性 B. 既具有酸性,又具有碱性 C. 只有酸性,没有碱性 D. 只有碱性,没有酸性 15.目前普通认为,质子和中子都是由被称为 u 夸克和 d 夸克的两类夸克组成。u 夸克带电 量为 e3 2 , d 夸克带电量为 e3 1 , e 为基元电荷。下列论断可能正确的是 A. 质子由 1 个u 夸克和 1 个 d 夸克组成,中子由 1 个u 夸克和 2 个 d 夸克组成 B. 质子由 2 个u 夸克和 1 个 d 夸克组成,中子由 1 个u 夸克和 2 个 d 夸克组成 C. 质子由 1 个u 夸克和 2 个 d 夸克组成,中子由 2 个u 夸克和 1 个 d 夸克组成 D. 质子由 2 个u 夸克和 1 个 d 夸克组成,中子由 1 个u 夸克和 1 个 d 夸克组成 16.在光滑水平地面上有两个相同的弹性小球 A、B,质量都为 m。现 B 球静止,A 球向 B 球运动,发生正碰。已知碰撞过程中总机械能守恒,两球压缩最紧时的弹性势能为 Ep,, 则碰前 A 球的速度等于 A. m E p B. m E p2 C. 2 m E p D. 2 m E p2 17.图中 EF、GH 为平行的金属导轨,其电阻可不计,R 为电阻器,G 为电容器,AB 为可在 EF 和 GH 上滑动的导体横杆。有均匀磁场垂直与导轨平面。若用 I1 和 I2 分别表示图中该 处导线中的电流,则当横杆 AB A. 匀速滑动时,I1=0,I2=0 B. 匀速滑动时,I1≠0,I2≠0 C. 加速滑动时,I1=0,I2=0 D. 加速滑动时,I1≠0,I2≠0 18.质点所受的力 F 随时间变化的规律如图所示,力的方向 始终在一直线上。已知 t=0 时质点的速度为零。在图示 t1、 t2、t3 和 t4 各时刻中,哪一时刻质点的动能最大? A.t1 B.t2 C.t3 D.t4 19.为了观察门外情况,有人在门上开一小圆孔,将一块圆柱形玻璃嵌入其中,圆柱体轴线 与门面垂直。从圆柱底面中心看出去,可以看到的门外人射光线与轴线间的最大夹角称 做视场角。已知该玻璃的折射率为 n ,圆柱长为l ,底面半径为 r, 则视场角是 A. 22 arcsin lr nl  B. 22 arcsin lr nr  C. 22 arcsin lrn r  D. 22 arcsin lrn l  20.在如图所示的电路中,R1、R2、R3 和 R4 皆为定值电阻,R5 为可变电阻,电源的电动势 为。,内阻为 r。设电流表 A 的读数为 I,电压表 V 的读数为 U.当 R5 的滑动触点向图中 a 端移动时, A.I 变大,U 变小 B.I 变大,U 变大 C.I 变小,U 变大 D.I 变小,U 变小 2002 年普通高等学校招生全国统一考试(全国卷) 理科综合能力测试 本试卷分第Ⅰ卷(选择题)和第Ⅱ卷(非选择题)两部分,第Ⅰ卷 1 至 5 页,第Ⅱ卷 6 至 15 页,满分 300 分。考试用时 150 分钟。 第Ⅱ卷 (非选择题 共 10 题 共 180 分) 注意事项: 1.用钢笔或圆珠笔直接答在试题卷中(除题Ⅱ有特殊规定外) 2.答卷前将密封线内的项目填写清楚。 题 号 21 22 23 24 25 26 27 28 29 30 总 分Ⅰ Ⅱ Ⅰ Ⅱ 分 数 21.(15 分)为验证“镁是植物生活的必需元素”,三位同学进行了实验设 计, 下列是实验的基本思路。请分别指出三个实验思路能否达到实验目 的,为 什么?再写出你的设计思路。 (1)实验一:取生长状况一致的大豆幼苗,用符合实验要求的容器进行培养。对照组容器 内只盛有蒸馏水,实验组盛有用蒸馏水配制的镁盐溶液。两组置于相同的适宜条件下 培养,并对溶液通气,观察比较两组植物的生长发育情况。 答: 得 分 得分 评卷人 28 29 Ⅰ Ⅱ Ⅰ Ⅱ (2)实验二:取生长状况一致的大豆幼苗,栽培在盛有砂性土壤的容器中(砂性上壤肥力 均匀,容器符合实验要求),对照组浇以蒸馏水,实验组浇以蒸馏水配制的镁盐溶液, 两组置于相同的适宜条件下培养,观察比较两组植物的生长发育情况。 答: 得 分 (3)实验三:取生长状况一致的大豆幼苗,栽培在盛有砂性土壤的容器中(破性土壤肥力 均匀,容器符合实验要求),对照组浇以含有植物必需的各种元素的完全营养液,实验 组浇以不含镁离子的完全营养液,两组置于相同的适宜条件下培养,观察比较两组植物 的生长发育情况。 答: 得 分 (4)下面由你进行实验设计,请写出你的设计思路。 答: 得 分 22.(9 分)科学家应用生物技术培育出了一种抗虫棉,它能产生毒素, 杀死 害虫,目前正在大面积推广种植。科学家还研究了害虫的邀传基础, 发 现不抗毒素对抗毒素为显性(此外分别用 B 和 b 表示)。据此 回答: (1)种植抗虫棉,有利于生态环境保护,这是因为_____________________________。 得 分 (2)棉田不抗霉渍害虫的基因型为___________;抗毒素害虫的基因型为________________。 得 分 得分 评卷人 (3)不抗毒案害虫与抗毒素害虫杂交,则子代的基因型为_____________________________。 得 分 23.(12 分)如图所示:淀粉水解可产生某有机化合物 A 人在不同的氧化 剂作用下,可以生成 B(C6H14O6)或 C(C6H10O8),B 和 C 都不能发 生银镜反 应。A、B 和 C 都可以被强还原剂还原成为 D(C6H14O6)。B 脱水可得 到五元环的酯类化合物 E 或六元环的酯类化合物 F。 已知,相关物质被氧化的难易次序是: RCHO 最易,R-CH2OH 次之, 最难。 请在下列空格中填写 A、B、C、D、E、F 的结构简式。 A:______________ B:______________ C:_______________ D:______________ E:______________ F:_______________ 得 分 24.(14 分)在 25℃,101KPa 条件下,将 15L O2 通人 10L CO 和 H2 的 混合气中,使其完全燃烧,于燥后,恢复至原来的温度和压强。 (1)若剩余气体的体积是 15 L,则原 CO 和 H2 的混合气中 V(CO)=_________L,V(H2)=___________ L. 得 分 (2)若剩余气体的体积为 a L,则原 CO 和 H2 的混合气中 V(CO):V(H2)=______。 得分 评卷人 得分 评卷人 得 分 (3) 若剩余气体的体积为 aL,则 a 的取值范围是_____________。 25.(12 分)已知: ①A、B、C、D 四种物质均含元素 X,有的还可能合有元素 Y、Z。元素 Y、 X、Z 的原子序数依次递增。 ②X 在 A、B、C、D 中都不呈现它的最高化合价。 ③室温下单质 A 与某种常见一元强碱溶液反应,可得到 B 和 C. ④化合物 D 受热催化分解,可制得元素 Y 的单质。 (1)元素 X 是___________________________,Z 是__________________________。 得 分 (2)写出③中反应的化学方程式:_______________________________________。 得 分 (3)写出④中反应的化学方程式:_______________________________________。 得 分 26.(20 分)蹦床是运动员在一张绷紧的弹性网上蹦跳、翻滚并做各种 空中动 作的运动项目。一个质量为 60 kg 的运动员,从离水平网面 3.2m 高 处自 由下落,着网后沿竖直方向蹦回到离水平网面 5.0m 高处。已知运动 员与 网接触的时间为 1.2s。若把在这段时间内网对运动员的作用力当作恒力处理,求此力的大 小。(g=10m/s2) 得分 评卷人 得分 评卷人 得 分 27.(20 分)电视机的显像管中,电子束的们转是用磁偏转技术实现的。 电子 束经过电压为 U 的加速电场后,进人一圆形匀强磁场区,如图所示。 磁 场方向垂直于圆面。磁场区的中心为 O,半径为 r。当不加磁场时, 电子束将通过 O 点而打到屏幕的中心 M 点。为了让电子束射到屏幕边缘 P,需要加进场, 使电于束们转一己知角度 ,此时磁场的磁感应强度 B 应为多少? 得 分 28.(14 分)昆虫能分泌信息素。下列是一种信息素的结构简式; 得分 评卷人 CH3(CH2)5CH=CH(CH2)9CHO Ⅰ.(8 分)指出该物质中的任意..一种官能团,为该官能团提供一种简单 的鉴 别方法并简述实验现象,写出与鉴别方法有关反应的化学方程式并 指出 反应类型,完成下表。 官能团 鉴别方法及 实验现象 有关反应的化学方程式 反应类别 得 分 Ⅱ.(6 分)昆虫的性外激素属于信息素,根据其化学结构可以合成性引诱剂,性引诱剂可以 用于 防治害虫,请写出如何将性引诱剂用于害虫的防治。 ①__________________________________________________________________。 得 分 ②__________________________________________________________________。 29.(37 分)大气压强对许多物理实验和化学实验有着重要影响。 得分 评卷人 Ⅰ.(17 分)现用“验证玻意耳定律”的仪 器来测量大气压强 p0。注射器针筒已被 固定在竖直方向上,针筒上所标刻度是 注射器的容积,量大刻度 Vm=10ml。注射器活塞已装上钩码框架,如图所示。此外,还有 一架托盘天平、若干钩码、一把米尺、一个针孔橡皮冒和少许润滑油。 (1)下面是实验步骤,试填写所缺的②和⑤。 ①用米尺测出注射器针筒上全部刻度的长度 L。 ②_____________________________________________________________。 ③把适量的润滑油抹在注射器的活塞上,将活塞插入针筒中,上下拉动活塞,使活 塞与 针筒的间隙内均匀地涂上润滑油。 ④将活塞插到适当的位置。 ⑤_____________________________________________________________。 ⑥在钩码框架两侧挂上钩码,记下挂上的钩码质量 m1。小达到平衡后,记下注射 器中空气柱的体积 V1。在这个过程中不要用手接触注射器以保证空气柱温度不 变。 ⑦增加钩码的个数,使钩码的质量增大为 m2,达到平衡后,记下空气柱的体积 V2。 得 分 (2)求出计算大气压强 p0 的公式。(用已给的和测得的物理量表示) 得 分 Ⅱ.(20 分)制取氨气并完成喷泉实验(图中夹持装置均已略去)。 得分 评卷人 得分 评卷人 (1)写出实验室制取氨气的化学方程式: ____________________________________________________________________。 得 分 ( 2 ) 收 集 氨 气 应 使 用 ______________- 法 , 要 得 到 干 燥 的 氮 气 可 选 用 _______________________做干燥剂。 得 分 (3)用图 1 装置进行喷泉实验,上部烧瓶已装满干燥氨气,引发水上喷泉的操作是 _______________________________________________________________________________ __ 。 该 实 验 的 原 理 是 ____________________________________________________________________ _______________________________________________________________________________ _______________________________________________________________________________ _______。 得 分 (4)如果只提供如图 2 的装置,请说明引发喷泉的方法。 答: 30.(27 分)有三根长度皆为 l=1.00m 的不可伸长的绝缘轻线,其中两根 的一 端固定在天花板上的 O 点,另一端分别拴有质量皆为 m=1.00× 得分 评卷人 10-2kg 的 带电小球 A 和 B,它们的电量分别为-q 和+q,q=1.00×10-7C。A、 B 之间用第三根线连接起来。空间中存在大小为 E=1.00×106N/C 的匀强电场,场强方向沿 水平方向右,平衡时 A、B 球的位置如图所示。现将 O、B 之间的线烧断,由于有空气阻力, A、B 球最后会达到新的平衡位置。求最后两球的机械能与电势能总和与烧断前相比改变了 多少。(不计两带电小球间相互作用的静电力) 得 分 2002 年全国普通高等学校招生全国统一考试 物 理 (广东、广西、河南用) 本试卷分第Ⅰ卷(选择题)和第Ⅱ卷(非选择题)两部分,满分 150 分,考试时间 120 分钟. 第Ⅰ卷(选择题 共 40 分) 一、本题共 10 小题,每小题 4 分,共 40 分.在每小题给出的四个选项中,有的小题只有一 个选项正确,有的小题有多个选项正确.全部选对的得 4 分,选不全的得 2 分,有选错 或不答的得 0 分. 1.下面说法中正确的是 A.光子射到金属表面时,可能有电子发出 B.光子射到金属表面时,一定有电子发出 C.电子轰击金属表面时,可能有光子发出 D.电子轰击金属表面时,一定没有光子发出 2.图中a、b、c为三个物块,M、N为两个轻质弹簧,R 为跨过光滑定滑轮的轻绳,它 们连接如图所示并处于平衡状态. A.有可能N处于拉伸状态而M处于压缩状态 B.有可能N处于压缩状态而M处于拉伸状态 C.有可能N处于不伸不缩状态而M处于拉伸状态 D.有可能N处于拉伸状态而M处于不伸不缩状态 3.处于基态的一群氢原子受某种单色光的照射时,只发射波长为λ1、λ2、λ3的三种 单色光,且λ1>λ2>λ3,则照射光的波长为 A.λ1 B.λ1+λ2+λ3 C.λ2λ3/λ2+λ3 D.λ1λ2/λ1+λ2 4.如图所示,在原来不带电的金属细杆ab附近P处,放置一个正点电荷.达到静电平 衡后, A.a端的电势比b端的高 B.b端的电势比d点的低 C.a端的电势不一定比d点的低 D.杆内c处场强的方向由a指向b 5.分子间同时存在吸引力和排斥力,下列说法中正确的是 A.固体分子间的吸引力总是大于排斥力 B.气体能充满任何容器是因为分子间的排斥力大于吸引力 C.分子间的吸引力和排斥力都随分子间距离的增大而减小 D.分子间吸引力随分子间距离的增大而增大,而排斥力随距离的增大而减小 6.跨过定滑轮的绳的一端挂一吊板,另一端被吊板上的人拉住,如图所示.已知人的质 量为 70kg,吊板的质量为 10kg,绳及定滑轮的质量、滑轮的摩擦 均可不计.取重力加速度g=10m/s2.当人以 440N的力拉绳时, 人与吊板的加速度a和人对吊板的压力F分别为 A.a=1.0m/s2,F=260N B.a=1.0m/s2,F=330N C.a=3.0m/s2,F=110N D.a=3.0m/s2,F=50N 7.竖直上抛一球,球又落回原处,已知空气阻力的大小正比于球的速度. A.上升过程中克服重力做的功大于下降过程中重力做的功 B.上升过程中克服重力做的功等于下降过程中重力做的功 C.上升过程中克服重力做功的平均功率大于下降过程中重力的平均功率 D.上升过程中克服重力做功的平均功率等于下降过程中重力的平均功率 8.在图中虚线所示的区域存在匀强电场和匀强磁场.取坐标如图所示,一带电粒子沿x 轴正方向进入此区域,在穿过此区域的过程中运动方向始终不发 生偏转.不计重力的影响,电场强度E和磁感应强度B的方向可 能是 A.E和B都沿x轴方向 B.E沿y轴正向,B沿z轴正向 C.E沿z轴正向,B沿y轴正向 D.E、B都沿z轴方向 9.远距离输电线路的示意图如下图所 示,若发电机的输出电压不变,则下列 叙述中正确 的是 A.升压变压器的原线圈中的电流与用户用电设备消耗的功率无关 B.输电线中的电流只由升压变压器原副线圈的匝数比决定 C.当用户用电器的总电阻减少时,输电线上损失的功率增大 D.升压变压器的输出电压等于降压变压器的输入电压 10.一列在竖直方向振动的简谐横波,波长为λ,沿正x方向传播,某一时刻,在振动 位移向上且大小等于振幅一半的各点中,任取相邻的两点P1、P2,已知P1的x坐标 小于P2的x坐标. A.若 <λ/2,则P1向下运动,P2向上运动 B.若 <λ/2,则P1向上运动,P2向下运动 C.若 >λ/2,则P1向上运动,P2向下运动 D.若 >λ/2,则P1向下运动,P2向上运动 第Ⅱ卷(非选择题 共 110 分) 二、本题共 3 小题,共 20 分,把答案填在题中的横线上或按题目要求作图. 11.(5 分)用螺旋测微器(千分尺)测小球直径时,示数如图所 示,这时读出的数值是_________,单位是_________. 12.(8 分)现有器材:量程为 10.0mA、内阻约 30Ω~40Ω的 电流表一个,定值电阻 R1 =150Ω,定值电阻R2=100Ω,单刀单掷开关S,导线若干,要求利用这些器材 测量一干电池(电动势约 1.5V)的电动势. (1)按要求在实物图上连线. (2)用已知量和直接测得量表示的待测电动势的表达式为E=_________,式中各直 接测得量的意义是____________________________________. 13.(7 分)一个有一定厚度的圆盘,可以绕通过中心垂直于盘面的水平轴转动,用下面 的方法测量它匀速转动时的角度速. 实验器材:电磁打点计时器,米尺,纸带,复写纸片. 实验步骤: (1)如图所示,将电磁打点计时器固定在桌面上,将纸带的一端穿过打点计时器的限位 孔后,固定在待测圆盘的侧面上,使得圆盘转动时,纸带可以卷在圆盘侧面上. (2)启动控制装置使圆盘转动,同时接通电源,打点计时器开始打点. (3)经过一段时间,停止转动和打点,取下纸带,进行测量. ①由已知量和测得量表示的角速度的表达式为ω=_________,式中各量的意义是: ______________________________________________________. ②某次实验测得圆盘半径r=5.50×10-2m,得到的纸带的一段如下图所示,求得角速 度为_________. 三、本题共 7 小题,共 90 分.解答应写出必要的文字说明、方程式和重要演算步骤.只写 出最后答案的不能得分.有数值计算的题,答案中必须明确写出数值和单位. 14.(11 分)有人利用安装在气球载人舱内的单摆来确定气球的高度.已知该单摆在海平面 处的周期是T0.当气球停在某一高度时,测得该单摆周期为T.求该气球此时离海平 面的高度h.把地球看作质量均匀分布的半径为R的球体. 15.(12 分)如图所示,半径为R、单位长度电阻为λ的均匀导体圆环固定在水平面上,圆 环中心为O.匀强磁场垂直水平面方向向下,磁感强度为B.平行 于直径MON的导体杆,沿垂直于杆的方向向右运动.杆的电阻可 以忽略不计,杆与圆环接触良好,某时刻,杆的位置如图,∠aO b=2θ,速度为v,求此时刻作用在杆上安培力的大小. 16.(12 分)如下一系列核反应是在恒星内部发生的, 其中p为质子,α为α粒子,e+为正电子,ν为一种中微子.已知质子的质量为mp =1.672648×10-27kg,α粒子的质量为mα=6.644929×10-27kg,正电子的质量为me =9.11×10-31kg,中微子的质量可忽略不计,真空中的光速c=3.00×108m/s.试 计算该系列核反应完成后释放的能量. 17.(13 分)雨过天晴,人们常看到天空中出现彩虹,它是由阳光照射到空中弥漫的水珠 上时出现的现象.在说明这个现象时,需要分析光线射入水珠后的光路.一细束光线射 入水珠,水珠可视为一个半径为 R 的球,球心O到入射光线的垂直距离为d.水的折射 率为n. (1)在图上画出该束光线射入水珠内经一次反射后又从水珠中射出的光路图. (2)求这束光线从射向水珠到射出水珠每一次偏转的角度. 18.(13 分)现有m=0.90kg的硝酸甘油[C3H5(NO3)3]被密封于体积V0=4.0 ×10-3m3的容器中,在某一时刻被引爆,瞬间发生激烈的化学反应,反应的产物全是 氮、氧……等气体.假设:反应中每消耗 1kg的硝酸甘油释放能量U=6.00×106J /kg;反应产生的全部混合气体温度升高 1K所需能量Q=1.00×103J/K;这些 混合气体满足理想气体状态方程pV/T=C(常量),其中常量C=240J/K.已知 在反应前硝酸甘油的温度T0=300K.若设想在化学反应发生后容器尚未破裂,且反应 释放的能量全部用于升高气体的温度,求器壁所受的压强. 19.(14 分)下面是一个物理演示实验,它显示:图中自由下落的物体A和B经反弹后,B 能上升到比初始位置高得多的地方.A是某种材料做成的实心球,质量m1=0.28kg, 在其顶部的凹坑中插着质量m2=0.10kg的木棍B.B只是松松地插在凹坑中,其下 端与坑底之间有小空隙.将此装置从A下端离地板的高度H=1.25m处由静止释放.实 验中,A触地后在极短时间内反弹,且其速度大小不变;接着木棍B脱离球A开始上升, 而球A恰好停留在地板上.求木棍B上升的高度,重力加速度g=10m/s2. 20.(15 分)如图(a)所示,A、B为水平放置的平行金属板,板间距离为d(d远小于 板的长和宽).在两板之间有一带负电的质点P.已知若在A、B间加电压Uo,则质点 P可以静止平衡.现在A、B间加上如图(b)所示的随时间t变化的电压u.在t= 0 时质点P位于A、B间的中点处且初速为零.已知质点P能在A、B之间以最大的幅 度上下运动而又不与两板相碰,求图(b)中u改变的各时刻t1、t2、t3及tn的表 达式.(质点开始从中点上升到最高点,及以后每次从最高点到最低点或从最低点到最 高点的过程中,电压只改变一次.) 2002 年全国普通高等学校招生统一考试 物理参考答案 (广东、广西、河南卷) 一、 1.AC 2.AD 3.D 4.B 5.C 6.B 7.BC 8.AB 9.C 10.AC 二、 11.8.473mm 12.(1)如图所示;(2)(I1I2/(I1-I2))R2I1是外电阻为 R1 时的电流,I2是外 电阻为R1和R2串联时的电流 13.(1)(x2-x1)/T(n-1)rT为电磁打点计时器打点的时间间隔,r为圆盘的半 径,x2、x1是纸带上选定的两点分别对应的米尺上的刻度值,n为选定的两点间的打点数 (含两点) (2)6.8r/s 三、14.根据单摆周期公式,有 T0=2π ,T=2π . 其中l是单摆长度,g0和g分别是两地点的重力加速度.根据万有引力公式,得 g0=GM/R2,g=GM/(R+h)2. 其中G是引力常量,M是地球质量.由以上各式解得 h=((T/T0)-1)R. 15.如图所示,杆切割磁力线时,ab部分产生的感应电动势 E=vB(2Rsinθ), 此时弧acb和弧adb的电阻分别为 2λR(π-θ)和 2λRθ,它们并联后的电阻 为 r=2λRθ(π-θ)/π, 杆中的电流为I=E/r, 作用在杆上的安培力为F=IB(2Rsinθ), 由以上各式解得 F=(2πvB2R/λ)(sin2θ/θ(π-θ)). 16.为求出系列反应后释放的能量,将题中所给的诸核反应方程左右两侧分别相加,消去 两侧相同的项,系列反应最终等效为 4p——→α+2e++2ν. 设反应后释放的能量为Q,根据质能关系和能量守恒得 4mpc2=mαc2+2mec2+Q, 代入数值可得 Q=3.95×10-12J. 17.(1)光路如图(a)所示. (2)以i、r表示入射光线的入射角、折射角,由折射定律有 sini=nsinr, 以δ1、δ2、δ3表示每一次偏转的角度,如图(b)所示,由反射定律、折射定律和几 何关系可知 sini=d/R, δ1=i-r,δ2=π-2r,δ3=i-r. 由以上各式解得 δ1=(sin-1d/R)-(sin-1d/nR). δ2=π-2sin-1d/nR. δ3=(sin-1d/R)-(sin-1d/nR). 18.化学反应完成后,硝酸甘油释放的总能量为 W=mU, 设反应后气体的温度为T,根据题意,有 W=Q(T-T0), 器壁所受的压强为 p=CT/V0, 联立以上各式并代入数据,得 p=3.4×108PA. 19.根据题意,A碰地板后,反弹速度的大小v1等于它下落到地面时速度的大小,即 v1= , A刚反弹后,速度向上,立刻与下落的B碰撞,碰前B的速度 v2= . 由题意,碰后A速度为零,以v2′表示B上升的速度,根据动量守恒,有 m1v1-m2v2=m2v2′, 令h表示B上升的高度,有 h=v2′2/2g. 由以上各式并代入数据,得 h=4.05m. 20.设质点P的质量为m,电量大小为q,根据题意,当A、B间的电压为U0时,有 qU0/d=mg, 当两板间的电压为 2U0时,P的加速度向上,其大小为a,则 (q2U0/d)-mg=ma, 解得a=g. 当两板间的电压为零时,P自由下落,加速度为g,方向向下. 在t=0 时,两板间的电压为 2U0,P自A、B间的中点向上做初速度为零的匀加速运动, 加速度为g.设经过时间τ1,P的速度变为v1,此时使电压变为零,让P在重力作用下做 匀减速运动,再经过时间τ1′,P正好到达A板且速度为零,故有 v1=gτ1,0=v1-gτ1′, (1/2)d=(1/2)gτ1 2+v1τ1′-(1/2)gτ1′2, 由以上各式,得 τ1=τ1′,τ1=( /2) , 因为t1=τ1,得t1=( /2) . 在重力作用下,P由A板处向下做匀加速运动,经过时间τ2,速度变为v2,方向向下, 这时加上电压使P做匀减速运动,经过时间τ2′,P到达B板且速度为零,故有 v2=gτ2,0=v2-gτ2′, d=(1/2)gτ2 2+v2τ2′-(1/2)gτ2′2, 由以上各式,得τ2=τ2′,τ2= , 因为t2=t1+τ1′+τ2, 得t2=( +1) . 在电场力与重力的合力作用下,P由B板处向上做匀加速运动,经过时间τ3,速度变为 v3,此时使电压变为零,让P在重力作用下做匀减速运动.经过时间τ3′,P正好到达A 板且速度为零,故有 v3=gτ3,0=v3-gτ3′, d=(1/2)gτ3 2+v3τ3′-(1/2)gτ3′2 由上得τ3=τ3′,τ3= , 因为t3=t2+τ2′+τ3, 得t3=( +3) . 根据上面分析,因重力作用,P由A板向下做匀加速运动,经过时间τ2,再加上电压, 经过时间τ2′,P到达B且速度为零,因为t4=t3+τ3′+τ2, 得t4=( +5) . 同样分析可得 tn=( +2n-3) .(n≥2) 2002 年全国普通高等学校招生全国统一考试 物理(上海卷) 考生注意: 1. 答卷前,考生务必将姓名、准考证号、校验码等填写清楚。 2. 本试卷共 8 页,满分 150 分。考试时间 120 分钟。考生应用钢笔或圆珠等将答案直接写 在试卷上。 3. 第 19、20、21、22、23 题要求写出必要的文字说明、方程式和重要的演算步骤。只写 出最后答案,而未写出主要演算过程的,不能得分。有数字计算的问题,答案中必须明 确写出数值和单位。 一.(40 分)选择题。本大题共 8 小题,每小题 5 分。每小题给出的四个答案中,至少有一 个是正确的。把正确答案全选出来,并将正确答案前面的字母填写在题后的方括号内。 每一小题全选对的得 5 分;选对但不全得部分分;有选错或不答的得 0 分。填写在方括 号外的字母,不作为选出的答案。 1.图中 P 为放在匀强电场中的天然放射源,其放出的射线 在电 场的作用下分成 a、b、c 三束,以下判断正确的是 A.a 为α射线、b 为β射线 B.a 为β射线、b 为γ射线 C.b 为γ射线、C 为α射线 D.b 为α射线、C 为γ射线 2.下列各图中,P 表示压强,V 表示体积,T 表示热力学温度,t 表示摄氏温度,各图中正 确描述一定质量理想气体等压变化规律的是 A. B. C. D. 3.在如图所示电路中,当变阻器局的滑动头 P 向 b 端移动时 A.电压表示数变大,电流表示数变小 B.电压表示数变小,电流表示数变大 C.电压表示数变大,电流表示数变大 D.电压表示数变小,电流表示数变小 4.如图所示,S1、S2 是振动情况完全相同的两个机械波波 源,振幅为 A,a、b、c 三点分别位于 S1、S2 连线的中垂线 上,且 ab=bc。某时刻 a 是两列波的波峰相遇点,c 是两列 波的波谷相遇点,则 A.a 处质点的位移始终为 2A B.c 处质点的位移始终为-2A C.b 处质点的振幅为 2A D.d 处质点的振幅为 2A 5.如图所示,A、B 为大小、形状均相同且内壁光滑,但用不同材料 制成的圆管,竖直固定在相同高度。两个相同的磁性小球,同时从 A、 B 管上端的管口无初速释放,穿过 A 管的小球比穿过 B 管的小球先落 到地面。下面对于两管的描述中可能正确的是 A.A 管是用塑料制成的,B 管是用铜制成的 B.A 管是用铝制成的,B 管是用胶木制成的 C.A 管是用胶木制成的,B 管是用塑料制成的 D.A 管是用胶木制成的,B 管是用铝制成的 6.如图所示,在粗糙水平面上固定一点电荷 Q,在 M 点无初速释放一带有恒定电量的小物 块,小物块在 Q 的电场中运动到 N 点静止,则从 M 点运动到 N 点的过程中 A.小物块所受电场力逐渐减小 B.小物块具有的电势能逐渐减小 C.M 点的电势一定高于 N 点的电势 D.小物块电势能变化量的大小一定等于克服摩擦力做的功 7.一航天探测器完成对月球的探测任务后,在离开月球的过程中,由静止开始沿着与月球 表面成一倾斜角的直线飞行,先加速运动,再匀速运动。探测器通过喷气而获得推动力。 以下关于喷气方向的描述中正确的是 A.探测器加速运动时,沿直线向后喷气 B.探测器加速运动时,竖直向下喷气 C.探测器匀速运动时,竖直向下喷气 D.探测器匀速运动时,不需要喷气 8.太阳从东边升起,西边落下,是地球上的自然现象,但在某些条件下,在纬度较高地区 上空飞行的飞机上,旅客可以看到太阳从西边升起的奇妙现象。这些条件是 A.时间必须是在清晨,飞机正在由东向西飞行,飞机的速度必须较大 B.时间必须是在清晨,飞机正在由西向东飞行,飞机的速度必须较大 C.时间必须是在傍晚,飞机正在由东向西飞行,飞机的速度必须较大 D.时间必须是在傍晚,飞机正在由西向东飞行,飞机的速度不能太大 二、(20 分)填空题。本大题共 5 小题,每小题 4 分。答案写在题中横线上的空白处,不要 求写出演算过程。 9.研究物理问题时,常常需要忽略某些次要因素,建立理想化的物理模型。例如“质点” 模型忽略了物体的体积、形状,只计其质量。请再写出两个你所学过的物理模型的名称: 和 模型。 10.完成核反应方程: 234 90 Th→ 234 91 Pa+ 。 Th234 90 衰变为 Pa234 91 的半衰期是 1.2 分钟,则 64 克 234 90 Th 经过 6 分钟还有 克尚未 衰变。 11.按照有关规定,工作场所受到的电磁辐射强度(单位时间内垂直通过单位面积的电磁辐 射能量)不得超过 0.50 瓦/米 2。若某一小型无线通讯装置的电磁辐射功率是 1 瓦,那 么在距离该通讯装置 以外是符合规定的安全区域(已知球面面积 为 S= 4πR2)。 12.在与 x 轴平行的匀强电场中,一带电量为 1.0×10-8 库仑、质量为 2.5×10-3 千克的物体 在光滑水平面上沿着 x 轴作直线运动,其位移与时间的关系是 x=0.16t-0.02t2,式中 x 以米为单位,t 以秒为单位。从开始运动到 5 秒末物体所经过的路程为 米,克服电场力所作的功为 焦耳。 13.磁场具有能量,磁场中单位体积所具有的能 量叫做能量密度,其值为 B2/2μ,式中 B 是磁感 强度,μ是磁导率,在空气中μ为一已知常数。 为了近似测得条形磁铁磁极端面附近的磁感强度 B,一学生用一根端面面积为 A 的条形磁铁吸住一相同面积的铁片 P,再用力将铁片与磁铁 拉开一段微小距离Δl,并测出拉力 F,如图所示。因为 F 所作的功等于间隙中磁场的能量, 所以由此可得磁感强度 B 与 F、A 之间的关系为 B= 。 三、(30 分)实验题 14.(5 分)如图所示为一显示薄膜干涉现象的实验装置,P 是 附有肥皂膜的铁丝圈,S 是一点燃的酒精灯。往火焰上洒些盐后, 在肥皂膜上观察到的干涉图象应是下图中的 A B C D 15.(7 分)如图所示器材可用来研究电磁感应现象 及到定感应电流方向。 (1)在给出的实物图中,用实线作为导线将实 验仪器连成实验电路。 (2)将线圈 L1 插入 L2 中,会上开关。能使感 应电流与原电流的绕行方向相同的实验操作是 A.插入软铁棒。 B.拔出线圈 L1。 C.使变阻器阻值变大。 D.断开开关。 16.(6 分)如图所示为一实验小车中利用光脉冲测量车 速和行程的装置的示意图,A 为光源,B 为光电接收器, A、B 均固定在车身上,C 为小车的车轮,D 为与 C 同 轴相连的齿轮。车轮转动时,A 发出的光束通过旋转齿 轮上齿的间隙后变成脉冲光信号,被 B 接收并转换成电信号,由电子电路记录和显示。若 实验显示单位时间内的脉冲数为 n,累计脉冲数为尼 N,则要测出小车的速度和行程还必须 测量的物理量或数据是 ;小车速度的表达式为 v= ;行 程的表达式为 s= 。 17.(8 分)有一组同学对温度计进行专题研究。他们通过查阅资料得知十七世纪时伽利略 曾设计过一个温度计,其结构为,一麦杆粗细的玻璃管,一端与一鸡蛋大小的玻璃泡相 连,另一端竖直插在水槽中,并使玻璃管内吸入一段水柱。根据管中水柱高度的变化可 测出相应的温度。为了研究“伽利略温度计”,同学们按照资料中的描述自制了如图所 示的测温装置,图中 A 为一小塑料瓶,B 为一吸管,通过软木 塞与 A 连通,管的下端竖直插在大水槽中,使管内外水面有一 高度差 h。然后进行实验研究: (1)在不同温度下分别测出对应的水柱高度 h,记录的实验数 据如下表所示 温度(℃) 17 19 21 23 25 27 h (cm) 30.0 24.9 19.7 14.6 9.4 4.2 △h=hn-1-hn 2.1 根据表中数据计算相邻两次测量水柱的高度差,并填入表内的空格。由此可得结论: ①当温度升高时,管内水柱高度 h 将 (填:变大,变小,不变); ②水柱高度 h 随温度的变化而 (填:均匀,不均匀)变化;试从理论上分析并 证明结论②的正确性(提示:管内水柱产生的压强远远小于一个大气压): 。 (2)通过实验,同学们发现用“伽利略温度计”来测温度,还存在一些不足之处,其 中主要的不足之处有:① ; ② 。 18.(4 分)已知某一区域的地下埋有一根与地表 面平行的直线电缆,电缆中通有变化的电流,在 其周围有变化的磁场,因此可以通过在地面上测 量闭合试探小线圈中的感应电动势来探测电缆的 确切位置、走向和深度。当线圈平面平行地面测量时,在地面上 a、c 两处测得试探线圈中 的电动势为零,b、d 两处线圈中的电动势不为零;当线圈平面与地面成 45°夹角时,在 b、 d 两处测得试探线圈中的电动势为零。经过测量发现,a、b、c、d 恰好位于边长为 1 米的正 方形的四个顶角上,如图所示。据此可以判定地下电缆在 两点连线的正下方,离地表 面的深度为 米。 四、(60 分)计算题 19.(10 分)上端开口的圆柱形气缸竖直放置,截面积为 0.2 米 2 的活塞 将一定质量的气体和一形状不规则的固体 A 封闭在气缸内。温度为 300 K 时,活塞离气缸底部的高度为 0.6 米;将气体加热到 330K 时,活塞上 升了 0.05 米,不计摩擦力及固体体积的变化。求物体 A 的体积。 20.(8 分)一卫星绕某行星作匀速圆周运动,已知行星表面的重力加速 度为 g 行,行星的质量 M 与卫星的质量 m 之比 M/m=81,行星的半径 R 行与卫星的半 径 R 卫之比 R 行/R 卫=3.6,行星与卫星之间的距离 r 与行星的半径 R 行之比 r/R 行=60。 设卫星表面的重力加速度为 g 卫,则在卫星表面有: 卫mg r MmG 2 …… 经过计算得出:卫星表面的重力加速度为行星表面的重力加速度的三千六百分之一。上 述结果是否正确?若正确,列式证明;若错误,求出正确结果。 21.(13 分)如图所示,一自行车上连接踏脚板的连杆长 R1, 由踏脚板带动半径为 r1 的大齿盘,通过链条与半径为 r2 的后 轮齿盘连接,带动半径为 R2 的后轮转动。 (1)设自行车在水平路面上匀速行过时,受到的平均 阻力为 f,人蹬踏脚板的平均作用力为 F,链条中的张力为 T, 地面对后轮的静摩擦力为 fs,通过观察,写出传动系统中有几个转动轴,分别写出对应的力 矩平衡表达式; (2)设 R1=20 厘米,R2=33 厘米,踏脚大齿盘与后轮齿盘的齿数分别为 48 和 24,计 算人蹬踏脚板的平均作用力与平均阻力之比; (3)自行车传动系统可简化为一个等效杠杆。以 R1 为一力臂,在右框中画出这一杠杆 示意图,标出支点,力臂尺寸和作用力方向。 22.(3 分)如图所示,两条互相平行的光滑金属导轨位于水平面内,距离为 l=0.2 米,在 导轨的一端接有阻值为 R=0.5 欧的电阻,在 X≥0 处有一与水平面垂直的均匀磁场,磁感 强度 B=0.5 特斯拉。一质量为 m=o.1 千克的金属直 杆垂直放置在导轨上,并以 v0=2 米/秒的初速度进入磁场,在安培力和一垂直于杆的水平 外力 F 的共同作用下作匀变速直线运动,加速度大小为 a=2 米/秒 2、方向与初速度方向相 反。设导轨和金属杆的电阻都可以忽略,且接触良好。求: (1)电流为零时金属杆所处的位置; (2)电流为最大值的一半时施加在金属杆上外力 F 的大小和方向; (3)保持其他条件不变,而初速度 v0 取不同值,求开始时 F 的方向与初速度 v0 取值的 关系。 23.(16 分)如图所示为利用电磁作用输送非导电液体装置的示意图。一边长为 L、截面为 正方形的塑料管道水平放置,其右端面上有一截面积为 A 的小喷口,喷口离地的高度 为 h。管道中有一绝缘活塞。在活塞的中部和上部分别嵌有两根金属棒 a、b,其中棒 b 的两端与一电压表相连,整个装置放 在竖直向上的匀强磁场中。当棒 a 中 通有垂直纸面向里的恒定电流 I 时, 活塞向右匀速推动液体从喷口水平射 出,液体落地点离喷口的水平距离为 S。若液体的密度为ρ,不计所有阻力,求: (1)活塞移动的速度; (2)该装置的功率; (3)磁感强度 B 的大小; (4)若在实际使用中发现电压表的读数变小,试分析其可能的原因。 2002 年全国普通高等学校招生统一考试(上海卷) 物理参考答案 一.选择题 1.BC 2.AC 3.B 4.CD 5.AD 6.ABD 7.C 8.C 二.填空题 9.点电荷、理想气体等 10. e0 1 ,2 11.0.40 12.0.34,30×10-5 13.(2μF/A)1/2 三.实验题 14.D 15.(1)如图所示 (2)B C D 16.车轮半径 R 和齿轮的齿数 p,2πRn/p,2Πrn/p 17.(1)5.2,5.1,5.2,5.2, ①变小,②均匀 封闭气体近似作等压变化 V/T=△T/△V=k(k 为常数) △V=k△T=k△t ∴△h=△V/S=k△t/S 即 h 随温度的变化而均匀变化(S 为管的截面积) (2)①测量温度范围小;②温度读数受大气压影响 18.ac, 0.71 四.计算题 19.设 A 的体积为 V,T1=300K,T2=330K,S=0.2 米 2,h=0.6 米, h2=0.6+0.05=0.65 米 ① 等压变化 1 1 T VSh  = 2 2 T VSh  ② (h1S-V)T2=(h2S-V)T1 ∴ V= STT ThTh 12 1221   ③ = 2.0300330 30065.03306.0   =0.02(米 3) 20.所得的结果是错误的。 ①式中的 g 卫并不是卫星表面的重力加速度,而是卫星绕行星作匀速圆周运动的向心加 速度。 正确解法是 卫星表面 2 卫R mG =g 卫 ① 行星表面 2 行R MG =g 行 ② ( 卫 行 R R )2 M m = 行 卫 g g ∴ g 卫=0.16g 行 21.(1)自行车传动系统中的转动轴个数为 2 对踏脚齿盘中心的转动轴可列出:FR1=Tr1 ① 对后轮的转动轴可列出: Tr2=fsR2 ② (2)由 FR1=Tr1 Tr2=fsR2 及 fs=f ③ 可得 2 1 Rf FR s = 2 1 r r = 24 48 ④ ∴ f F = 12 21 Rr Rr = 2024 3348   = 10 33 =3.3 ⑤ (3)如图所示, 22.(1)感应电动势ε=Blv,I=ε/R ∴ I=0 时 v=0 ∴ x=v02/2a=1(米) ① (2)最大电流 Im=Blv0/R I’=Im/2=Blv0/2R 安培力 f=I’Bl=B2l2v0/2R ② =0.02(牛) 向右运动时 F+f=ma F=ma-f=0.18(牛) 方向与 x 轴相反 ③ 向左运动时 F-f=ma F=ma+f=0.22(牛) 方向与 x 轴相反 ④ (3)开始时 v=v0, f=ImBl=B2l2v0/R F+f=ma, F=ma-f=ma-B2l2v0/R ⑤ ∴ 当 v0<maR/B2l2=10 米/秒 时,F>0 方向与 x 轴相反 ⑥ 当 v0>maR/B2l2=10 米/秒 时,F<0 方向与 x 轴相同 ⑦ 23.(1)设液体从喷口水平射出的速度为内,活塞移动的速度为 v v0=s h g 2 ① v0A=Vl2 ② v( 2L A )v0= 2L As h g 2 ③ (2)设装置功率为 P,△t 时间内有△m 质量的液体从喷口射出 P△t= 2 1 △m(v02-v2) ④ ∵ △m=L2v△tρ ⑤ ∴ P= 2 1 L2vρ(v02-v2)= 2 A (1- 4 2 L A )v03 ∴ P= 2 3 4 324 )2( 2 )( h g L sALA  ⑥ (3)∵ P=F 安 v ⑦ ∴ 2 1 L2ρv(v02- 4 2 L A v02)=BILv ⑧ ∴ B= 3 242 0 2 )( IL ALv  = 3 224 4 )( IhL gsAL  ⑨ (4)∵ U=BLv ∴ 喷口液体的流量减少,活塞移动速度减小,或磁场变小等会引起电压表读 数变小 2003 年普通高等学校招生全国统一考试(新课程卷)物理(江苏) 第Ⅰ卷(选择题 共 40 分) 一、本题共 10 小题;每小题 4 分,共 40 分. 在每小题给出的四个选项中,有的小题只有一 个选项正确,有的小题有多个选项正确,全部选对的得 4 分,选不全的得 2 分,有选错 或不答的得 0 分. 1.下列说法中正确的是 A 质子与中子的质量不等,但质量数相等 B 两个质子间,不管距离如何,核力总是大于库仑力 C 同一种元素的原子核有相同的质量数,但中子数可以不同 D 除万有引力外,两个中子之间不存在其它相互作用力 2.用某种单色光照射某种金属表面,发生光电效应,现将该单色光的光强减弱,则 A 光电子的最大初动能不变 B 光电子的最大初动能减少 C 单位时间内产生的光电子数减少 D 可能不发生光电效应 3.如图,甲分子固定在坐标原点 O,乙分子位于 x 轴上,甲分子对乙分子的作用力与两分 子间距离的关系如图中曲线所示,F>0 为斥力,F<0 为引力,a、b、c、d 为 x 轴上四 个特定的位置,现把乙分子从 a 处静止释放,则 A 乙分子从 a 到 b 做加速运动,由 b 到 c 做减速运动 B 乙分子由 a 到 c 做加速运动,到达 c 时速度最大 C 乙分子由 a 到 b 的过程中,两分子间的分子势能一直减少 D 乙分子由 b 到 d 的过程中,两分子间的分子势能一直增加 4.铀裂变的产物之一氦 90( 90 36 Kr)是不稳定的,它经过一系列衰变最终成为稳定的锆 90 ( 90 40 Zr),这些衰变是 A 1 次α衰变,6 次β衰变 B 4 次β衰变 C 2 次α衰变 D 2 次α衰变,2 次β衰变 5.两块大小、形状完全相同的金属平板平行放置,构成一平行板电容器,与它相连接的电 路如图所示,接通开关 K,电源即给电容器充电. A 保持 K 接通,减小两极板间的距离,则两极板间电场的电场强度减小 B 保持 K 接通,在两极板间插入一块介质,则极板上的电量增大 C 断开 K,减小两极板间的距离,则两极板间的电势差减小 D 断开 K,在两极板间插入一块介质,则两极板间的电势差增大 6.一定质量的理想气体, A 先等压膨胀,再等容降温,其温度必低于起始温度 B 先等温膨胀,再等压压缩,其体积必小于起始体积 C 先等容升温,再等压压缩,其温度有可能等于起始温度 D 先等容加热,再绝热压缩,其内能必大于起始内能 7.一弹簧振子沿 x 轴振动,振幅为 4cm,振子的平衡位置位于 x 轴上的 O 点,图 1 中的 a、 b、c、d 为四个不同的振动状态;黑点表示振子的位置,黑点上的箭头表示运动的方向, 图 2 给出的①②③④四条振动图线,可用 于表示振子的振动图象。 A 若规定状态 a 时 t=0,则图象为① B 若规定状态 b 时 t=0,则图象为② C 若规定状态 c 时 t=0,则图象为③ D 若规定状态 d 时 t=0,则图象为④ 8.如图,一玻璃柱体的横截面为半圆形,细的单色光束从空气射向柱体的 O 点(半圆的圆 心),产生反射光束 1 和透射光束 2,已知玻璃折射率为 3 ,入射解为 45°(相应的折 射角为 24°),现保持入射光不变,将半圆柱绕通过 O 点垂直于图 面的轴线顺时针转过 15°,如图中虚线所示,则 A 光束 1 转过 15° B 光束 1 转过 30° C 光束 2 转过的角度小于 15° D 光束 2 转过的角度大于 15° 9.原子从一个能级跃迁到一个较低的能级时,有可能不发射光子,例如在某种条件下,铬 原子的 n=2 能级上的电子跃迁到 n=1 能级上时并不发射光子,而是将相应的能量转交 给 n=4 能级上的电子,使之能脱离原子,这一现象叫做俄歇效应,以这种方式脱离了原 子的电子叫做俄歇电子,已知铬原子的能级公式可简化表示为 En=-A/n2,式中 n=1, 2,3……表示不同能级,A 是正的已知常数,上述俄歇电子的动能是 A 3/16A B 7/16A C 11/16 D 13/16A 10.如图,a 和 b 都是厚度均匀的平玻璃板,它们之间的夹角为Φ,一细光束以入射角θ从 P 点射入,θ>Φ已知此光束由红光和蓝光组成,则当光束透过 b 板后, A 传播方向相对于入射光方向向左偏转Φ角 B 传播方向相对于入射光方向向右偏转Φ角 C 红光在蓝光的左边 D 红光在蓝光的右边 第Ⅱ卷(非选择题 共 110 分) 二、本题共 3 小题,共 21 分,把答案填在题中的横线上或按题目要求作答. 11.(6 分)图中为示波器面板,屏上显示的是一亮度很低、线条较粗 且模糊不清的波形。 (1)若要增大显示波形的亮度,应调节 旋钮。 (2)若要屏上波形线条变细且边缘清晰,应调节 旋钮。 (3)若要将波形曲线调至屏中央,应调节 与 旋钮。 12.(7 分)实验装置如图 1 所示;一木块放在水平长木板上,左侧拴 有一细软线,跨过固定在木板边缘的滑轮与一重物相连,木块右侧与 打点计时器的纸带相连,在重物牵引下,木块在木板上向左运动,重物落地后,木块继续向 左做匀减速运动,图 2 给出了重物落地后,打点计时 器在纸带上打出的一些点,试根据给出的数据,求木 块与木板间的摩擦因数μ。要求写出主要的运算过程, 结果保留 2 位有效数字。(打点计时器所用交流电频率 为 50Hz,不计纸带与木块间的拉力。取重力加速度 g =10m/s2) 13.(8 分)要测量一块多用电表直流 10 mA档的内阻 RA(约为 40Ω),除此多用电表外, 还有下列器材:直流电源一个(电动势 E 约为 1.5V,内阻可忽略不计),电阻一个(阻值 R 约为 150Ω),电键一个,导线若干。 要求(1)写出实验步骤;(2)给出 RA 的表达式. 三、本题共 7 小题,89 分,解答应写出必要的文字说明、方程式和重要演算步骤.只写出最 后答案的不能得分. 有效值计算的题,答案中心须明确写出数值和单位. 14.(12 分)据美联社 2002 年 10 月 7 日报道,天文学家在太阳系的 9 大行星之外,又发现 了一颗比地球小得多的新行星,而且还测得它绕太阳公转的周期约为 288 年. 若把它和地球 绕太阳公转的轨道都看作圆,问它与太阳的距离约是地球与太阳距离的多少倍。(最后结果 可用根式表示) 15.(12 分)当物体从高空下落时,空气阻力随速度的增大而增大,因此经过一段距离后将 匀速下落,这个速度称为此物体下落的终极速度. 已知球形物体速度不大时所受的空气阻力 正比于速度 v,且正比于球半径 r,即阻力 f=krv,k 是比例系数。对于常温下的空气,比例 系为 k=3.4×10-4Ns/m2。已知水的密度ρ=1.0×103kg/m3,取重力加速度 g=10m/s2。试求 半径 r=0.10mm 的球形雨滴在无风情况下的终极速度 vr。(结果取两位数字) 16.(13 分)在如图所示的电路中,电源的电动势ε=3.0V,内阻 r=1.0Ω,电阻 R1=10Ω, R2=10Ω,R3=30Ω,R4=35Ω;电容器的电容 C=uF,电容器原来不带电.求接通电键 K 后流过 R4 的总电量。 17.(13 分)串列加速器是用来产生高能离子的装置.图中虚线框内为其主体的原理示意图, 其中加速管的中部 b 处有很高的正电势 U。a、c 两端均有电极接地(电势为零)。现将速度 很低的负一价碳离子从 a 端输入,当离子到达 b 处 时,可被设在 b 处的特殊装置将其电子剥离,成为 n 价正离子,而不改变其速度大小,这些正 n 价碳 离子从 c 端飞出后进入一与其速度方向垂直的、磁 感强度为 B 的匀强磁场中,在磁场中做半径为 R 的圆周运动.已知碳离子的质量 m=2.0×10 -26kg,U=7.5×105V,B=0.05T,n=2,基元电荷 e=1.6×10-19C,求 R。 18.(13 分)如图所示,两根平行金属导轨固定在水平桌面上,每根导轨每米的电阻为 r0= 0.10Ω/m,导轨的端点 P、Q 用电阻可忽略的导线相连,两导轨间的距离 l=0.20m。有随时 间变化的匀强磁场垂直于桌面,已知磁感强度 B 与时间 t 的关系为 B=kt,比例系数 k=0.020T/s,一电阻不计的 金属杆可在导轨上无摩擦地滑动,在滑动过程中保持与 导轨垂直,在 t=0 时刻,金属杆紧靠在 P、Q 端,在外力作用下,杆以恒定的加速度从静 止开始向导轨的另一端滑动,求在 t=6.0s 时金属杆所受的安培力。 19.(13 分)图 1 所示为一根竖直悬挂的不可伸长的轻绳,下端拴一小物块 A,上端固定在 C 点且与一能测量绳的拉力的测力传感器相连.已知有一质量为 m0 的子弹 B 沿水平方向以速 度 v0 射入 A 内(未穿透),接着两者一起绕 C 点在竖直面内做圆周运动,在各种阻力都可 忽略的条件下测力传感器测得绳的拉力 F 随时间t 的变化关系如图 2 所示。已知子弹射入的 时间极短,且图 2 中 t=0 为 A、B 开始以相同速度运动的时刻,根据力学规律和题中(包 括图)提供的信息,对反映悬挂系统本身性质的物理量(例如 A 的质量)及 A、B 一起运 动过程中的守恒量,你能求得哪些定量的结果? 20.(13 分)(1)如图 1,在光滑水平长直轨道上,放着一个静止的弹簧振子,它由一轻弹 簧两端各联结一个小球构 成,两小球质量相等。现 突然给左端小球一个向右 的速度μ0,求弹簧第一次 恢复到自然长度时,每个小球的速度。 (2)如图 2,将 N 个这样的振子放在该轨道上,最左边的振子 1 被压缩至弹簧为某一 长度后锁定,静止在适当位置上,这时它的弹性势能为 E0。其余各振子间都有一定的距离, 现解除对振子 1 的锁定,任其自由运动,当它第一次恢复到自然长度时,刚好与振子 2 碰撞, 此后,继续发生一系列碰撞,每个振子被碰后刚好都是在弹簧第一次恢复到自然长度时与下 一个振子相碰.求所有可能的碰撞都发生后,每个振子弹性势能的最大值。已知本题中两球 发生碰撞时,速度交换,即一球碰后的速度等于另一球碰前的速度。 参考答案 一、全题 40 分,每小题 4,每小题全选对的给 4 分,选不全的给 2 分,有选错的给 0 分, 不答的给 0 分. 1.A 2.AC 3.BC 4.B 5.BC 6.CD 7.AD 8.BC 9.C 10.D 二、全题 21 分,其中 11 题 6 分,12 题 7 分,13 题 8 分. 11.(1)辉度(或写为 ) (2)聚焦(或写为○)垂直位移(或写为↑↓)水平位移(或 写为→←) 12.由给出的数据可知,重物落地后,木块在连续相等的时间 T 内的位移分别是: ,31.4,81.4,29.5,76.5 ,25.7,71.6,21.7,72.7 8765 4321 cmscmscmscms cmscmscmscms   以 a 表示加速度,根据匀变速直线运动的规律,有 2 2 48372615 /0.30.04sT 4)]()()()[(4 1 sma aTsssssssss   解得又知 重物落地后木块只受摩擦力的作用,以 m 表示木块的质量,根据牛顿定律,有 30.0  解得mamg 13.(1)实验步骤:①用多用电表的直流电压档测量电源电动势 E. ②用多用电表的  档测 电阻阻值 R. ③将多用电表置于直流 10mA 档,与电阻 R 及电键串联后接在电源两端, 合上电键,记下多用电表读数 I. (2) RI ERA  三、参考解答: 14.(12 分)设太阳的质量为 M;地球的质量为 0m 绕太阳公转的周期为 T0,太阳的距离为 R0,公转角速度为 0 ;新行星的质量为 m ,绕太阳公转的周期为 T,与太阳的距离为 R,公转角速度为 ,根据万有引力定律和牛顿定律,得 0 0 0 2 002 0 02 2 22        TT RmR MmGRmR MmG 由以上各式得 3 2 00 )(T T R R  已知 T=288 年,T0=1 年 得 3 2 0 288(44 或 R R ) 15.(12 分)雨滴下落时受两个力作用:重力,方向向下;空气阻力,方向向上,当雨滴达 到终极速度 rv 后,加速度为零,二力平衡,用 m 表示雨滴质量,有 0 Tkrvmg ①  3 3 4 rm  ② 由①②得终极速度 k grvr 3 4 3  ③ 代入数值得 rv =1.2m/s . 16.(13 分)由电阻的串并联公式,得闭合电路的总电阻为 rRRR RRRR   321 321 )( 由欧姆定律得,通过电源的电流 RI  电源的端电压 IrU   电阻 R3 两端的电压 URR RU 32 3  通过 R4 的总电量就是电容器的电量 Q=CU′ 由以上各式并代入数据解得 CQ 4100.2  17.(13 分)设碳离子到达b 处时的速度为 1v ,从 c 端射出时的速度为 2v ,由能量关系得 eUmv 2 12 1 ① eUmvmv  2 1 2 2 2 1 2 1 ②进入磁场后,碳离子做圆周运动,可得 R vmBnev 2 2 2  ③ 由以上三式可得 e nmU BnR )1(21  ④ 由④式及题给数值可解得 .75.0 mR  18.(13 分)以 a 表示金属杆运动的加速度,在t 时刻,金属杆与初始位置的距离 2 2 1 atL  此时杆的速度 atv  ,这时,杆与导轨构成的回路的面积 LIS  ,回路中的感应电动势 kt BttB t BBlvt BS     )(ktB而 回路的总电阻 02LrR  回路中的感应电流 Ri  作用于杆的安培力 BliF  解得 tr lkF 0 22 1 2 3 ,代入数据为 NF 31044.1  19.(13 分)由图 2 可直接看出,A、B 一起做周期性运动,运动的周期 T=2t0 ① 令 m 表示 A 的质量, l 表示绳长. 1v 表示 B 陷入 A 内时即 0t 时 A、B 的速度(即圆 周运动最低点的速度), 2v 表示运动到最高点时的速度,F1 表示运动到最低点时绳的拉 力,F2 表示运动到最高点时绳的拉力,根据动量守恒定律,得 1000 )( vmmvm  ② 在最低点和最高点处运用牛顿定律可得 t vmmgmmF 2 1 001 )()(  ③ t vmmgmmF 2 2 002 )()(  ④ 根据机械能守恒定律可得 2 20 2 100 )(2 1)(2 1)(2 vmmvmmgmml  ⑤ 由图 2 可知 02 F ⑥ mFF 1 ⑦ 由以上各式可解得,反映系统性质的物理量是 06 mg Fm m  ⑧ g F vml m 2 2 0 2 0 5 36 ⑨ A、B 一起运动过程中的守恒量是机械能 E,若以最低点为势能的零点,则 2 10 )(2 1 vmmE  ⑩ 由②⑧⑩式解得 g F vmE m 2 0 2 03 ⑾ 20.(13 分)(1)设每个小球质量为 m ,以 1u 、 2u 分别表示弹簧恢复到自然长度时左右两 端小球的速度. 由动量守恒和能量守恒定律有 021 mumumu  (以向右为速度正方 向) 2 0 2 2 2 1 2 1 2 1 2 1 mumumu  解得 021201 ,00, uuuuuu  或 由于振子从初始状态到弹簧恢复到自然长度的过程中,弹簧一直是压缩状态,弹性力使 左端小球持续减速,使右端小球持续加速,因此应该取解: 021 ,0 uuu  (2)以 v1、v1’分别表示振子 1 解除锁定后弹簧恢复到自然长度时左右两小球的速度,规定 向右为速度的正方向,由动量守恒和能量守恒定律,mv1+mv1’=0 0 2 1 2 1 2 1 2 1 Evmmv  解得 .,, 0 1 0 1 0 1 0 1 m Evm Evm Evm Ev  或 在这一过程中,弹簧一直是压缩状态,弹性力使左端小球向左加速,右端小球向右加速, 故应取解: m Evm Ev 0 1 0 1 ,  振子 1 与振子 2 碰撞后,由于交换速度,振子 1 右端小 球速度变为 0,左端小球速度仍为 1v ,此后两小球都向左运动,当它们向左的速度相同 时,弹簧被拉伸至最长,弹性势能最大,设此速度为 10v ,根据动量守恒定律: 1102 mvmv  用 E1 表示最大弹性势能,由能量守恒有 2 11 2 10 2 10 2 1 2 1 2 1 mvEmvmv  解得 01 4 1 EE  2003 年全国普通高等学校招生全国统一考试 物理(广东卷) 第Ⅰ卷(选择题 共 40 分) 一、本题共 10 小题;每小题 4 分,共 40 分. 在每小题给出的四个选项中,有的小题只有一 个选项正确,有的小题有多个选项正确,全部选对的得 4 分,选不全的得 2 分,有选错 或不答的得 0 分. 1.下列说法中正确的是 A.质子与中子的质量不等,但质量数相等 B.两个质子间,不管距离如何,核力总是大于库仑力 C.同一种元素的原子核有相同的质量数,但中子数可以不同 D.除万有引力外,两个中子之间不存在其它相互作用力 2.用某种单色光照射某种金属表面,发生光电效应,现将该单色光的光强减弱,则 A.光电子的最大初动能不变 B.光电子的最大初动能减少 C.单位时间内产生的光电子数减少 D.可能不发生光电效应 3.相距很远的两个分子,以一定的初速度相向运动,直到距离最小。在这个过程中,两分 子间的分子势能 A.一直增大 B.一直减小 C.先增大,后减小 D.先减小,后增大 4.铀裂变的产物之一氪 90( 90 36 Kr)是不稳定的,它经过一系列衰变最终成为稳定的锆 90 ( 90 40 Zr),这些衰变是 A.1 次衰变,6 次衰变 B.4 次衰变 C.2 次衰变 D.2 次衰变,2 次衰变 5.两块大小、形状完全相同的金属平板平行放置,构成一平 行板电容器,与它相连接的电路如图所示,接通开关 K,电源 即给电容器充电. A.保持 K 接通,减小两极板间的距离,则两极板间电场 的电场强度减小 B.保持 K 接通,在两极板间插入一块介质,则极板上的电量增大 C.断开 K,减小两极板间的距离,则两极板间的电势差减小 D.断开 K,在两极板间插入一块介质,则两极板间的电势差增大 6.一定质量的理想气体, A.先等压膨胀,再等容降温,其温度必低于起始温度 B.先等温膨胀,再等压压缩,其体积必小于起始体积 C.先等容升温,再等压压缩,其温度有可能等于起始温度 D.先等容加热,再绝热压缩,其内能必大于起始内能 7.一弹簧振子沿 x 轴振动,振幅为 4cm,振子的平衡位置位于 x 轴上的 0 点,图 1 中的 a、 b、c、d 为四个不同的振动状态;黑点表示振子的位置,黑点上的箭头表示运动的方向, 图 2 给出的①②③④四条振动图线,可用于表示振子的振动图象。 A.若规定状态 a 时 t=0,则图象为① B.若规定状态 b 时 t=0,则图象为② C.若规定状态 c 时 t=0,则图象为③ D.若规定状态 d 时 t=0,则图象为④ 8.如图,一玻璃柱体的横截面为半圆形,细的单色光束从空 气 射向柱体的 O 点(半圆的圆心),产生反射光束 1 和透射光束 2, 已知玻璃折射率为 3 ,入射解为 45°(相应的折射角为 24°),现保持入射光不变,将半圆柱绕通过 O 点垂直于图面 的 轴线顺时针转过 15°,如图中虚线所示,则 A.光束 1 转过 15° B.光束 1 转过 30° C.光束 2 转过的角度小于 15° D.光束 2 转过的角度大于 15° 9.原子从一个能级跃迁到一个较低的能级时,有可能不发射光子,例如在某种条件下,铬 原子的 n=2 能级上的电子跃迁到 n=1 能级上时并不发射光子,而是将相应的能量转交 给 n=4 能级上的电子,使之能脱离原子,这一现象叫做俄歇效应,以这种方式脱离了 原子的电子叫做俄歇电子,已知铬原子的能级公式可简化表示为 En= 2n A ,式中 n=1, 2,3……表示不同能级,A 是正的已知常数,上述俄歇电子的动能是 A. 16 3 A B. 16 7 A C. 16 11 A D. 16 13 A 10.如图,a 和 b 都是厚度均匀的平玻璃板,它们之间的夹角为,一细光束以入射角从 P 点射入,> .已知此光束由红光和蓝光组成,则当光束透过 b 板后, A.传播方向相对于入射光方向向左偏转角 B.传播方向相对于入射光方向向右偏转角 C.红光在蓝光的左边 D.红光在蓝光的右边 第Ⅱ卷(非选择题 共 110 分) 二、本题共 3 小题,共 21 分. 11.(6 分)下图为—正在测量中的多用电表表盘。 (1)如果是用直流 10V 档测量电压,则读数为 V。 (2)如果是用×100档测量电阻,则读数为 。 (3)如果是用直流 5mA 档测量电流,则读数为 mA。 12.(7 分)实验装置如图 1 所示;一木块放在水平长木板上,左侧拴有一细软线,跨过固 定在木板边缘的滑轮与一重物相连,木块右侧与打点计时器的纸带相连,在重物牵引下,木 块在木板上向左运动,重物落地后,木块 继续向左做匀减速运动,图 2 给出了重物 落地后,打点计时器在纸带上打出的一些 点,试根据给出的数据,求木块与木板间 的摩擦因数。要求写出主要的运算过程, 结果保留 2 位有效数字。(打点计时器所用 交流电频率为 50Hz,不计纸带与木块间的拉力。取重力加速度 g=10m/s2) 13.(8 分)要测量一块多用电表直流 10mA 档的内阻 RA(约为 40),除此多用电表外, 还有下列器材:直流电源一个(电动势 E 约为 1.5V,内阻可忽略不计),电阻一个(阻 值 R 约为 150),电键一个,导线若干。 要求(1)写出实验步骤;(2)给出 RA 的表达式. 三、本题共 7 小题,89 分,解答应写出必要的文字说明、方程式和重要演算步骤.只写出最 后答案的不能得分. 有效值计算的题,答案中必须明确写出数值和单位. 14.(12 分)据美联社 2002 年 10 月 7 日报道,天文学家在太阳系的 9 大行星之外,又发现 了一颗比地球小得多的新行星,而且还测得它绕太阳公转的周期约为 288 年. 若把它和 地球绕太阳公转的轨道都看作圆,问它与太阳的距离约是地球与太阳距离的多少倍。(最 后结果可用根式表示) 15.(12 分)当物体从高空下落时,空气阻力随速度的增大而增大,因此经过一段距离后将 匀速下落,这个速度称为此物体下落的终极速度. 已知球形物体速度不大时所受的空气 阻力正比于速度 v,且正比于球半径 r,即阻力 f=krv,k 是比例系数。对于常温下的空 气,比例系为 k=3.4×10-4Ns/m2。已知水的密度=1.0×103kg/m3,取重力加速度 g= 10m/s2。试求半径 r=0.10mm 的球形雨滴在无风情况下的终极速度 vT。(结果取两位数 字) 16.(13 分)在如图所示的电路中,电源的电动势ε=3.0V, 内阻 r=1.0,电阻 R1=10,R2=10,R3=30,R4=35; 电容器的电容 C=F,电容器原来不带电.求接通电键 K 后 流过 R4 的总电量。 17.(13 分)串列加速器是用来产生高能离子 的装置.图中虚线框内为其主体的原理示意图,其中加速管的中部 b 处有很高的正电势 U,a、c 两端均有电极接地(电势为零)。现将速度很低的负一价碳离子从 a 端输入, 当离子到达 b 处时,可被设在 b 处的特殊装置将其电子剥离,成为 n 价正离子,而不改 变其速度大小,这些正 n 价碳离子从 c 端飞出后进入一与其速度方向垂直的、磁感强度 为 B 的匀强磁场中,在磁场中做半径为 R 的圆周运动.已知碳离子的质量 m=2.0×10- 26kg,U=7.5×105V,B=0.5T,n=2,基元电荷 e=1.6×10-19C,求 R. 18.(13 分)在图 1 所示区域(图中直角坐标系 Oxy 的 1、3 象限)内有匀强磁场,磁感强 度方向垂直于图面向里,大小为 B.半径为 l、圆心角为 60°的扇形导线框 OPQ 以角速 度ω绕 O 点在图面内沿逆时针方向匀速转动,导线框回路电阻为 R. (1)求线框中感应电流的最大值 I0 和交变感应电流的频率 f。 (2)在图 2 上画出线框转一周的时间内感应电流 I 随时间 t 变化的图像(规定与图 1 中线框的位置相应的时刻为 t=0) 19.(13 分)图 1 所示为一根竖直悬挂的不可伸长的轻绳,下端拴一小物块 A,上端固定在 C 点且与一能测量绳的拉力的测力传感器相连.已知有一质量为 m0 的子弹 B 沿水平方向 以速度 v0 射入 A 内(未穿透),接着两者一起绕 C 点在竖直面内做圆周运动,在各种阻 力都可忽略的条件下测力传感器测得绳的拉力 F 随时间t 的变化关系如图 2 所示。已知 子弹射入的时间极短,且图 2 中 t=0 为 A、B 开始以相同速度运动的时刻,根据力学规 律和题中(包括图)提供的信息,对反映悬挂系统本身性质的物理量(例如 A 的质量) 及 A、B 一起运动过程中的守恒量,你能求得哪些定量的结果? 20.(13 分)(1)如图 1,在光滑水平长直轨道上,放着一个静止的弹簧振子,它由一轻弹 簧两端各联结一个小球构成,两小球质量相等。现突然给左端小球一个向右的速度μ0, 求弹簧第一次恢复到自然长度时,每个小球的速度。 (2)如图 2,将 N 个这样的振子放在该轨道上,最左边的振子 1 被压缩至弹簧为某一 长度后锁定,静止在适当位置上,这时它的弹性势能为 E0。其余各振子间都有一定的距离, 现解除对振子 1 的锁定,任其自由运动,当它第一次恢复到自然长度时,刚好与振子 2 碰撞, 此后,继续发生一系列碰撞,每个振子被碰后刚好都是在弹簧第一次恢复到自然长度时与下 一个振子相碰.求所有可能的碰撞都发生后,每个振子弹性势能的最大值。已知本题中两球 发生碰撞时,速度交换,即一球碰后的速度等于另一球碰前的速度。 2003 年全国普通高等学校招生统一考试(广东卷) 物理参考答案 一、全题 40 分,每小题 4,每小题全选对的给 4 分,选不全的给 2 分,有选错的给 0 分, 不答的给 0 分. 1.A 2.AC 3.D 4.B 5.BC 6.CD 7.AD 8.BC 9.C 10.D 二、全题 21 分,其中 11 题 6 分,12 题 7 分,13 题 8 分. 11.(1)6.5 (2)8.0×102 (3)3.25 12.由给出的数据可知,重物落地后,木块在连续相等的时间 T 内的位移分别是: ,cm31.4,cm81.4,cm29.5,cm76.5 ,cm25.7,cm71.6,cm21.7,cm72.7 8765 4321   ssss ssss 以 a 表示加速度,根据匀变速直线运动的规律,有 2 2 48372615 /0.30.04s 4)]()()()[(4 1 smaT aTsssssssss   解得又知 重物落地后木块只受摩擦力的作用,以 m 表示木块的质量,根据牛顿定律,有 30.0  解得mamg 13.(1)实验步骤:①用多用电表的直流电压档测量电源电动势 E. ②用多用电表的  档测电阻阻值 R. ③将多用电表置于直流 10mA 档,与电阻 R 及电键串联后接在电源两 端,合上电键,记下多用电表读数 I. (2) RI ERA  三、参考解答: 14.(12 分)设太阳的质量为 M;地球的质量为 0m 绕太阳公转的周期为 T0,太阳的距离为 R0,公转角速度为 0 ;新行星的质量为 m ,绕太阳公转的周期为 T,与太阳的距离为 R,公转角速度为 ,根据万有引力定律和牛顿定律,得 0 0 0 2 002 0 02 2 22        TT RmR MmGRmR MmG 由以上各式得 3 2 00 )(T T R R  已知 T=288 年,T0=1 年 得 3 2 0 288(44 或 R R ) 15.(12 分)雨滴下落时受两个力作用:重力,方向向下;空气阻力,方向向上,当雨滴达 到终极速度 Tv 后,加速度为零,二力平衡,用 m 表示雨滴质量,有 0 Tkrvmg ①  3 3 4 rm  ② 由①②得终极速度 k grvT 3 4 3 ③ 代入数值得 Tv =1.2m/s . 16.(13 分)由电阻的串并联公式,得闭合电路的总电阻为 rRRR RRRR   321 321 )( 由欧姆定律得,通过电源的电流 R I  电源的端电压 IrU   电阻 R3 两端的电压 URR RU 32 3  通过 R4 的总电量就是电容器的电量 Q=CU′ 由以上各式并代入数据解得 CQ 4100.2  17.(13 分)设碳离子到达b 处时的速度为 1v ,从 c 端射出时的速度为 2v ,由能量关系得 eUmv 2 12 1 ① neUmvmv  2 1 2 2 2 1 2 1 ② 进入磁场后,碳离子做圆周运动,可得 R vmBnev 2 2 2  ③ 由以上三式可得 e nmU BnR )1(21  ④ 由④式及题给数值可解得 .75.0 mR  18.(13 分) (1)在从图 1 中位置开始(t=0 转 过 60°的过程中,经Δt,转角Δ=ωΔt,回路的磁通增量为 Bl 2 2 1  由法拉第电磁感应定律,感应电动势为 t  因匀速转动,这就是最大的感应电动势。由欧姆定律可求得 I0= 2 2 1 BlR  前半圈和后半圈 I(t)相同,故感应电流频率等于旋转频率的 2 倍, f=   (2)图线如图所示 19.(13 分)由图 2 可直接看出,A、B 一起做周期性运动,运动的周期 T=2t0 ① 令 m 表示 A 的质量, l 表示绳长. 1v 表示 B 陷入 A 内时即 0t 时 A、B 的速度(即圆周 运动最低点的速度), 2v 表示运动到最高点时的速度,F1 表示运动到最低点时绳的拉力, F2 表示运动到最高点时绳的拉力,根据动量守恒定律,得 1000 )( vmmvm  ② 在最低点和最高点处运用牛顿定律可得 l vmmgmmF 2 1 001 )()(  ③ l vmmgmmF 2 2 002 )()(  ④ 根据机械能守恒定律可得 2 20 2 100 )(2 1)(2 1)(2 vmmvmmgmml  ⑤ 由图 2 可知 02 F ⑥ mFF 1 ⑦ 由以上各式可解得,反映系统性质的物理量是 06 mg Fm m  ⑧ g F vml m 2 2 0 2 0 5 36 ⑨ A、B 一起运动过程中的守恒量是机械能 E,若以最低点为势能的零点,则 2 10 )(2 1 vmmE  ⑩ 由②⑧⑩式解得 g F vmE m 2 0 2 03 ⑾ 20.(13 分)(1)设每个小球质量为 m ,以 1u 、 2u 分别表示弹簧恢复到自然长度时左右两 端小球的速度. 由动量守恒和能量守恒定律有 021 mumumu  (以向右为速度正方向) 2 0 2 2 2 1 2 1 2 1 2 1 mumumu  解得 0, 201  uuu 021 ,0 uuu 或 由于振子从初始状态到弹簧恢复到自然长度的过程中,弹簧一直是压缩状态,弹性力使 左端小球持续减速,使右端小球持续加速,因此应该取解: 021 ,0 uuu  (2)以 v1、v1´分别表示振子 1 解除锁定后弹簧恢复到自然长度时左右两小球的速度,规定 向右为速度的正方向,由动量守恒和能量守恒定律, mv1+mv1´=0 0 2 1 2 1 2 1 2 1 Evmmv  解得 m Ev 0 1  , m Ev 0 1  m Ev 0 1 或 , m Ev 0 1  在这一过程中,弹簧一直是压缩状态,弹性力使左端小球向左加速,右端小球向右加速, 故应取解: m Ev 0 1  , m Ev 0 1  振子 1 与振子 2 碰撞后,由于交换速度,振子 1 右端小球速度变为 0,左端小球速度仍 为 1v ,此后两小球都向左运动,当它们向左的速度相同时,弹簧被拉伸至最长,弹性势能 最大,设此速度为 10v ,根据动量守恒定律: 1102 mvmv  用 E1 表示最大弹性势能,由能量守恒有 2 11 2 10 2 10 2 1 2 1 2 1 mvEmvmv  解得 01 4 1 EE  振子 2 被碰撞后瞬间,左端小球速度为 m E0 右端小球速度为 0. 以后弹簧被压缩,当 弹簧再恢复到自然长度时,根据(1)题结果,左端小球速度 v2=0,右端小球速度 m Ev 0' 2  , 与振子 3 碰撞,由于交换速度,振子 2 右端小球速度变为 0,振子 2 静止,弹簧为自然长度, 弹性势能为 E2=0. 同样分析可得 E2=E3= ……EN-1=0 振子 N 被碰撞后瞬间,左端小球速度 m EvN 0 1   ,右端小球速度为 0,弹簧处于自然 长度.此后两小球都向右运动,弹簧被压缩,当它们向右的速度相同时,弹簧被压缩至最短, 弹性势能最大. 此速度为 vN0,根据动量守恒定律, 2mvN0=mv´N-1 用 EN 表示最大弹性势能,根据能量守恒,有 2 1 2 0 2 0 2 1 2 1 2 1  NNNN mvEmvmv 解得 04 1 EEN  2003 年全国普通高等学校招生全国统一考试物理(上海卷) 考生注意: 1. 答卷前,考生务必将姓名、准考证号、校验码等填写清楚。 2. 本试卷共 8 页,满分 150 分。考试时间 120 分钟。考生应用钢笔或圆珠笔将答案直接写 在试卷上。 3. 第 19、20、21、22、23 题要求写出必要的文字说明、方程式和重要的演算步骤。只写 出最后答案,而未写出主要演算过程的,不能得分。有数字计算的问题,答案中必须明确写 出数值和单位。 第Ⅰ卷(共 60 分) 一、(40 分)选择题,本大题共 8 小题,每小题 5 分,每小题给出的四个答案中,至少有一 个是正确的。把正确答案全选出来,并将正确答案前面的字母填写在题后的方括号内。 每一小题全选对的得 5 分,选对但不全,得部分分;有选错或不答的,得 0 分。填写在 方括号外的字母,不作为选出的答案。 1.在核反应方程 )(17 8 14 7 4 2 XONHe  的括弧中,X 所代表的粒子 A. H1 1 B. H2 1 C. e0 1 D. n0 1 2.关于机械波,下列说法正确的是 A.在传播过程中能传递能量 B.频率由波源决定 C.能产生干涉,衍射现象 D.能在真空中传播 3.爱因斯坦由光电效应的实验规律,猜测光具有粒子性,从而提出光子说,从科学研究的 方法来说,这属于 A.等效替代 B.控制变量 C.科学假说 D.数学归纳 4.一个质量为 0.3kg 的弹性小球,在光滑水平面上以 6m/s 的速度垂直撞到墙上,碰撞后小 球沿相反方向运动,反弹后的速度大小与碰撞前相同,则碰撞前后小球速度变化量的 大小△v 和碰撞过程中墙对小球做功的大小 W 为 A.△v=0 B.△v=12m/s C.W=0 D.W=10.8J 5.一负电荷仅受电场力的作用,从电场中的 A 点运动到 B 点,在此过程中该电荷作初速度 为零的匀加速直线运动,则 A、B 两点电场强度 EA、EB 及该电荷的 A、B 两点的电势能 εA、、εB 之间的关系为 A.EA=EB. B.EA<EB. C.εA=εB. D.εA>εB 6.粗细均习的电阻丝围成的正方形线框置于有界匀强磁场中,磁场方向垂直于线框平面, 其边界与正方形线框的边平行。现使线框以同样大小的速度沿四个不同方向平移出磁 场,如图所示,则在移出过程中线框的一边 a、b 两点间电势差绝对值最大的是 7.一质量不计的直角形支架两端分别连接质量为 m 和 2m 的小球 A 和 B。支架的两直角边 长度分别为 2l 和 l,支架可绕固定轴 O 在竖直平面内无摩擦 转动,如图所示。开始时 OA 边处于水平位置,由静止释放, 则 A.A 球的最大速度为 2 gl B.A 球速度最大时,两小球的总重力势能最小 C.A 球速度最大时,两直角边与竖直方向的夹角为 45° D.A、B 两球的最大速度之比 1:2: BA vv 8.劈尖干涉是一种薄膜干涉,其装置如图 1 所示, 将一块平板玻璃放置在另一平板玻璃之上,在一端夹 入两张纸片,从而在两玻璃表面之间形成一个劈形空 气薄膜。当光垂直入射后,从上往下看到的干涉条纹 如图 2 所示。干涉条纹有如下特点: (1) 任意一条明条纹或暗条纹所在位置下面的薄膜厚度相等; (2)任意相邻明条纹和暗条纹所对应的薄膜厚度差恒定。现若在图 1 装置中抽去一张纸片, 则当光垂直入射到新的劈形空气薄膜后,从上往下观察到的干涉条纹 A.变疏 B.变密 C.不变 D.消失 二、(20 分)填空题,本大题共 5 小题,每小题 4 分。答案写在题中横线上的空白处或指定 位置,不要求写出演算过程。 9.卢瑟福通过 实验,发现了原子中间有 一个很小的核,并由此提出了原子的核式结构模型。 右面平面示意图中的四条线表示α粒子运动的可能轨迹, 在图中完成中间两条α粒子的运动轨迹。 10.细绳的一端在外力作用下从 t=0 时刻开始做简谐振动,激发出一列简谐横波。在细绳 上选取 15 个点,图 1 为 t=0 时刻各点所处的位置,图 2 为 t=T/4 时刻的波形图(T 为 波的周期)。在图 3 中画出 t=3T/4 时刻的波形图。 11.有质量的物体周围存在着引力场。万在引力和库仑力有类似的规律,因此我们可以用定 义静电场场强的方法来定义引力场的场强。由此可得,与质量为 M 的质点相距 r 处的 引力场场强的表达式为 EG= (万有引力恒量用 G 表示)。 12.若氢原子的核外电子绕核作半径为 r 的匀速圆周运动,则其角速度ω= ; 电子绕核的运动可等效为环形电流,则电子运动的等效电流 I= 。(已知电 子的质量为 m,电量为 e,静电力恒量用 k 表示) 13.某登山爱好者在攀登珠穆朗峰的过程中,发现他携带的手表表面玻璃发生了爆裂。这种 手表是密封的,出厂时给出的参数为:27℃时表内气体压强为 1×105Pa;在内外压强 差超过 6×104Pa 时,手表表面玻璃可能爆裂。已知当时手表处的气温为-13℃,则手 表表面玻璃爆裂时表内气体压强的大小为 Pa;已知外界大气压强随高度 变化而变化,高度每上升 12m,大气压强降低 133Pa,设海平面大气压为 1×105Pa,则 登山运动员此时的海拔高度约为 m。 三、(30 分)实验题。其中第 14、15 小题和第 17 小题(1)为选择题,选出全部正确答案, 选对但不全,得部分分;有选错或不答的,得 0 分。 14.(5 分)如图所示,在研究平抛运动时,小球 A 沿轨道滑下,离开轨道末端(末端水平) 时撞开轻质接触式开关 S,被电磁铁吸住的小球 B 同时自由下落。改变整个装置的高度 H 做同样的实验,发现位于同一高度的 A、B 两球总是同时 落地,该实验现象说明了 A 球在离开轨道后 A.水平方向的分运动是匀速直线运动。 B.水平方向的分运动是匀加速直线运动。 C.竖直方向的分运动是自由落体运动。 D.竖直方向的分运动是匀速直线运动。 15.(5 分)在右图所求的光电管的实验中,发现用一定频率的 A 单色照射光电管时,电流表指针会发生偏转, 而用另一频率的 B 单色光照射时不发生光电效应,那么 A.A 光的频率大于 B 光的频率。 B.B 光的频率大于 A 光的频率。 C.用 A 光照射光电管时流过电流表 G 的电流方向是 a 流向 b。 D.用 A 光照射光电管时流过电流表 G 的电流方向是 b 流向 a。 16.(6 分)如图所示,在“有固定转动轴物体的平衡条件 实验中,调节力矩盘使其平衡,弹簧秤的 读数为 N,此时力矩盘除受到钩码作用力 F1、F2、F3 和弹簧拉力 F4 外,主要还受 力和 力的作用,如果每个钩码的质量均为 0.1kg,盘上各圆的半径分别是 0.05m、0.10m、0.15m、0.20m(取 g=10m/s2), 则 F2 的力矩是 N·m。有同学在做这个实验时,发现顺时针力矩之和与逆 时针力矩之和存在较大差距。检查发现读数和计算均无差错,请指出造成这种差距的一 个可能原因,并提出简单的检验方法(如例所示,将答案填在下表空格中)。 可能原因 检验方法 例 力矩盘面没有调到竖直 用一根细线挂一钩码靠近力矩盘面,如果细线与力矩 盘面间存在一个小的夹角,说明力矩盘不竖直。 答 17.(7 分)有同学在做“研究温度不变时气体的压强跟体积的关系”实验时,用连接计算 机的压强传感器直接测得注射器内气体的压强值。 缓慢推动活塞,使注射器内空气柱从初始体积 20.0ml 变为 12.0ml。实验共测了 5 次, 每次体积值直接从注射器的刻度 上读出并输入计算机,同时由压强传感器 测得对应体积的压强值。实验完成后, 计算机屏幕上立刻显示出如下表中所示的实验 结果。 序号 V (ml) P (×105Pa) PV (×105Pa·ml) 1 20.0 1.0010 20.020 2 18.0 1.0952 19.714 3 16.0 1.2313 19.701 4 14.0 1.4030 19.642 5 12.0 1.6351 19.621 (1)仔细观察不难发现,pV(×105Pa·ml)一栏中的数值越来越小,造成这一现象的可 能原因是 A.实验时注射器活塞与筒壁间的摩擦力不断增大。 B.实验时环境温度增大了。 C.实验时外界大气压强发生了变化。 D.实验时注射器内的空气向外发生了泄漏。 (2)根据你在(1)中的选择,说明为了减小误差,应采取的措施是: 18.(7 分)图 1 为某一热敏电阻(电阻值随温度 的改变而改变,且对温度很敏感)的 I—U 关系曲线图。 (1) 为了通过测量得到图 1 所示 I—U 关系的完整 曲线,在图 2 图 3 两个电路中应选择的是图 ; 简要说明理由: (电源电动势为 9V,内阻不计,滑线变阻器的阻值为 0-100Ω)。 (2)在图 4 电路中,电源电压恒为 9V,电流表读数为 70mA,定值电阻 R1=250Ω。由热 敏电阻的 I—U 关系曲线可知,热敏电阻两端的电压为 V;电阻 R2 的阻值 为 Ω。 (3)举出一个可以应用热敏电阻的例子: 。 四、(60 分)计算题。 19.(10 分)如图所示,1、2、3 为 p—V 图中一定量理想气体的 三个状态,该理想气体由状态 1 经过程 1—3—2 到达状态 2。试利用气体实验定律证明: 2 22 1 11 T Vp T Vp  20.(10 分)如图所示,一高度为 h=0.2m 的水平面在 A 点处与一 倾角为θ=30°的斜面连接,一小球以 v0=5m/s 的速度在平面上向右运动。 求小球从 A 点运动到地面所需的时间(平面与 斜面均光滑,取 g=10m/s2)。某同学对此题的 解法为:小球沿斜面运动, 则 ,sin2 1 sin 2 0 tgtvh   由此可求得落地 的时间 t。 问:你同意上述解法吗?若同意,求出所需的时间;若不同意,则说明理由并求出你 认为正确的结果。 21.(12 分)质量为 m 的飞机以水平速度 v0 飞离跑道后逐渐 上升,若飞机在此过程中水平速度保持不变,同时受到重力 和竖直向上的恒定升力(该升力由其它力的合力提供,不含 重力)。今测得当飞机在水平方向的位移为 l 时,它的上 升高 度为 h,求:(1)飞机受到的升力大小;(2)从起飞到上 升至 h 高度的过程中升力所作 的功及在高度 h 处飞机的动能。 22.(14 分)如图所示,OACO 为置于水平面内的光 滑闭合金属导轨,O、C 处分别接有短电阻丝(图中 粗线表法),R1=4Ω、R2=8Ω(导轨其它部分电阻 不计)。导轨 OAC 的形状满足方程 )3sin(2 xy  (单位:m)。磁感强度 B=0.2T 的匀强磁场方向垂直于导轨平面。一足够长的金属棒 在水平外力 F 作用下,以恒定的速率 v=5.0m/s 水平向右在导轨上从 O 点滑动到 C 点,棒与导思接触良好且始终保持与 OC 导轨 垂直,不计棒的电阻。 求:(1)外力 F 的最大值; (2) 属棒在导轨上运动时电阻丝 R1 上消耗的最大功率; (3)在滑动过程中通过金属棒的电流 I 与时间 t 的关系。 23.(14 分)为研究静电除尘,有人设计了一个盒状容器,容器侧面是绝缘的透明有机玻璃, 它的上下底面是面积 A=0.04m2 的金属板,间距 L=0.05m,当连接到 U=2500V 的高 压电源正负两极时,能在两金属板间产生一个匀强电场,如图所示,现把一定量均匀 分布的烟尘颗粒密闭在容器内,每立方米有烟尘颗粒 1013 个,假设 这些颗粒都处于静止状态,每个颗粒带电量为 q=+1.0×10-17C,质量为 m=2.0×10-15kg, 不考虑烟尘颗粒之间的相互作用和空气阻力,并忽略烟尘颗粒所受重力。 求合上电键后: (1)经过多长时间烟尘颗粒可以被全部吸附? (2) 除尘过程中电场对烟尘颗粒共做了多少功? (3)经过多长时间容器中烟尘颗粒的总动能达到最大? 2003 年全国普通高等学校招生统一考试(上海卷) 物理参考答案 一、选择题 1.A 2.ABC 3.C 4.BC 5.AD 6.B 7.BCD 8.A 二、填空题 9.α粒子散射,见图(1) 10.见图(2) 11. 2r MG 12. mr k r e mr k r e 2, 2 13.8.7×104,6613(数值在 6550 到 6650 范围内均可) 三、实验题 14.C 15.AC 16.1.9(1.8~2.0 均可),重,支持,0.1 可能原因 检验方法 答 转轴摩擦力太 大 安装力矩盘后,轻轻转动盘面,如果盘面转动很快停止,说明 摩擦太大。 或 力矩盘重心 没有在中心 安装力矩盘后,在盘的最低端做一个标志,轻轻转动盘面,如 果标志始终停留在最低端,说明重心在这个标志和中心之间。 17.(1)D (2)在注射器活塞上涂上润滑油增加密封性 18.(1)2;图 2 电路电压可从 0V 调到所需电压,调节范围较大。(或图 3 电路不能测得 0V 附近的数据) (2)5.2;111.8(111.6-112.0 均给分) (3)热敏温度计(提出其他实例,只要合理均 给分) 四、计算题 19.设状态 3 的温度为 T 1—3 为等压过程 T V T V 2 1 1  ① 3—2 为等容过程 2 21 T p T p  ② 消去 T 即得 2 22 1 11 T Vp T Vp  ③ 20.不同意。小球应在 A 点离开平面做平抛运动,而不是沿斜面下滑。正确做法为:落地 点与 A 点的水平距离 )(110 2.0252 00 mg hvtvs  ① 斜面底宽 )(35.032.0 mhctgl   ② ls  小球离开 A 点后不会落到斜面,因此落地时间即为平抛运动时间。 ∴ )(2.010 2.022 sg ht  ③ 21.(1)飞机水平速度不变 tvl 0 ① y 方向加速度恒定 2 2 1 ath  ② 消去 t 即得 2 02 2 vl ha  ③ 由牛顿第二定律 )21( 2 02 vgl hmgmamgF  ④ (2)升力做功 )21( 2 02 vgl hmghFhW  ⑤ 在 h 处 l hvahatvt 022  ⑥ ∴ )41(2 1)(2 1 2 2 2 0 22 0 l hmvvvmE tk  ⑦ 22.(1)金属棒匀速运动 安外 FF  BLv ① I=ε/R 总 ② F 外=BIL=B2L2v/R 总 ③ )(22sin2max mL   ④ )(3/8 21 21  RR RRR总 ⑤ ∴ )(3.08/30.522.0 22 max NF  ⑥ (2) )(14/0.522.0// 222 1 222 1 2 1 WRvLBRP   ⑦ (3)金属棒与导轨接触点间的长度随时间变化 ))(3sin(2 mxL  且 ,vtx  BLv , ∴ ))(3 5sin(4 3)3sin(2 AtvtR Bv RI   总总 ⑧ 23.(1)当最靠近上表面的烟尘颗粒被吸附到下板时,烟尘就被全部吸附烟尘颗粒受到的电 场力 F=qU/L ① mL qUtatL 22 1 2 2  ② ∴ )(02.02 sLqU mt  ③ 2) NALqUW 2 1 ④ =2.5×10-4(J) ⑤ (3)设烟尘颗粒下落距离为 x )()(2 1 2 xLNAxL qUxLNAmvEk  ⑥ 当 2 Lx  时 EK 达最大, 2 12 1 atx  )(014.02 1 sLqU m a xt  ⑦ 2004 年普通高等学校招生全国统一考试(湖南、湖北卷) 理科综合能力测试 本试卷分第Ⅰ卷(选择题)和第Ⅱ卷(非选择题)两部分,第Ⅰ卷 1 至 4 页,第Ⅰ卷 5 至 10 页,满分 300 分,考试用时 150 分钟,考试结束后,将本试卷和答题卡一并交回。 第Ⅰ卷 注意事项: 1. 答卷前,考生务必将自己的姓名、准考证号写在试题卷和等题卡上、并将准考证号 条形码粘贴在答题卡上指定位置。 2. 每小题选出答案后,用 2B 铅笔把答题卡上对应题目的答案标号涂黑,如需改动,用 橡皮擦干净后,再选涂其它答案标号, 3. 本卷共 21 题,每题 6 分,共 126 分,在每题给出的四个选项中,只有一项是符合题 目要求的。 以下数据可提供解题时参考: 原子量:C 12 N 14 O 16 Na 23 Mg 24 P 31 Cl 35.5 K 39 Ca 40 Fe 56 1. 下列关于光合作用强度的叙述正确的是 A. 叶片从幼到老光合作用强度不变 B. 森林或农田中植株上部叶片和下部叶片光合作用强度有差异 C. 光合作用强度是由基因决定的,因此是固定不变的 D. 在相同光照条件下,各种植物的光合作用强度相同 2. 某生物的体细胞染色体数为 2n。该生物减数分裂的第二次分裂与有丝分裂相同之处是 A. 分裂开始前,都进行染色体的复制 B. 分裂开始时,每个细胞中的染色体数都是 2n C. 分裂过程中,每条染色体的着丝点都分裂成两个 D. 分裂结束后,每个子细胞的染色体数都是 n 3. 用一定量的甲状腺激素连续饲喂正常成年小白鼠 4 周,与对照组比较,实验组小白鼠表 现为 A. 耗氧量增加、神经系统的兴奋性降低 B. 耗氧量增加、神经系统的兴奋性增强 C. 耗氧量减少、神经系统的兴奋性降低 D. 耗氧量减少、神经系统的兴奋性增强 4. 下列属于生态系统食物网特征的是 A. 一种生物只能被另一种生物捕食 B. 食物链的环节数是无限的 C. 一种生物可能属于不同的营养级 D. 食物网上的生物之间都是捕食关系 5. 用动物细胞工程技术获取单克隆抗体,下列实验步骤中错误..的是 A. 将抗原注入小白鼠体内,获得能产生抗体的 B 淋巴细胞 B. 用纤维素酶处理 B 淋巴细胞与小白鼠骨髓瘤细胞 C. 用聚乙二醇作诱导剂,促使能产生抗体的 B 淋巴细胞与小白鼠骨髓瘤细胞融会 D. 筛选杂交瘤细胞,并从中选出能产生所需抗体的细胞群,培养后提取单克隆抗体 6. 在 pH=1 含 Ba2+离子的溶液中,还能大量存在的离子是 A. AlO  2 B. ClO— C. Cl— D. SO 2 4 7. 物质的量浓度相同的下列溶液中,符合按 pH 由小到大顺序排列的是 A. Na2CO3 NaHCO3 NaCl NH4Cl B. Na2CO3 NaHCO NH4Cl NaCl C. (NH4)2SO4 NH4Cl NaNO3 Na2S D. NH4Cl (NH4)2SO4 Na2S NaNO3 8. 已知 ⑴ H2(g)+ 2 1 O2(g)==H2O(g) △H1= a kJ·mol-1 ⑵ 2H2(g)+ O2(g)==2H2O(g) △H2= b kJ·mol-1 ⑶ H2(g)+ 2 1 O2(g)==H2O(l) △H3= c kJ·mol-1 ⑷ 2H2(g)+ O2(g)==2H2O(l) △H4= d kJ·mol-1 下列关系式正确的是 A. ad>0 C. 2a=b<0 D. 2c=d>0 9. 将 0.1 mol·L-1 醋酸溶液加水稀释,下列说法正确的是体 A. 溶液中 c(H+)和 c(OH—)都减小 B. 溶液中 c(H+)增大 C. 醋酸电离平衡向左移动 D. 溶液的 PH 值增大 10. 下列叙述正确的是 a b xo A. 同温同压下,相同体积的物质,它们的物质的量必相等 B. 任何条件下,等物质的量的乙烯和一氧化碳所含分子数必相等 C. 1L 一氧化碳气体一定比 1L 氧气的质量小 D. 等体积、等物质的量浓度的强酸中所含的 H+数一定相同 11. 若 1 mol 某气态烃 CxHy,完全燃烧,需用 3 mol O2,则 A. x=2,y=2 B. x=2,y=4 C. x=3,y=6 D. x=3,y=8 12. 下列分子中,所有原子不可能...共处在同一平面上的是 A. C2H2 B. C2S C. NH3 D. C6H6 13. 常温下,下列各组物质不能用同一种试剂通过化学反应区别的是 A. MnO2 CuO FeO B. (NH4)2SO4 K2SO4 NH4Cl C. AgNO3 KNO3 Na2CO3 D. Na2CO3 NaHCO3 KNO3 14. 现有 1200 个氢原子被激发到量子数为 4 的能级上,若这些受激氢原子最后都回到基态, 则在此过程中发出的光子总数是多少?假定处在量子数为 n 的激发态的氢原子跃迁到 各较低能级的原子数都是处在该激发态能级上的原子总数的 1 1 n 。 A. 2200 B. 2000 C. 1200 D. 2400 15. 下面是四种与光有关的事实: ① 用光导纤维传播信号 ② 用透明的标准样板和单色光检查平面的平整度 ③ 一束白光通过三棱镜形成彩色光带 ④ 水面上的油膜呈现彩色 其中,与光的干涉有关的是 A. ①④ B. ②④ C. ①③ D. ②③ 16. 一定量的气体吸收热量,体积膨胀并对外做功,则此过程的末态与初态相比 A. 气体内能一定增加 B. 气体内能一定减小 C. 气体内能一定不变 D. 气体内能是增是减不能确定 17. 如图,一简谐横波在 x 轴上传播,轴上 a、b 两点相距 12m。t=0 时 a 点为波峰,b 点为 波谷;t=0.5s 时,a 点为波谷,b 点为波峰。则下列判断正确的是 A. 波一定沿 x 轴正方向传播 B. 波长可能是 8m C. 周期可能是 0.5s D. 波速一定是 24m/s 18. 如图所示,四个完全相同的弹簧都处于水平位置,它们的右端受到大小皆为 F 的拉力作 用,而左端的情况各不相同:①中弹簧的左端固定在墙上,②中弹簧的左端受大小也为 F 的拉力作用,③中弹簧的左端拴一小物块,物块在光滑的桌面上滑动,④中弹簧的左 端拴一小物块,物块在有摩擦的桌面上滑动。若认为弹簧的质量都为零,以 l1、l2、l3、 l4 依次表示四个弹簧的伸长量,则有 ① FF ② F ③ ④F F A. l2>l1 B. l4>l3 C. l1>l3 D. l2=l4 19. 一直升飞机停在南半球的地磁极上空。该处地磁场的方向竖直向上,磁感应强度为 B。 直升飞机螺旋桨叶片的长度为 l,螺旋桨转动的频率为 f,顺着地磁场的方向看螺旋桨, 螺旋桨按顺时针方向转动。螺旋桨叶片的近轴端为 a,远轴端为 b,如图所示。忽略 a 到转轴中心线的距离,用ε表示每个叶片中的感应电动势,则 A. ε=πfl2B,且 a 点电势低于 b 点电势 B. ε=2πfl2B,且 a 点电势低于 b 点电势 C. ε=πfl2B,且 a 点电势高于 b 点电势 D. ε=2πfl2B,且 a 点电势高于 b 点电势 20. 如图一绝缘细杆的两端各固定着一个小球,两小球带有等量异号的电荷,处于匀强电场 中,电场方向如图中箭头所示。开始时,细杆与电场方向垂直, 即在图中Ⅰ所示的位置;接着使细杆绕其中心转过 90°,到达 图中Ⅱ所示的位置;最后使细杆移到图中Ⅲ所示的位置。以 W1 表示细杆由位置Ⅰ到位置Ⅱ过程中电场力对两小球所做的 功,W2 表示细杆由位置Ⅱ到位置Ⅲ过程中电场力对两小球所做 的功,则有 A. W1=0,W2≠0 B. W1=0,W2≠0 C. W1≠0,W2=0 D. W1≠0,W2≠0 21. 放在水平地面上的一物块,受到方向不变的水平推力 F 的 作用,F 的大小与时间 t 的关系和物块速度 v 与时间 t 的关 系如图所示。取重力加速度 g=10m/s2。由此两图线可以求 得物块的质量 m 和物块与地面之间的动摩擦因数μ分别为 A. m=0.5kg,μ=0.4 B. m=1.5kg,μ= 15 2 C. m=0.5kg,μ=0.2 D. m=1kg,μ=0.2 2004 年普通高等学校招生全国统一考试(湖南、湖北卷) 理科综合能力测试 第Ⅱ卷 (非选择题 共 10 题 共 174 分) 注意事项: 1. 用钢笔或圆珠笔直接答在试题卷中(除题目有特殊规定外)。 2. 答卷前将密封线内的项目填写清楚。 题号 22 23 24 25 26 27 28 29 30 31 32 总分 分数 22.(18 分)用以下器材测量一待测电阻 Rx 的阻值(900~1000Ω): 电源 E,具有一定内阻,电动势约为 9.0V; 电压表 V1,量程为 1.5V,内阻 r1=750Ω; 电压表 V2,量程为 5V,内阻 r2=2500Ω; 滑线变阻器 R,最大阻值约为 100Ω; 单刀单掷开关 K,导线若干。 ⑴测量中要求电压表的读数不小于其量程的 3 1 ,试画 出测量电阻 Rx 的一种实验电路原理图(原理图中的元件要 用题图中相应的英文字母标注)。 ⑵根据你所画的电路原理图在题给的实物图上画出联线。 ⑶若电压表 V1 的读数用 U1 表示,电压表 V2 的读数用 U2 表示,则由已知量和测得量 表示 Rx 的公式为 Rx=________。 23.(16 分)一水平放置的水管,距地面高 h=1.8m,管内截面积 S=2.0cm2。有水从管口以不 变的速度 v=2.0m/s 源源不断地沿水平方向射出,设水流稳定后在空中有多少立方米的 水。 24.(18 分) 如图所示,在 y>0 的空间中存在匀强电场,场强沿 y 轴负 方向;在 y<0 的空间中,存在匀强磁场,磁场方向垂直 xy 平面 (纸面)向外。一电量为 q、质量为 m 的带正电的运动粒子,经过 y 轴上 y=h 处的点 P1 时 速率为 v0,方向沿 x 轴正方向;然后,经过 x 轴上 y=2h 处的 P2 点进入磁场,并经过 y 轴上 y=-2h 处的 P3 点。不计重力。求 ⑴电场强度的大小。 ⑵粒子到达 P2 时速度的大小和方向。 ⑶磁感应强度的大小。 25.(20 分) 柴油打桩机的重锤由气缸、活塞等若干部件组成,气缸与活塞 间有柴油与空气的混合物。在重锤与桩碰撞的过程中,通过压缩使 混合物燃烧,产生高温高压气体,从而使桩向下运动,锤向上运动。 现把柴油打桩机和打桩过程简化如下: 柴油打桩机重锤的质量为 m,锤在桩帽以上高度为 h 处(如图 1)从静止开始沿竖直轨道自由落下,打在质量为 M(包括桩帽) 的钢筋混凝土桩子上。同时,柴油燃烧,产生猛烈的推力,锤和桩 分离,这一过程的时间极短。随后,桩在泥土中向下移动一距离 l。 已知锤反跳后到达最高点时,锤与已停下的桩帽之间的距离也为 h (如图 2)。已知 m=1.0×103kg,M=2.0×103kg,h=2.0m,l=0.20m, 重力加速度 g=10m/s2,混合物的质量不计。设桩向下移动的过程泥土对桩的作用力 F 是恒 力,求此力的大小。 26.(16 分) 粉末状试样 A 是由等物质的量的 MgO 和 Fe2O3 组成的混合物。进行如下实验: ①取适量 A 进行铝热反应,产物中有单质 B 生成; ②另取 20g A 全部溶于 0.15L 6.0mol·L-1 盐酸中,得溶液 C; ③将①中得到的单质 B 和溶液 C 反应,放出 1.12L(标况)气体,同时生成溶液 D,还 残留有固体物质 B; ④用 KSCN 溶液检验时,溶液 D 不变色。 请填空: ⑴①中引发铝热反应的实验操作是______________,产物中的单质 B 是_____________。 ⑵②中所发生的各反应的化学方程式是_______________________。 ⑶③中所发生的各反应的离子方程式是_______________________。 ⑷若溶液 D 的体积仍视为 0.15L,则该溶液中 c(Mg2+)为____________。 c(Fe2+)为____________。 27.(14 分) 科学家发现某药物 M 能治疗心血管疾病是因为它在人体内能释放出一种“信使分子” D,并阐明了 D 在人体内的作用原理,为此他们荣获了 1998 年诺贝尔生理学或医学奖。 请回答下列问题: ⑴已知 M 的分子量为 227,由 C、H、O、N 四种元素组成,C、H、N 的质量分数依次 为 15.86%、2.20%和 18.50%。则 M 的分子式是__________________。D 是双原子分子,分 子量为 30,则 D 的分子式为_________________。 ⑵油脂 A 经下列途径可得到 M。 反应①的化学方程式是_________________________。 反应②的化学方程式是_________________________。 ⑶C 是 B 和乙酸在一定条件下反应生成是化合物,分子量为 134,写出 C 所有可能的 结构简式______________________________________________________________________。 ⑷若将 0.1mol B 与足量的金属反应,则需消耗______________g 金属钠。 28.(14 分) 在玻璃圆筒盛有两种无色的互不相溶的中性液体。上层液体插入两 根石墨电极,圆筒内还放有一根下端弯成环状的玻璃搅棒,可以上下搅 动液体,装置如右图。接通电源,阳极周围的液体呈现棕色,且颜色由 浅变深,阴极上有气泡生成。停止通电,取出电极,用搅棒上下剧烈搅 动。静置后液体又分成两层,下层液体呈紫红色,上层液体几乎无色。 根据上述实验回答: ⑴阳极上的电极反应式为__________________________________。 ⑵阴极上的电极反应式为__________________________________。 ⑶原上层液体是__________________________________。 ⑷原下层液体是__________________________________。 ⑸ 搅 拌 后 两 层 液 体 颜 色 发 生 变 化 的 原 因 是 ______________________________________ __________________________________________________________________________ 。 ⑹要检验上层液体中含有的金属离子,其方法是________________________________, 现 象 是 ____________________________________________________________________。 29.(16 分) O || 抗击“非典”期间,过氧乙酸(CH3C——O——OH)是广为使用的消毒剂。它可由 H2O 和冰醋酸反应制取,所以在过氧乙酸中常含有残留的 H2O。测定产品中过氧乙酸浓度 c0 涉 及下列反应: ⑴ □ MnO  4 + □H2O2 +□ H+ === □ Mn2+ + □ O2 + □ H2O ⑵ H2O2 + 2I- + 2 H+ === I2 + 2 H2O O || ⑶ CH3C——O——OH + 2I- + 2 H+ ===CH3COOH + I2 + 2 H2O ⑷ I2 + 2S2O 2 3 ==== S4O 2 6 + 2I- 请回答以下问题: ⑴配平反应①的离子方程式(配平系数填入以下方框内): □ MnO  4 + □H2O2 +□ H+ === □ Mn2+ + □ O2 + □ H2O ⑵用 Na2S2O3 标准溶液滴定 I2 时(反应④)选用的指示剂是_____________________。 ⑶取 b0 mL 待测液,用硫酸使溶液酸化,在用浓度为 a1 mol·L-1 的 KMnO4 标准溶液滴 定其中的 H2O2,耗用的 KMnO4 体积为 b1 mL(反应①,滴定过程中 KMnO4 不与过氧乙酸 反应) 另取 b0 mL 待测液,加入过量的 KI,并用硫酸使溶液酸化,此时过氧乙酸和残留的 H2O2 都能跟 KI 反应生成 I2(反应②和③)。再用浓度为 a2 mol·L-1 的 Na2S2O3 标准溶液滴定生成 的 I2,耗用 Na2S2O3 溶液体积为 b2 mL。 请根据上述实验数据计算过氧乙酸的浓度(用含 a1、a2、b0、b1、b2 的代数示表示)。 c0=______________________。 ⑷为计算待测液中过氧乙酸的浓度 c0,加入的 KI 的质量已过量但没有准确称量,是否 影响测定结果_______________(填是或否)。 30.(22 分) 试回答下列⑴——⑵题。 ⑴在一些性状的遗传中,具有某种基因型的合子不能完成胚胎发育,导致后代中不存 在该基因型的个体,从而使性状的分离比例发生变化。小鼠毛色的遗传就是一个例子。 一个研究小组,经大量重复实验,在小鼠毛色的遗传的研究中发现: A. 黑色鼠与黑色鼠杂交,后代全部为黑色鼠。 B. 黄色鼠与黄色鼠杂交,后代中黄色鼠与黑色鼠发比例为 2:1 C. 黄色鼠与黄色鼠杂交,后代中黄色鼠与黑色鼠发比例为 1:1 根据上述实验结果,回答下列问题:(控制毛色的显性基因用 A 表示,隐性基因用 a 表示) ①黄色鼠的基因型是_____________,黑色鼠发基因型是_________________。 ②推测不能完成胚胎发育的合子的基因型是____________________。 ③写出上述 B、C 两个杂交组合的遗传图解。 ⑵回答下列问题: ①真核生物基因的编码区中能够编码蛋白质的序列称为__________,不能够编码蛋白 质的序列称为_____________。 ②一般来说,如果知道了某真核生物的一条多肽链的氨基酸序列,你能否确定其基因 编码区的 DNA 序列?为什么? 31.(22 分) 胰高血糖素对小白鼠和人具有相同的生理作用。为了验证“胰高血糖素具有升温血糖 的生理作用”,请以小白鼠为实验对象设计实验步骤,预测和解释实验应出现的结果,并写 出实验结论。 (一)实验材料和用具: 正常实验小白鼠 2 只,生理盐水,用生理盐水配制的适宜浓度的胰高血糖素溶液,班 氏糖定性试剂,注射器,试管,烧杯等。 (二)实验步骤: (实验提示:采用腹腔注射药,给药剂量不作实验设计要求;给药 1 小时后,用注射 器在小白鼠膀胱处穿刺取尿液。) (三)实验结果的预测、解释和结论: 参考答案 1—10 B C B C B C C C D B 11—21 B C D A B D B D A C A 22.(18 分)(1) (2) (1) Rx= 2112 211 rUrU rrU  或 Rx= 1 1 12 )( rU UU  23.(16 分) 以 t 表示水由喷口处到落地所用的时间,有 h= 2 1 gt2 ① 单位时间内喷出的水量为 Q=Sυ ② 空中水的总量应为 V=Qt ③ 由以上各式得 V=S·V· g h2 ④ 代入数值得 V=2.4×10-4 m3 ⑤ 24.(18 分) (1)粒子在电场、磁场中运动的轨迹如图所示。设粒子从 P1 到 P2 的时间为 t,电场度的大 小为 E,粒子在电场中的加速度为 a,由牛顿第二定律及运动学公式有 qE=ma ① υ0t=2A ② 2 1 at2=h ③ 由①、②、③式解得 E= qh mp 2 2 0 ④ (2)粒子到达 P2 时速度沿 x 方向的分量仍为υ0,以υ1 表示速度沿 y 方向分量的大小,υ 表示速度的大小,θ表示速度和 x 轴的夹角,则有 υ 2 1 =2ah ⑤ υ= 2 0 2 1   ⑥ tgθ= 0 1   ⑦ 由②、③、⑤式得 υ1=υ0 ⑧ 由⑥、⑦、 ⑧式得 υ= 02 ⑨ θ=45° ⑩ (3)设磁场的磁感应强度为 B,在洛仑兹力作用下粒子做匀速圆周运动,由牛顿第二定律 qυB=m r 2 ○11 r 是圆周的半径。此圆周与 x 轴和 y 轴的交点分别为 P2、P3。因为 OP2=OP3,θ=45°,由 几何关系可知,连线 P2P3 为圆轨道的直径,由此可求得 r= h2 ○12 由⑨、○11 、○12 可得 B= qh m 0 ○13 25.(20 分) 锤自由下落,碰桩前速度υ1 向下, υ1= gh2 ① 碰后,已知锤上升高度为(h-l),故刚碰后向上的速度为 υ2= )(2 lhg  ② 设碰后桩的速度为 V,方向向下,由动量守恒, mυ1=MV-mυ2 ③ 桩下降的过程中,根据功能关系, 2 1 MV2+Mgl=Fl ④ 由①、②、③、④或得 F=Mg+ ])(22)[( lhhlhM m l mg  ⑤ 代入数值,得 F=2.1×105N ⑥ 26.(16 分)(每空 2 分,共 16 分) (1)加少量 KClO3,插上 Mg 条并将其点燃 Fe (2)Fe2O3+6HCl====2FeCl3+3H2O MgO+2HCl====MgCl2+H2O (3)Fe+2H+====Fe2++H2 Fe+2Fe3+====3Fe2+ (4)c(Mg2+)====0.67 mol ·L-1 c(Fe2+)====2.3 mol·L-1 27。(14 分) (1)C3H5O9N3 NO 28。(14 分) (1)2I――2e―=====I2 (2 ) 2H++2e―====H2 (3 )KI(或 NaI 等)水溶液 (4 )CCl4(或 CHCl3 等) (5 )I2 在 CCl4 中的溶解度大于在水中的溶解度,所以绝大部分 I2 都转移到 CCl4 中 (6 )焰色反应 透过蓝色钴玻璃观察火焰呈柴紫色(其它合理答案同样给分。例如,若③ 中答 NaI 水溶液,这里答火焰呈黄色。) 29。(16 分) (1)2 5 6 2 5 8 (2 )淀粉溶液 (3 ) ob baba 2 5 2222  (4 )否 30。(22 分) (1) ①Aa aa ②AA ③B : A a × A a 黄色 黄色 IAA : 2Aa : Iaa 不存活 黄色 黑色 C: A a × aa 黄色 黑色 IAa : Iaa 黄色 黑色 (2) ①外显子、内含子 ②不能。 首先,一种氨基酸可以有多种密码子; 其次,一般地说真核生物的基因具有 内含子。 31。(20 分) (二)实验步骤: (1)确定 1 只鼠为实验鼠,腹腔注射胰高血糖素溶液;另一只鼠为对照鼠,腹腔注射等容 量生理盐水。 (2)将两支试管分别编号为 1 号和 2 号,各加入等量的班氏糖定性试剂。 (3)给药 1 小时后,对两只小白鼠采尿液,实验鼠尿液放入 1 号试管内,对照鼠尿液放入 2 号试管内。 (4)将两支试管摇匀后,放入盛有开水的烧杯内加热煮沸,待冷却后,观察两支试管溶液 颜色的变化。 (三)实验结果的预测、解释和结论: 1 号试管中应该出现砖红色沉淀,表明实验鼠尿液中有葡萄糖;2 号试管中仍为蓝色溶液, 表明对照鼠尿液中无葡萄糖。 实验结论:实验鼠血糖升高,超过一定数值而出现糖尿,是胰高血糖素具有升高血糖的生 理作用所引起的。 2004 年普通高等学校招生全国考试统一考试 全国卷Ⅰ (物理部分) 第Ⅰ卷(选择题 共 126 分) 本卷共 21 题,每题 6 分,共 126 分。 14.本题中用大写字母代表原子核。E 经α衰变成为 F,再经β衰变成为 G,再经α衰变成 为 H。上述系列衰变可记为下式: 另一系列衰变如下: 已知 P 是 F 的同位素,则 A.Q 是 G 的同位素,R 是 H 的同位素 B.R 是 E 的同位素,S 是 F 的同位素 C.R 是 G 的同位素,S 是 H 的同位素 D.Q 是 E 的同位素,R 是 F 的同位素 15.如图所示,ad、bd、cd 是竖直面内三根固定的光滑细杆, a 、 b、c、d 位于同一圆周上, a 点为圆周的最高点,d 点为 最 低点。每根杆上都套着一个小滑环(图中未画出),三个 滑 环分别从 a、b、c 处释放(初速为 0),用 t1、、、t2、、t3 依 次 表示各滑环到达 d 所用的时间,则 A.t1 、t2、>t3 C.t3 > t1、>t2、 D.t1=、t2、=t3 16.若以μ表示水的摩尔质量,υ表示在标准状态下水蒸气的摩尔体积,ρ为在标准状态下 a b c d β α β β α α F E G H S R Q P 水蒸气的密度,NA 为阿佛加德罗常数,m、Δ分别表示每个水分子的质量和体积,下面是 四个关系式: ① NA= υρ m ② ρ= μ NAΔ ③ m = μ NA ④ Δ= υ NA 其中 A. ①和②都是正确的 B.①和③都是正确的 C.③和④都是正确的 D. ①和④都是正确的 17.一列简谐横波沿 x 轴负方向传播,图 1 是 t =1s 时的波形图,图 2 是波中某振动质元位 移随时间变化的振动图线(两图用同一时间起点),则图 2 可能是图 1 中哪个质元的振动 图线? A.x=0 处的质元 B.x=1m 处的质元 C.x=2m 处的质元 D. x=3m 处的质元 18.图中电阻 R1、R2、R3 的阻值相等,电池的内阻 不 计。开关 K 接通后流过 R2 的电流是 K 接通前的 A.1 2 B.2 3 C.1 3 D. 1 4 19.下表给出了一些金属材料的逸出功。 材料 铯 钙 镁 铍 钛 逸出功(10—19J) 3.0 4.3 5.9 6.2 6.6 现用波长为 400 nm 的单色光照射上述材料,能产生光电效应的材料最多有几种?(普朗 克常量 h=6.6×10-34J·s , 光速 c=3.0×10 8m/s) A.2 种 B.3 种 C.4 种 D.5 种 20.下列哪个说法是正确的? A.游泳运动员仰卧在水面静止不动时处于失重状态 B.蹦床运动员在空中上升和下落过程中都处于失重状态 C.举重运动员在举起杠铃后不动的那段时间内处于超重状态 D.体操运动员双手握住单杠吊在空中不动时处于失重状态 21.发出白光的细线光源 ab,长度为 l0 ,竖直放置,上端 a 恰好在水面以下,如图。现考 虑线光源 ab 发出的靠近水面法线(图中的虚线)的细 光束经水 面折射后所成的像,由于水对光有色散作用,若以 l1 表 示红光成 的像的长度,l2 表示蓝光成的像的长度,则 A. l1< l2< l0 B. l1> l2> l0 C. l2 > l1> l0 D. l2 < l1< l0 t/s 0 1 2 3 4 5 6 y/m 图 2 x/m 0 1 2 3 4 5 6 y/m 图 1 R1 R2 R3 K a b 水 第Ⅱ卷(非选择题) 本卷共 10 题,共 174 分。 22.(18 分) ⑴图中给出的是用螺旋测微器测量一金属薄板厚度时的示数,此读数应为 mm ⑵实验室内有一电压表mV ,量程为 150mV 内阻约为 150Ω 。现要将其改装成量程为 10mA 的电流表,并进行校准。为此,实验室提供如下器材:干电池 E(电动势为 1.5V ),电阻 箱 R,滑线变阻器 Rˊ,电流表 A (有 1.5mA , 15mA 与 150mA 三个量程)及开关 K。 (a)对电流表改装时必须知道电压表的内阻。可用图示的电路测量电压表mV 的内阻。在 既不损坏仪器又能使精确度尽可能高的条件下,电路中的电流表 A 应选用的量程 是 。若合上 K,调节滑线变阻器后测得电压表的读数为 150mV,电流表 A 的 读数为 1.05mA,则电压表的内阻 RmV 为 。(取三位有效数字) (b)在对改装成的电流表进行校准时,把 A 作为标准电流表,画出对改装成的电流表进行 校准的电路原理图(滑线变阻器作限流使用),图中各元件要用题中给出的符号或字母标注。 图中电阻箱的取值是 (取三位有效数字),电流表 A 应选的量程 是 。 23.(16 分)在勇气号火星探测器着陆的最后阶段,着陆器降落到火星表面上,再经过多次弹 跳才停下来。假设着陆器第一次落到火星表面弹起后,到达最高点时高度为 h ,速度方向 是水平的,速度大小为 v0 ,求它第二次落到火星表面时速度的大小,计算时不计火星大气 阻力。已知火星的一个卫星的圆轨道的半径为 r ,周期为 T,火星可视为半径为 r0 的均匀 球体。 24.(18 分)图中 a1b1c1d1 和 a2b2c2d2 为在同一竖直平 面 内 的金属导轨,处在磁感强度 B 的匀强磁场中,磁场方 向 垂 直导轨所在平面(纸面)向里。导轨的 a1b1 段与 a2b2 段 是 竖直的,距离为 l1;c1d1 段与 c2d2 段也是竖直的,距离 为 l2。 x1y1 与 x2y2 为两根用不可伸长的绝缘轻线相连的金属 细杆, 质量分别为 m1、m2,它们都垂直于导轨并与导轨保持 光 滑 mV Rˊ K A 0 5 15 10 b1 a1 a2 x2 b2 c2 d2 y2 x1 c1 y1 d1 F 接触。两杆与导轨构成的回路的总电阻为 R。F 为作用与金属杆 x1y1 上竖直向上的恒力。已 知两杆运动到图示位置时,已匀速向上运动,求此时作用于两杆的重力的功率的大小和回路 电阻上的热功率。 25.(20 分)一小圆盘静止在桌布上,位于 一方桌的水平面的中央。桌布的一边与桌的 AB 边重合,如图。已知盘与桌布间的动摩 擦因数为μ1 ,盘与桌面间的动摩擦因数为 μ2 。现突然以恒定的加速度 a 将桌布抽离 桌面,加速度的方向是水平的且垂直于 AB 边。若圆盘最后未从桌面掉下,则加速度 a 满足的条件是什么?(以 g 表示重力加速 度) 参考答案 14.B、15.D、16.B、17.A、18.B、19.A、20.B、21.D 22.⑴6.124 ⑵(a)1.5mA 、143Ω (b)如图所示 、16.8Ω、15mA 23.答案: υ= 8π2h T2 r3 r02 +υ02 、 24.答案:P= F-(m1+m2)g B2(l2-l1)2 R(m1+m2)g Q=[F-(m1+m2)g B(l2-l1) ]2R 25.答案:a ≥ μ1+2μ2 μ2 μ1g 2004 年普通高等学校招生全国统一考试 全国卷Ⅳ(理综) 本试卷分第Ⅰ卷(选择题)和第Ⅱ卷(非选择题)两部分。 注意事项: 1.答第Ⅰ卷前,考生务必将自己的姓名、准考证号、考试科目涂写在答题卡上。 2.每小题选出答案后,用铅笔把答题卡上对应题目的答案标号涂黑。如需改动,用橡 皮擦干净后,再选涂其他答案标号,不能答在试题卷上。 3.本卷共 21 小题,每小题 6 分,共 126 分。在每小题给出的四个选项中,只有一项是 最符合题目要求的。 以下数据可供解题时参考: A a B A mV E K R/ R 原子量:H—1 C—12 O—16 S—32 Fe—56 1.某种病毒已侵入人体细胞内,机体免疫系统对该靶细胞发挥的免疫作用是 ( ) A.体液免疫 B.细胞免疫 C.自身免疫 D.非特异性免疫 2.下列关于光合作用和呼吸作用的叙述,错误的是 ( ) A.光合作用和呼吸作用都包括一系列氧化还原反应 B.光合作用和呼吸作用必须在有水的条件下进行 C.光合作用的全部反应是呼吸作用全部反应的逆转 D.光合作用和呼吸作用都是能量转化过程 3.下列关于实验的描述,正确的是 ( ) A.将在蔗糖溶液中已发生质壁分离的洋葱表皮细胞转到更高浓度的蔗糖溶液中,则发 生质壁分离复原 B.将斐林试剂加入到蔗糖溶液中,加热后出现砖红色沉淀 C.将肝脏研磨液煮沸冷却后,加入到过氧化氢溶液中立即出现大量气泡 D.将双缩脲试剂加入到蛋清稀释液中,溶液变成紫色 4.肺炎双球菌中的 S 型具有多糖类荚膜,R 型则不具有。下列叙述错误..的是 ( ) A.培养 R 型活细菌时加 S 型细菌的多糖类物质,能产生一些具荚膜的细菌 B.培养 R 型活细菌时加 S 型细菌 DNA 的完全水解产物,不能产生具荚膜的细菌 C.培养 R 型活细菌时加 S 型细菌 DNA,能产生具荚膜的细菌 D.培养 R 型活细菌时加 S 型细菌的蛋白质,不能产生具荚膜的细菌 5.一个池塘有生产者(浮游植物)、初级消费者(植食性鱼类)、次级消费者(肉食性鱼类) 和分解者(微生物)。其中生产者固定的全部能量为 a,流入初级消费者、次级消费者 和分解者的能量依次为 b、c、d,下列表述正确的是 ( ) A.a=b+d B.a>b+d C.aQ2 17.图中 M 是竖直放置的平面镜,镜离地面的距离可调节。甲、乙二人站在镜前,乙离镜 的距离为甲离镜的距离的 2 倍,如图所示。二人略错开,以便甲能看到乙的像。以 l 表 示镜的长度,h 表示乙的身高,为使甲能看到镜中乙的全身像,l 的最小值为 ( ) A. h3 1 B. h2 1 C. h4 3 D.h 18.已知:一简谐横波在某一时刻的波形图如图 所示,图中位于 a、b 两处的质元经过四分之 一周期后分别运动到 a 、b 处。某人据此做 出如下判断:①可知波的周期,②可知波的 传播速度,③可知的波的传播方向,④可知 波的波长。其中正确的是( ) A.①和④ B.②和④ C.③和④ D.②和③ 19.如图,在倾角为 的固定光滑斜面上,有一用绳子拴着的长木板,木板上站着一只猫。 已知木板的质量是猫的质量的 2 倍。当绳子突然断开时,猫立即沿着板向上跑,以保持 其相对斜面的位置不变。则此时木板沿斜面下滑的加速度为 ( ) A. sin2 g B. sing C. sin2 3 g D.2 sing 20.如图所示,轻杆的一端有一个小球,另一端有光滑的固定轴 O。现给球一初速度,使球 和杆一起绕 O 轴在竖直面内转动,不计空气阻力,用 F 表 示球到达最高点时杆对小球的作用力,则 F( ) A.一定是拉力 B.一定是推力 C.一定等于 0 D.可能是拉力,可能是推力,也可能等于 0 21.一平行板电容器的电容为 C,两板间的距离为 d,上板带正电,电量为 Q,下板带负电, 电量也为 Q,它们产生的电场在很远处的电势为零。两个带异号电荷的小球用一绝缘刚 性杆相连,小球的电量都为 q,杆长为 l,且 lmp+mn D.mpF2。试求在两个物块运动过程中轻线的拉 力 T。 24.(22 分) 空间中存在方向垂直于纸面向里的匀强磁场,磁感应强度为 B,一带电量为+q、质量为 m 的粒子,在 p 点以某一初速开始运动,初速方向在图中纸面内如图中 P 点箭头所示。该 粒子运动到图中 Q 点时速度方向与 P 点时速度方向垂直,如图中 Q 点箭头所示。已知 P、Q 间的距离为 l。若保持粒子在 P 点时的速度不变,而将匀强磁场换成匀强电场,电场方向与 纸面平行且与粒子在 P 点时速度方向垂直,在此电场作用下粒子也由 P 点运动到 Q 点。不 计重力。求: (1)电场强度的大小。 (2)两种情况中粒子由 P 运动到 Q 点所经历的时间之差。 25.(22 分) 如图所示,在一光滑的水平面上有两块相同的木板 B 和 C。重物(A 视质点)位于 B 的右端,A、B、C 的质量相等。现 A 和 B 以同一速度滑向静止的 C,B 与 C 发生慧碰。碰 后 B 和 C 粘在一起运动,A 在 C 上滑行,A 与 C 有摩擦力。已知 A 滑到 C 的右端面未掉下。 试问:从 B、C 发生正碰到 A 刚移动到 C 右端期间,C 所走过的距离是 C 板长度的多少倍? 26.(18 分) 下面图表示制备无水盐 E 的主要步骤: 已知 B 是石灰石,D 盐含有约 49%的结晶水,无水盐 E 可用作干燥剂。取少量 E 溶于 水,向其中滴加硝酸后,再滴加硝酸银溶液,有白色沉淀 F 生成。 填写以下空白(写出化学式或名称): A ,C ,D , E ,F 。 27.(18 分) 芳香化合物 A、B 互为同分异构体,B 的结构筒式是 。A 经①、 ②两步反应得 C、D 和 E。B 经①、②两步反应得 E、F 和 H。上述反应过程、产物性质及 相互关系如图所示。 (提示: (1)写出 E 的结构简式 。 (2)A 有 2 种可能的结构,写出相应的结构简式 。 (3)F 和小粒金属钠反应的化学方程式是 , 实验现象是 ,反应类型是 。 (4)写出 F 在浓 H2SO4 作用下在 170℃发生反应的化学方程式: 实验现象是 ,反应类型是 。 (5)写出 F 与 H 在加热和浓 H2SO4 催化作用下发生反应的化学方程式 , 实验现象是 ,反应类型 是 。 (6)在 B、C、D、F、G、I 化合物中,互为同系物的是 。 28.(14 分) 根据下图及描述,回答下列问题: (1)关闭图 A 装置中的止水夹 a 后,从长颈漏斗向试管中注入一定量的水,静置后如图所 示。试判断:A 装置是否漏气?(填“漏气”、“不漏气”或“无法确 定”) , 判断理由: 。 (2)关闭图 B 装置中的止水夹 a 后,开启活塞 b,水不断往下滴,直至企部流入烧瓶。试 判断:B 装置是否漏气?(填“漏气”、“不漏气”或“无法确定”) , 判断理由: 。 29.(16 分) A、B、C、D、E 分别代表 5 种微粒,每种微粒中都含有 18 个电子,其中 A 和 C 都是 由单原子形成的阴离子,B、D 和 E 都是分子;又知在水溶液中 A 跟 B 反应可生成 C 和 D;E 具有强氧化性。请回答: (1)用化学符号表示上述 5 种微粒: A ,B ,C ,D ,E 。 (2)在水溶液中 A 跟 B 反应的离子方程式是: 。 30.(13 分) 右图示意的是人体内物质代谢与能量代谢的关系(部分)。 试按图中标号填写有关内容。 ① ② ③ ④ ⑤ ⑥ ⑦ ⑧ ⑨ ⑩ ○11 ○12 ○13 注:①⑤⑥⑦⑨表示过程, ②③④⑧⑩表示物质, ○11 表示能量, ○12 和○13 为高能化合物。 31.(17 分) 在一个透明的容器中加入适量 NaHCO3 稀溶液,将杨树叶片迅速封入其中,装置如图所 示,摇动容器,使容器内空气中的 CO2 和溶液中的 CO2 达到动态平衡,在保持温度不 变的条件下,进行如下实验,试根据实验回答下列问题: (1)光照几分钟后,容器内溶液的 pH (增大、减小),其原因是 。 (2)随着光照时间的延长,溶液 pH 的变化速度 趋于变(快、慢),其原因是 。 (3)若将装置置于暗室中,一段时间后,溶液的 pH (增大、减小),其原因是 。 (4)该装置可以用来研究植物的 和 。 2004 年普通高等学校招生全国统一考试 全国卷Ⅲ(理综) (老课程)答案 1—10 ABCDBCAADB 11—21 CCCCDDDBADB Ⅱ卷包括 10 小题,共 174 分。 22.(20 分) (1)54.14 (2)①将 K2 闭合,K2 断开,记下电压表读数 U1。 ②K1、K2 均闭合,记下电压表读数 U2。 结果:电源电动势 E=U1 内阻 r= RU UU 2 21  23.设两物块一起运动的加速度为 a,则有 F1-F2=(m1+m2)a ① 根据牛顿第二定律,对质量为 m1 的物块有 F1-T=m1a ② 由①、②两式得 21 1221 mm FmFmT   ③ 24.(22 分) (1)粒子在磁场中做匀速圆周运动,以 v0 表示粒子在 P 点的初速度,R 表示圆周的半 径,则有 qv0B=m R v 2 0 ① 由于粒子在 Q 点的速度垂直它在 p 点时的速度,可知粒子由 P 点到 Q 点的轨迹为 4 1 圆 周,故有 2 lR  ② 以 E 表示电场强度的大小,a 表示粒子在电场中加速度的大小,tE 表示粒子在电场中由 p 点运动到 Q 点经过的时间,则有 qE=ma ③ 2 2 1 EatR  ④ R=v0tE ⑤ 由以上各式,得 m qlBE 2 2 ⑥ (2)因粒子在磁场中由 P 点运动到 Q 点的轨迹为 4 1 圆周,故运动经历的时间 tE 为圆周 运动周期 T 的 4 1 ,即有 tE= 4 1 T ⑦ 而 0 2 v RT  ⑧ 由⑦⑧和①式得 qB mtE 2  ⑨ 由①⑤ 两式得 qB mtE  ⑩ qB mtt ER )12(   ○11 25.(22 分) 设 A、B、C 的质量均为 m。碰撞前,A 与 B 的共同速度为 v0,碰撞后 B 与 C 的共同 速度为 v1。对 B、C,由动量守恒定律得 mv0=2mv1 ① 设 A 滑至 C 的右端时,三者的共同速度为 v2。对 A、B、C,由动量守恒定律得 2mv0=3mv2 ② 设 A 与 C 的动摩擦因数为μ,从发生碰撞到 A 移至 C 的右端时 C 所走过的距离为 s, 对 B、C 由功能关系 2 1 2 2 )2(2 1)2(2 1 vmvmmgs  ③ 设 C 的长度为 l,对 A,由功能关系 2 2 2 0 2 1 2 1)( mvmvlsmg  ④ 由以上各式解得 3 7 l s ⑤ 26.(18 分) A. HCl(盐酸), C.Ca(OH)2(石灰乳) D.CaCl2·6H2O, E.CaCl2, F.AgCl。 27.(18 分) 28.(14 分) (1)不漏气 由于不漏气,加水后试管内气体体积减小,导致压强增大,长颈漏斗内的水面高出试 管内的水面。 (2)无法确定 由于分液漏斗和烧瓶间有橡皮管相连,使分液漏斗中液面上方和烧瓶中液面上方的压 强相同,无论装置是否漏气,都不影响分液漏斗中的液体滴入烧瓶。 29.(16 分) (1)S2-, HCl, Cl-, H2S, F2 (2)S2-+2H+=H2S↑ 30.(13 分) ①消化(酶分解) ②蛋白质 ③肝糖元 ④肌糖元 ⑤脱氨基 ⑥⑨有氧呼吸 ⑦无氧呼吸 ⑧尿素 ⑩脂肪 ○11 热能 ○12 ATP ○13 磷酸肌酸 31.(17 分) (1)增大、植物光合作用消耗 CO2 引起溶液中的碳酸氢根离子减少,使得氢离子浓度 降低,溶液 pH 增大。 (2)慢、随着光照时间的延长,光合作用使溶液中的碳酸氢根离子大量消耗,导致碳酸 氢根离子转化为 CO2 的量减少,氢离子浓度降低的速度减慢,溶液 pH 的变化速度转慢。 (3)减小,在暗室叶片进行呼吸作用,释放 CO2,容器中 CO2 浓度增加,使溶液中碳 酸增加,pH 减小。 (4)光合作用 呼吸作用。 2004 年全国普通高等学校招生全国统一考试 物理(广东卷) 本试卷分选择题和非选择题两部分,共 8 页。考试用时 120 分钟 第Ⅰ卷(选择题 共 40 分) 一、本题共 10 小题,每小题 4 分,共 40 分。在每小题给出的四个选项中,有的小题只有一 个选项正确,有的小题有多个选项正确。全部选对的得 4 分,选不全的得 2 分,有选错 或不答的得 0 分。 可能用到得物理量: 真空中光速 83.0 10 /c m s  万有引力常量 11 2 26.7 10 . /G N m kg  普朗克常量 346.6 10 .h J s  电子的电量的大小 191.6 10e C  静电力常量 9 2 29.0 10 . /k N m C  1.图示为氢原子的能级图,用光子能量为 13.07eV 的光照 射一群处于基态的氢原子,可能观测到氢原子发射的不同 波长有多少种? A.15 B.10 C.4 D.1 2.下列说法哪些是正确的 A.水的体积很难被压缩,这是分子间存在斥力的 宏观表现 B.气体总是很容易充满容器,这是分子间存在斥力的 宏观表现 C.两个相同的半球壳吻合接触,中间抽成真空(马德 堡半球),用力很难拉开,这是分子间存在吸引力的 宏观表现 D.用力拉铁棒的两端,铁棒没有断,这是分子间存在吸引力的宏观表现 3.一列简谐波沿一直线向左运动,当直线上某质点 a 向上运动到达最大位移时,a 点右方 相距 0.15m 的 b 点刚好向下运动到最大位移处,则这列波的波长可能是 A.0.6m B.0.3m C.0.2m D.0.1m 4.下列说法正确的是 A.机械能全部变成内能是不可能的 B.第二类永动机不可能制造成功的原因是因为能量既不会凭空产生,也不会凭空消失, 只能从一个物体转移到另一个物体,或从一种形式转化成另一种形式。 C.根据热力学第二定律可知,热量不可能从低温物体传到高温物体 D.从单一热源吸收的热量全部变成功是可能的 5.中子 n、质子 p、氘核 D 的质量分别为 .n p Dm m m、 、 现用光子能量为 E 的γ射线照射静 止氘核使之分解,反应的方程为 1 -13.61 2 -3.40 3 -1.51 4 -0.855 -0.54 -0.386 ∞ 0 n E/eV 左 a b 右• • D p n    若分解后中子、质子的动能可视为相等,则中子的动能是 A. 21[( ) ]2 D p nm m m c E   B. 21[( ) ]2 D n pm m m c E   C. 21[( ) ]2 D p nm m m c E   D. 21[( ) ]2 D n pm m m c E   6.分别用波长为λ和 3 4  的单色光照射同一金属板,发出的光电子的最大初动能之比为 1:2, 以 h 表示普朗克常量,c 表示真空中的光速,则此金属板的逸出功为 A. 1 2 hc  B. 2 3 hc  C. 3 4 hc D. 4 5 h c  7.用三根轻绳将质量为 m 的物块悬挂在空中,如图所示. 已知 ac 和 bc 与竖直方向的夹角分别为 030 和 060 ,则 ac 绳和 bc 绳中的拉力分别为 A. 3 1,2 2mg mg B. 1 3,2 2mg mg C. 3 1,4 2mg mg D. 1 3,2 4mg mg 8.如图所示,密闭绝热的具有一定质量的活塞,活塞的上部封 闭着气体,下部为真空,活塞与器壁的摩擦忽略不计,置于真 空中的轻弹簧的一端固定于容器的底部.另一端固定在活塞 上,弹簧被压缩后用绳扎紧,此时弹簧的弹性势能为 PE (弹簧 处于自然长度时的弹性势能为零),现绳突然断开,弹簧推动 活塞向上运动,经过多次往复运动后活塞静止,气体达到平衡态,经过此 过程 A. PE 全部转换为气体的内能 B. PE 一部分转换成活塞的重力势能,其余部分仍为弹簧的弹性势能 C. PE 全部转换成活塞的重力势能和气体的内能 D. PE 一部分转换成活塞的重力势能,一部分转换为气体的内能,其余部分仍为弹簧的弹 性势能 a b c m 理想气体 9.一杂技演员,用一只手抛球.他每隔 0.40s 抛出一球,接到球便立即把球抛出,已知除抛、接 球的时刻外,空中总有四个球,将球的运动看作是竖直方向的运动,球到达的最大高度 是(高度从抛球点算起,取 210 /g m s ) A. 1.6m B. 2.4m C.3.2m D.4.0m 10.在场强为 E 的匀强电场中固定放置两个小球 1 和 2,它们的质量相等,电荷分别为 1q 和 2q ( 1 2q q )。球 1 和球 2 的连线平行于电场线,如图。现同时放开 1 球和 2 球,于 是它们开始在电力的作用下运动,如果球 1 和求之间的距离可以取任意有限值,则两球 刚被放开时,它们的加速度可能是 A.大小相等,方向相同 B.大小不等,方向相反 C.大小相等,方向相同 D.大小相等,方向相反 第Ⅱ卷(非选择题 共 110 分) 二、按题目要求作答,解答题应写出必要的文字 说明、方程式和重要步骤,只写出最后答案 的不能得分,有数值计算的题,答案中必须 明确写出数值和单位。 11.(8 分)如图,画有直角坐标系 Oxy 的白纸 位于水平桌面上,M 是放在白纸上的半圆 形玻璃砖,其底面的圆心在坐标的原点,直 边与 x 轴重合,OA 是画在纸上的直线, 1 2P P、 为竖直地插在直线 OA 上的两枚大 头针, 3P 是竖直地插在纸上的第三枚大头 针, 是直线 OA 与 y 轴正方向的夹角, о о2 1 E · · · α β A y M O x P1 P2 P3  是直线 3OP 与轴负方向的夹角,只要直 线 OA 画得合适,且 3P 的位置取得正确, 测得角 和  ,便可求得玻璃得折射率。 某学生在用上述方法测量玻璃的折射率,在他画出的直线 OA 上竖直插上了 1 2P P、 两枚 大头针,但在 y<0 的区域内,不管眼睛放在何处,都无法透过玻璃砖看到 1 2P P、 的像,他 应该采取的措施是_____________________________________________________________ _________________________________________________________________.若他已透过玻 璃砖看到了 1 2P P、 的像,确定 3P 位置的方法是____________________________________ _________________________________________.若他已正确地测得了的 、 的值,则玻璃 的折射率 n=_____________________________. 12.(12 分)图中 R 为已知电阻, xR 为待测电阻, 1K 为单刀单掷开关, 2K 为单刀双掷开关, V 为电压表(内阻 极大),E 为电源(电 阻不可忽略)。现用 图中电路测量电源 电动势 及电阻 xR (1)写出操作步骤: R Rx V a b K1 K2 E (2)由 R 及测得的量,可测得 =_____________________, xR =________________. 13.(12 分)已经证实,质子、中子都是由上夸克和下夸克的两种夸克组成的,上夸克带电 为 2 3 e ,下夸克带电为 1 3 e ,e 为电子所带电量的大小,如果质子是由三个夸克组成的, 且各个夸克之间的距离都为l , 151.5 10l m  ,试计算质子内相邻两个夸克之间的静电 力(库仑力) 14.(14 分)一质量为 m 的小球,以初速度 0v 沿水平方向射出,恰好垂直地射到一倾角为 030 的固定斜面上,并立即反方向弹回。已知反弹速度的大小是入射速度大小的 3 4 ,求在 碰撞中斜面对小球的冲量大小 15.(15 分)如图,在水平面上有两 条平行导电导轨 MN、PQ,导 轨间距离为l ,匀强磁场垂直 于导轨所在的平面(纸面)向 里,磁感应强度的大小为 B, 两根金属杆 1、2 摆在导轨上, 与导轨垂直,它们的质量和电 阻分别为 1 2m m、 和 1R 2、R , 两杆与导轨接触良好,与导轨间的动摩擦因数为  ,已知:杆 1 被外力拖动,以恒定 的 速度 0v 沿导轨运动;达到稳定状态时,杆 2 也以恒定速度沿导轨运动,导轨的电阻可 忽 略,求此时杆 2 克服摩擦力做功的功率。 M 2 1 N 0v P Q 16.(16 分)某颗地球同步卫星正下方的地球表面上有一观察者,他用天文望远镜观察被太 阳光照射的此卫星,试问,春分那天(太阳光直射赤道)在日落 12 小时内有多长时间 该观察者看不见此卫星?已知地球半径为 R,地球表面处的重力加速度为 g,地球自转周 期为 T,不考虑大气对光的折射。 17.(16 分)图中,轻弹簧的一端固定,另一端与滑块 B 相连,B 静止在水平导轨上,弹簧 处在原长状态。另一质量与 B 相同滑块 A,从导轨上的 P 点以某一初速度向 B 滑行, 当 A 滑过距离 1l 时,与 B 相碰,碰撞时间极短,碰后 A、B 紧贴在一起运动,但互不 粘连。已知最后 A 恰好 返回出发点 P 并停止。 滑块 A 和B 与导轨的滑 B A 2l 1l 动摩擦因数都为  ,运动过程中弹簧最大形变量为 2l ,求 A 从 P 出发时的初速度 0v 。 18.(17 分)如图,真空室内存在匀强磁场,磁场方向垂直于纸面向里,磁感应强度的大小 B=0.60T,磁场内有一块平面感光板 ab,板面与磁场方向平行,在距 ab 的距离 16l cm 处,有一个点状的 放射源 S,它向各 个方向发射 粒子, 粒子的速度都是 63.0 10 /v m s  ,已知 粒子的电荷 与质量之比 75.0 10 /q C kgm   ,现只 考虑在图纸平面中运动的 粒子,求 ab 上被 粒子打中的区域的长度。 a b l S · 2004 年全国普通高等学校招生统一考试(广东卷) 物理参考答案 I 卷共 40 分,每小题 4 分. 每小题全选对的给 4 分,选不全的给 2 分,有选错的给 0 分,不 答的给 0 分. 1.B 2.AD 3.BD 4.D 5.C 6.B 7.A 8.D 9.C 10.ABC II 卷共 8 道题,共 110 分. 11.在白纸上另画一条与 y 轴正方向的夹角较小的直线 OA,把大头针 P1、P2 竖直地插在所 画的直线上,直到在 y<0 的区域内透过玻璃砖能看到 P1、P2 的像. 插上 P3 后,P3 刚好能挡住 P1、P2 的像.   sin sinn 12.(1)①K1 断开,K2 接到 a 端,记下电压表的读数 U1;②K2 仍接到 a 端,闭合 K1,记 下电压表的读数 U2;③K1 仍闭合,K2 接到 b 端,记下电压表的读数 U3. (2)U1 RUU U 32 3  13.质子带电为+e,所以它是由 2 个上夸克和 1 个下夸克组成的.按题意,三个夸克必位于 等边三角形的三个顶点处.这时上夸克与上夸克之间的静电力应为 2 2 2 9 43 2 3 2 l ek l ee kFm    ① 代入数值,得 mF =46N,为斥力. ② 上夸克与下夸克之间的静电力为 2 2 2 9 23 2 3 1 l ek l ee kFnd    ③ 代入数值,得 ndF =23N,为吸力. ④ 14.小球在碰撞斜面前做平抛运动.设刚要碰撞斜面时小球速度为 v .由题意, v 的方向与竖 直线的夹角为 30°,且水平分量仍为 v 0,如右图.由此得 v =2 v 0 ① 碰撞过程中,小球速度由 v 变为反向的 .4 3 v 碰撞时间极短,可不 计重力的冲量,由动量定理,斜面对小球的冲量为 mvvmI  )4 3( ② 由①、②得 02 7 mvI  ③ 15.解法一:设杆 2 的运动速度为 v,由于两杆运动时,两杆间和导轨构成的回路中的磁通 量发生变化,产生感应电动势 )( 0 vvBl  ① 感应电流 21 RRI   ② 杆 2 作匀速运动,它受到的安培力等于它受到的摩擦力, gmBlI 2 ③ 导体杆 2 克服摩擦力做功的功率 gvmP 2 ④ 解得 )]([ 2122 2 02 RR lB gmvgmP   ⑤ 解法二:以 F 表示拖动杆 1 的外力,以 I 表示由杆 1、杆 2 和导轨构成的回路中的电流,达 到稳定时,对杆 1 有 01  BIlgmF  ① 对杆 2 有 02  gmBIl  ② 外力 F 的功率 0FvPF  ③ 以 P 表示杆 2 克服摩擦力做功的功率,则有 0121 2 )( gvmRRIPP F  ④ 由以上各式得 )]([ 212202 RR lB gmvgmP g   ⑤ 16.设所求的时间为 t,用 m、M 分别表示卫星和地球的质量,r 表示卫星到地心的距离. 有 2 2 )2( Tmr r mMG  ① 春分时,太阳光直射地球赤道,如图所示,图中圆 E 表示赤道,S 表示卫星,A 表示观 察者,O 表示地心. 由图可看出当卫星 S 绕地心 O 转到图示位置以后(设地球自转是沿图中 逆时针方向),其正下方的观察者将看不见它. 据此再考虑到对称性,有 Rr sin ② Tt   2 2 ③ g R MG 2 ④ 由以上各式可解得 3 1 2 2 )4arcsin( gT RTt   ⑤ 17.令 A、B 质量皆为 m,A 刚接触 B 时速度为 1v (碰前),由功能关系,有 1 2 1 2 0 2 1 2 1 mglmvmv  ① A、B 碰撞过程中动量守恒,令碰后 A、B 共同运动的速度为 .2v 有 21 2mvmv  ② 碰后 A、B 先一起向左运动,接着 A、B 一起被弹回,在弹簧恢复到原长时,设 A、B 的共同速度为 3v ,在这过程中,弹簧势能始末两态都为零,利用功能关系,有 )2()2()2(2 1)2(2 1 2 2 3 2 2 lgmvmvm  ③ 此后 A、B 开始分离,A 单独向右滑到 P 点停下,由功能关系有 1 2 32 1 mglmv  ④ 由以上各式,解得 )1610( 210 llgv   ⑤ 18.参考解答:  粒子带正电,故在磁场中沿逆时针方向做匀速圆周运动, 用 R 表示轨道半径,有 R vmqvB 2  ① 由此得 Bmq vR )/(  代入数值得 R=10cm 可见,2R>l>R. 因朝不同方向发射的 粒子的圆轨迹都过 S,由此可知,某一圆轨迹在图中 N 左侧与 ab 相切,则此切点 P1 就是 粒子能打中的左侧最远点.为定出 P1 点的位置,可作平行于 ab 的直线 cd,cd 到 ab 的距离为 R,以 S 为圆心,R 为半径,作弧交 cd 于 Q 点,过 Q 作 ab 的垂线,它与 ab 的交点即为 P1. 22 1 )( RlRNP  ② 再考虑 N 的右侧。任何 粒子在运动中离 S 的距离不可能超过 2R,以 2R 为半径、S 为圆心作圆,交 ab 于 N 右侧的 P2 点,此即右侧能打到的最远点. 由图中几何关系得 22 2 )2( lRNP  ③ 所求长度为 2121 NPNPPP  ④ 代入数值得 P1P2=20cm ⑤ 2004 年全国普通高等学校招生全国统一考试 物理(上海卷) 本试卷分(选择题)和(非选择题)两部分,满分 150 分。考试用时 120 分钟。 第Ⅰ卷(共 60 分) 一、(40 分)选择题:本大题共 8 小题,每小题 5 分。每小题给出的四个答案中,至少有一 个是正确的,把正确答案全选出来,并将正确答案前面的字母填写在题后的方括号内。 每小题全选对的得 5 分;选对但不全,得部分分;有选错或不答的。得 0 分。填写在括 号外的字母,不作为选出的答案。 1.下列说法正确的是 A.光的干涉和衍射现象说明光具有波动性。 B.光的频率越大,波长越长。 C.光的波长越大,光子的能量越大。 D.光在真空中的传播速度为 3.00×108m/s 2.下列说法中正确的是 A.玛丽·居里首先提出原子的核式结构学说。 B.卢瑟福在 粒子散射实验中发现了电子。 C.查德威克在原子核人工转变的实验中发现了中子。 D.爱因斯坦为解释光电效应的实验规律提出了光子说。 3.火星有两颗卫星,分别是火卫一和火卫二,它们的轨道近似为圆。已知火卫一的周期为 7 小时 39 分。火卫二的周期为 30 小时 18 分,则两颗卫星相比 A.火卫一距火星表面较近。 B.火卫二的角速度较大 C.火卫一的运动速度较大。 D.火卫二的向心加速度较大。 4.两圆环 A、B 置于同一水平面上,其中 A 为均匀带电绝缘环,B 为导体环,当 A 以如图 所示的方向绕中心转动的角速度发生变化时,B 中产生如图所示方向的感应电流。则 A.A 可能带正电且转速减小。 B.A 可能带正电且转速增大。 C.A 可能带负电且转速减小。 D.A 可能带负电且转速增大。 5.物体 B 放在物体 A 上,A、B 的上下表面均与斜面平行(如图),当两者以相同的初速度 靠惯性沿光滑固定斜面 C 向上做匀减速运动时, A.A 受到 B 的摩擦力沿斜面方向向上。 B.A 受到 B 的摩擦力沿斜面方向向下。 C.A、B 之间的摩擦力为零。 D.A、B 之间是否存在摩擦力取决于 A、B 表面的性质。 6.某静电场沿 x 方向的电势分布如图所示,则 A.在 0—x1 之间不存在沿 x 方向的电场。 B.在 0—x1 之间存在着沿 x 方向的匀强电场。 C.在 x1—x2 之间存在着沿 x 方向的匀强电场。 D.在 x1—x2 之间存在着沿 x 方向的非匀强电场。 7.光滑水平面上有一边长为 l 的正方形区域处在场强为 E 的匀强电场中,电场方向与正方 形一边平行。一质量为 m、带电量为 q 的小球由某一边的中点,以垂直于该边的水平初 速 v0 进入该正方形区域。当小球再次运动到该正方形区域的边缘时,具有的动能可能 为 A.0 B. .2 1 2 1 2 0 qElmv  C. .2 1 2 0mv D. .3 2 2 1 2 0 qElmv  8.滑块以速率 v1 靠惯性沿固定斜面由底端向上运动,当它回到出发点时速率为 v2,且 v2< v1, 若滑块向上运动的位移中点为 A,取斜面底端重力势能为零,则 A.上升时机械能减小,下降时机械增大。 B.上升时机械能减小,下降时机械能也减小。 C.上升过程中动能和势能相等的位置在 A 点上方。 D.上升过程中动能和势能相等的位置在 A 点下方。 二、(20 分)填空题,本大题共 5 小题,每小题 4 分,答案写在题中横线上的空白处或指定 位置,不要求写了演算过程。 9.在光电效应实验中,如果实验仪器及线路完好,当光照射到光电管上时,灵敏电流计中 没有电流通过,可能的原因是: . 10.在光滑水平面上的 O 点系一长为 I 的绝缘细 线,线的另一端系一质量为 m、带电量为 q 的 小球.当沿细线方向加上场强为 E 的匀强电场后, 小球处于平衡状态.现给小球一垂直于细线的初 速度 v0,使小球在水平面上开始运动.若 v0 很小, 则小球第一次回到平衡位置所需时间为 . 11.利用扫描隧道显微镜(STM)可以得到物质表面原子排列的图象,从而可以研究物质的 构成规律,下面的照片是一些晶体材料表面的 STM 图象,通过观察、比较,可以看到 这些材料都是由原子在空间排列而构成的,具有一定的结构特征,则构成这些材料的原 子在物质表面排列的共同特点是 (1) ; (2) ;……. 12.两个额定电压为 220V 的白炽灯 L1 和 L2 的 U—I 特性曲线如图所示.L2 的额定功率约为 W;现将 L1 和 L2 串联后接在 220V 的电源上,电源内阻忽略不计.此时 L2 的实际功率约为 W. 13.A、B 两波相向而行,在某时刻的波形与位置如图所示.已知波的传播速度为 v,图中标 尺每格长度为 l,在图中画出又经过 t=7l/v 时的波形. 第Ⅱ卷(共 90 分) 三、(30 分)实验题。 14.(5 分)用打点计时器研究物体的自由落体运动,得到如图一段纸带,测得 AB=7.65cm, BC=9.17cm. 已知交流电频率是 50Hz,则打 B 点时物体的瞬时速度为 m/s. 如果实验测出的重力加速度值比公认值偏小,可能的原因是 . 15.(4 分)在测定一节干电池(电动势约为 1.5V,内阻约为 2Ω)的电动势和内阻的实验 中,变阻器和电压表各有两个供选:A 电压表量程为 15V,B 电压表量程为 3V,A 变 阻器为(20Ω,3A),B 变阻器为(500Ω,0.2A) 电压表应该选 (填 A 或 B),这是因为 . 变阻器应该选 (填 A 或 B),这是因为 . 16.(5 分)下图为一测量灯泡发光强度的装置.AB 是一个有刻度的底座,两端可装两个灯 泡,中间带一标记线的光度计可在底座上移动,通过观察可以确定两边灯泡在光度计上 的照度是否相同,已知照度与灯泡的发光强度成正比、与光度计到灯泡的距离的平方成 反比.现有一个发光强度为 I0 的灯泡 a 和一个待测灯泡 b.分别置于底座两端(如图). (1)怎样测定待测灯泡的发光强度 Ix? . (2)简单叙述一个可以减小实验误差的方法。 . 17.(6 分)一根长为约 30cm、管内载面积为 S=5.0×10-6m2 的玻璃管下端有 一个球形小容器,管内有一段长约 1cm 的水银柱.现在需要用比较准确的 方法测定球形小容器的容积 V.可用的器材有:刻度尺(量程 500mm)、 温度计(测量范围 0—100℃)、玻璃容器(高约 30cm,直径约 10cm)、 足够多的沸水和冷水. (1)简要写出实验步骤及需要测量的物理量; (2)说明如何根据所测得的物理量得出实验结果. 18.(10 分)小灯泡灯丝的电阻会随温度的升高而变大.某同学为研究这一现象,用实验得到 如下数据(I 和 U 分别表示小灯泡上的电流和电压): I(A) 0.12 0.21 0.29 0.34 0.38 0.42 0.45 0.47 0.49 0.50 U(V) 0.20 0.40 0.60 0.80 1.00 1.20 1.40 1.60 1.80 2.00 (1)在左下框中画出实验电路图. 可用的器材有:电压表、电流表、滑线变阻器(变化范 围 0—10Ω)、电源、小灯泡、电键、导线若干. (2)在右图中画出小煤泡的 U—I 曲线. (3)如果第 15 题实验中测得电池的电动势是 1.5V,内阻是 2.0Ω.问:将本题中的灯泡接在 该电池两端,小灯泡的实际功率是多少?(简要写出求解过程;若需作图,可直接画在 第(2)小题的方格图中) 四、(60 分)计算题。 19.(8 分)“真空中两个静止点电荷相距 10cm.它们之间相互作用力大小为 9×10-4N.当它们 合在一起时,成为一个带电量为 3×10-8C 的点电荷.问原来两电荷的带电量各为多少? 某同学求解如下: 根据电荷守恒定律: aCqq  8 21 103 (1) 根据库仑定律: bCCFk rqq      21524 9 222 21 101109 109 )1010( 以 12 / qbq  代入(1)式得: 01 2 1  baqq 解得 Cbaaq )104109103(2 14(2 1 151632 1   根号中的数值小于 0,经检查,运算无误.试指出求解过程中的问题并给出正确的解答. 20.(12 分)如图所示,一端封闭、粗细均匀的薄壁玻璃管开口向下竖直插在装有水银的水 银槽内,管内封闭有一定质量的空气,水银槽的截而积上下相同,是玻璃管截面积的 5 倍,开始时管内空气柱长度为 6cm,管内外水银面高度差为 50cm.将玻璃管沿竖直方向 缓慢上移(管口未离开槽中水银),使管内外水银而高度差变成 60cm.(大气压强相当 于 75cmHg)求: (1)此时管内空气柱的长度; (2)水银槽内水银面下降的高度; 21.(12 分)滑雪者从 A 点由静止沿斜面滑下,沿一平台后水平飞离 B 点,地面上紧靠平 台有一个水平台阶,空间几何尺度如图所示,斜面、平台与滑雪板之间的动摩擦因数为 μ. 假设滑雪者由斜面底端进入平台后立即沿水平方向运动,且速度大小不变.求: (1)滑雪者离开 B 点时的速度大小; (2)滑雪者从 B 点开始做平抛运动的水平距离 s. 22.(14 分)水平面上两根足够长的金属导轨平行固定放置,问距为 L,一端通过导线与阻 值为 R 的电阻连接;导轨上放一质量为 m 的金属杆(见右上图),金属杆与导轨的电阻 忽略不计;均匀磁场竖直向下.用与导轨平行的恒定拉力 F 作用在金属杆上,杆最终将 做匀速运动.当改变拉力的大小时,相对应的匀速运动速度 v 也会变化,v 与 F 的关系如 右下图.(取重力加速度 g=10m/s2) (1)金属杆在匀速运动之前做什么运动? (2)若 m=0.5kg,L=0.5m,R=0.5Ω;磁感应强度 B 为多大? (3)由 v—F 图线的截距可求得什么物理量?其值为多少? 23.(14 分)有人设计了一种新型伸缩拉杆秤.结构如图,秤杆的一端固定一配重物并悬一挂 钩,秤杆外面套有内外两个套筒,套筒左端开槽使其可以不受秤纽阻碍而移动到挂钩所 在位置(设开槽后套筒的重心仍在其长度中点位置),秤杆与内层套筒上刻有质量刻度. 空载(挂钩上不挂物体,且套筒未拉出)时,用手提起秤纽,杆秤恰好平衡,当物体挂 在挂钩上时,往外移动内外套筒待测物体的质量.已知秤杆和两个套筒的长度均为 16cm,套筒可移出的最大距离为 15cm,秤纽到挂钩的距离为 2cm,两个套筒的质量均 为 0.1kg.取重力加速度 g=10m/s2, (1)当杆秤空载平衡时,秤杆、配重物及挂钩所受重力相对秤纽的合力矩; (2)当在秤钩上挂一物体时,将内套筒向右移动 5cm,外套筒相对内套筒向右移动 8cm, 杆秤达到平衡,物体的质量多大? (3)若外层套筒不慎丢失,在称某一物体时,内层套筒的左端在读数为 1 千克处杆秤恰好 平衡,则该物体实际质量多大? 2004 年全国普通高等学校招生统一考试(上海卷) 物理参考答案 一、选择题 1.AB 2.CD 3.AC 4.BC 5.C 6.AC 7.ABC 8.BC 二、填空题 9.入射光波长太大(或反向电压太大) 10. qE ml 11.在确定方向上原子有规律地排列 在不同方向上原子的排列规律一般不同 原子排列具有一定对称性等 12.99,17.5 13. 三、实验题 14.2.10,下落过程中有存在阻力等 15.B,A 电压表量程过大,误差较大;A、B 变阻器额定电流过小且调节不便 16.(1)接通电源,移动光度计使两边的照度相同,测出距离 r1 和 r2,即可得待测灯泡的发 光强度 02 1 2 2 I r rI x  (2)测量多次,求平均等 17.(1)将水银柱以下的玻璃浸没在水中,改变水温,用温度计测得若干组(或两组)不同 水温(即气体温度)T 和气柱长度 x 的值 (2)方法一:气体作等压变化 CT xSV  即 VCTxS  ,作 TxS  图,截距的绝对 值即为 V 方法二:测两组数据 2211 , CTVSxCTVSx  可解得 12 1221 TT xTxTV   S 18.(1)见下图 (2)见右图 (3)作出 U= Ir 图线,可得小灯泡工作电流为 0.35 安,工作电压为 0.80 伏,因此小灯 泡实际功率为 0.28 瓦 四、计算题 19.题中仅给出相互作用力的大小,两点电荷可能异号,按电荷异号计算. 由 .101.103 215 21 8 21 bCqqaCqq   得 01 2 1  baqq ① 由此解得 Cq 8 1 105  ② Cq 8 2 102 ③ 20.(1)玻璃管内的空气作等温变化 220110 )()( lpgHplpgHp  ① 1 20 10 2 lpgHp pgHpl   ② )(10.006.06075 5075 m  ③ (2)设水银槽内水银面下降 x ,水银体积不变 xSHS  21 ④ )( 12 2 1 HHS Sx  ⑤ )(02.0)50.060.0(5 1 m ⑥ 21.(1)设滑雪者质量为 m,斜面与水平面夹角为 ,滑雪者滑行过程中克服摩擦力做功 mgLsLmgsmgW   )cos(cos ① 由动能定理 2 2 1)( mvmgLhHmg   ② 离开 B 点时的速度 )(2 LhHgv  ③ (2)设滑雪者离开 B 点后落在台阶上 hvtsgth 22 1 2 11 2 1  可解得 )(21 LhHhs  ④ 此时必须满足 hLH 2  ⑤ 当 hLH 2  ⑥ 时,滑雪者直接落到地面上, 22 2 22 1 vtsgth  可解得 )(22 LhHhs  ⑦ 22.(1)变速运动(或变加速运动、加速度减小的加速运动,加速运动)。 (2)感应电动势 vBL ① 感应电流 RI  ② 安培力 R LvBIBLFM 22  ③ 由图线可知金属杆受拉力、安增力和阻力作用,匀速时合力为零。 fR LvBF  22 ④ )(22 fF LB Rv  ⑤ 由图线可以得到直线的斜率 k=2, 12  kL RB (T) ⑥ (3)由直线的截距可以求得金属杆受到的阻力 f,f=2(N) ⑦ 若金属杆受到的阻力仅为动摩擦力,由截距可求得动摩擦因数 4.0 ⑧ 23.(1)套筒不拉出时杆秤恰好平衡,此时两套筒的重力相对秤纽的力矩与所求的合力矩相 等,设套筒长度为 L,合力矩 )2/(2 dLmgM  ① )(12.0)02.008.0(101.02 mN  ② (2)力矩平衡 )( 2111 xxmgmgxgdm  ③ md xxm 21 1 2  ④ )(9.01.002.0 08.005.02 kg ⑤ (3)正常称衡 1kg 重物时,内外两个套筒可一起向外拉出 x 力矩平衡 xmggdm  22 ⑥ )(1.002.01.02 1 2 2 mdm mx  ⑦ 外层套筒丢失后称物,此时内套筒左端离秤纽距离为 mdx 08.0 力矩平衡 )2/(2 LdxmgMgdm  ⑧ gd MLdxd mm  )2/(2 )(2.06.0)08.008.0(02.0 1.0 kg ⑨